You are on page 1of 274

MINISTRY OF EDUCATION AND SCIENCE

OF THE REPUBLIC OF KAZAKHSTAN

INTERNATIONAL INFORMATION TECHNOLOGY UNIVERSITY

B.Sh. Kulpeshov

SHORT COURSE
OF PROBABILITY THEORY
AND MATHEMATICAL STATISTICS

Almaty
2017
UDC519.2(075.8)
LBC 22.17я73
К91

Kulpeshov B.Sh.
К91 Short course of probability theory and mathematical statistics. –
Almaty, 2017 – 274 pages.
ISBN 978-601-7911-10-2

The present manual has been composed by using the lectures read by the
author at the International Information Technology University within last five
academic years for students of technical specialties of the English branch.
The present course studies elements of probability theory and mathematical
statistics: random events, geometric probabilities, combinatorial analysis,
independence of events, conditional probability, theorems of addition and
multiplication, random variables, mathematical expectation, dispersion, basic
laws of distribution, the law of large numbers and limit theorems, variation
series, sampling, methods of estimation, and testing of statistical hypotheses.
The course has been prepared for teaching Probability Theory and Mathematical
Statistics during one semester (15 weeks).
UDC519.2(075.8)
LBC 22.17я73
ISBN 978-601-7911-10-2
©Kulpeshov B.Sh., 2017


INTRODUCTION
Events (phenomena) observed by us could be subdivided on the
following three types: reliable, impossible and random.
A reliable (universal) event is an event that necessarily will happen if a
certain set of conditions S holds. For example, if a vessel contains water
with a normal atmospheric pressure and temperature 20 degrees, the
event «water in a vessel is in a liquid state» is reliable. In this example
the given atmospheric pressure and temperature of water make the set
of conditions S.
An impossible (null) event is an event that certainly will not happen if
the set of conditions S holds. For example, the event «water in a vessel
is in a rigid state» certainly will not happen if the set of conditions of
the previous example holds.
A random event is an event that can either occur, or not to occur for
holding the set of conditions S. For example, if a coin is tossed it can land
on one of two sides: heads or tails. Therefore, the event «the coin lands
on heads» is random. Each random event, in particular an appearance of
heads, is the consequence of a functioning very many random reasons
(in our example: the force with which the coin tossing, the form of the
coin and many others). It is impossible to take into account an influence
on result of all these reasons, as their number is very great and laws
of their functioning are unknown. Therefore, probability theory does
not pose the problem to predict that a single event whether or not will
happen – it simply is unable to solve this problem.
We have another picture if we consider random events that can multiply
be observed for holding the same conditions S, i.e. if the speech goes on
mass homogeneous random events. It appears that a rather large number
of homogeneous random events independently from their concrete
nature are subordinated to definite regularities, namely probability
regularities. Probability theory studies these regularities. Thus, the
subject of probability theory is studying probability regularities of mass
homogeneous random events.
Methods of probability theory are widely applied in various branches
of natural sciences and techniques: theory of reliability, theory of mass

3


service, theoretical physics, geodesy, astronomy, theory of shooting,


theory of mistakes of observation, theory of automatic control, general
theory of communication and many others theoretical and applied
sciences. Probability theory serves also to substantiate Mathematical
Statistics that in turn is used at planning and organizing a manufacture,
at analysis of technological processes, and for many other purposes.
All life a person has to make decisions: in personal sphere (in which
university or college to enter, with whom to communicate; how to
study); in public (to attend evenings, theatres, meetings, assemblies,
elections); in industrial (determining factors essentially influencing on
productivity, quality of materials and etc.); in scientific (promotion and
checking scientific hypotheses).
Decision-making usually pursues one of the following purposes:
forecasting of a future state of a process (an object); control (i.e. how
should to change certain parameters of an object in order that other
parameters have taken on a desirable value); an explanation of internal
structure of an object.
Usually decision-making is preceded with an analysis of known data
(based on previous experience, common sense, intuition, and etc.).
Aspiring to see and prove regularities in uncertain processes, the
humanity has developed the whole arsenal of methods that refer to as
mathematical statistics (applied statistics or data analysis).
Mathematical statistics is a section of mathematics in which
mathematical methods of ordering, processing and using statistical data
for scientific and practical conclusions are studied.
Abraham Wald (1902-1950) spoke that «mathematical statistics is
theory of decision-making in conditions of uncertainty».
In essence, mathematical statistics gives a unique, mathematically
proved apparatus for solving problems of control and forecasting at
absence of obvious regularities (presence of randomness) in investigated
processes.
It is wonderful that the science begun with consideration of gambles
was fated to become the major object of human knowledge …
Pierre-Simon Laplace (1749-1827)

4
Basic Concepts of Probability Theory

LECTURE 1

Basic Concepts of Probability Theory


Hereinafter, instead of speaking «the set of conditions S holds» we
shall speak briefly: «the trial has been made». Thus, an event will be
considered as a result of a trial.
Example. A shooter shoots in a target subdivided into four areas. One
shot is the trial. Hit in a certain area of the target is an event.
Example. There are colour balls in an urn. One takes at random one
ball from the urn. An extracting a ball from the urn is the trial. An
appearance of a ball of a certain colour is an event.
Events are incompatible if an appearance of one of them excludes
an appearance of other events in the same trial. Otherwise, they are
compatible.
Example. An item is extracted at random from a box with items. An
appearance of a standard item excludes an appearance of a non-standard
item. The events «a standard item has appeared» and «a non-standard
item has appeared» are incompatible.
Example. A coin is tossed. An appearance of «heads» excludes an
appearance of «tails». The events «heads have appeared» («the coin
lands on heads») and «tails have appeared» («the coin lands on tails»)
are incompatible.
For example, a landing two different prizes under only one ticket of a
lottery are incompatible events, and a landing the same prizes under
two tickets are compatible events. Obtaining marks «excellent»,
«good» and «satisfactory» by a student at an exam in one discipline are
incompatible events and an obtaining the same marks at exams in three
disciplines are compatible events.
Some events form a complete group if in result of a trial at least one
of them will appear. In other words, the appearance of at least one of
events of a complete group is a reliable event.

5
LECTURE 1

In particular, if events forming a complete group are pairwise


incompatible then in result of a trial one and only one of these events
will appear.
Example. Two tickets of a money-thing lottery have been bought. One
necessarily will happen one and only one from the following events:
«a landing a prize on the first ticket and a non-landing a prize on the
second», «a landing a prize on both tickets», «a non-landing a prize on
the first ticket and a landing a prize on the second», «a non-landing a
prize on both tickets». These events form a complete group of pairwise
incompatible events.
Example. A shooter has made one shot in a target. One necessarily
will happen one from the following two events: hit, miss. These two
incompatible events form a complete group.
Events are equally possible if there is reason to consider that none of
them is more possible (probable) than other.
Example. An appearance of heads and an appearance of tails at tossing a
coin are equally possible events. Appearances of «one», «two», «three»,
«four», «five» or «six» on a tossed die are equally possible events.
Several events are uniquely possible if at least one of them will necessarily
happen as a result of a trial. For example, the events consisting in that
a family with two children has: A – «two boys», B – «one boy and one
girl» and C – «two girls» are uniquely possible.

Classical Definition of Probability


Example. Let an urn contain 6 identical, carefully shuffled balls, and
2 of them are red, 3 – blue and 1 – white. Obviously, the possibility to
take out at random from the urn a colour ball (i.e. red or blue) is more
than the possibility to extract a white ball.
Whether it is possible to describe this possibility by number? It appears
it is possible. This number is said to be the probability of an event
(appearance of a colour ball). Thus, the probability is the number
describing the degree of possibility of an appearance of an event.

6
Classical Definition of Probability

Let the event A be an appearance of a colour ball. We call each of


possible results of a trial (the trial is an extracting a ball from the urn)
by elementary event. We denote elementary events by ω1, ω2, ω3 and et
cetera. In our example the following 6 elementary events are possible:
ω1 – the white ball has appeared; ω2, ω3 – a red ball has appeared; ω4,
ω5,ω6 – a blue ball has appeared. These events form a complete group
of pairwise incompatible events (it necessarily will be appeared only
one ball) and they are equally possible (a ball is randomly extracted; the
balls are identical and carefully shuffled).
We call those elementary events in which the event interesting for us
occurs, as favorable to this event. In our example the following 5 events
favor to the event A (appearance of a colour ball): ω2, ω3, ω4, ω5, ω6.
In this sense the event A is subdivided on some elementary events; an
elementary event is not subdivided into other events. It is the distinction
between the event A and an elementary event.
The ratio of the number of favorable to the event A elementary events
to their total number is said to be the probability of the event A and it
is denoted by P(A). In the considered example we have 6 elementary
events; 5 of them favor to the event А. Therefore, the probability that the
taken ball will be colour is equal to P(A) = 5/6. This number gives such
a quantitative estimation of the degree of possibility of an appearance
of a colour ball which we wanted to find.
The probability of the event A is the ratio of the number of favorable
elementary events for this event to their total number of all equally
possible incompatible elementary events forming a complete group.
Thus, the probability of the event A is determined by the formula:
m
P (A) =
n
where m is the number of elementary events favorable to A; n is the
number of all possible elementary events of a trial. Here we suppose
that elementary events are incompatible, equally possible and form a
complete group.
The definition of probability implies the following its properties:

7
LECTURE 1

Property 1. The probability of a reliable event is equal to 1.


In fact, if an event is reliable then each elementary event of a trial favors
to the event. In this case m = n and consequently P(A) = m/n = n/n = 1.
Property 2. The probability of an impossible event is equal to 0.
Indeed, if an event is impossible then none of elementary events of a
trial favors to the event. In this case m = 0 and consequently P(A) =
m/n = 0/n = 0.
Property 3. The probability of a random event is the positive number
between 0 and 1.
In fact, a random event is favored only part of the total number of
elementary events of a trial. In this case 0 < m < n; then 0 < m/n < 1
and consequently 0 < P(A) < 1.
Thus, the probability of an arbitrary event A satisfies the double
inequality:
0 ≤ P(A) ≤ 1

Relative Frequency
The relative frequency (statistical probability) of an event is the ratio
of the number of trials, in which the event has appeared, to the total
number of actually made trials.
Thus, the relative frequency of the event A is defined by the formula:
m
W (A) =
n
where m is the number of appearances of the event, n is the total number
of trials.
Comparing the definitions of probability and relative frequency, we
conclude: the definition of probability does not demand that the trials
should be made actually; the definition of relative frequency assumes
that the trials were made actually. In other words, the probability is

8
Geometric Probabilities

calculated before an experiment, and the relative frequency – after an


experiment.
Example. The quality department has detected 3 non-standard items in a
group consisting of 80 randomly selected items. The relative frequency
of appearance of non-standard items is W(A) = 3/80.
Example. There have been made 24 shots in a target, and 19 hits were
registered. The relative frequency of hit in the target is W(A) = 19/24.
The long observations have shown that if experiments are made in
identical conditions, in each of which the number of trials is rather
great, the relative frequency has a stability property. This property is
that for different experiments the relative frequency is changed a little
(the less changes, the more trials were made), oscillating about some
constant number. There was found out that this constant number is the
probability of appearance of the event.
Thus, if the relative frequency is established by a practical experiment,
the obtained number can be accepted for approximate value of
probability.

Geometric Probabilities
To overcome defect of the classical definition of probability consisting
that it is inapplicable to trials with infinite number of events (outcomes)
enter geometric probabilities – the probability of hit of a point in area
(segment, part of a plane and etc.).
Let a segment l be a part of a segment L. A point is set (thrown) at random
in the segment L. It means that the following suppositions hold: the
thrown point can appear in any point of the segment L, the probability
of hit of the point in the segment l is proportional to the length of this
segment and does not depend on its disposition concerning the segment
L. In these suppositions the probability of hit of the point in the segment
l is determined by the equality

P = the length of l / the length of L

9
LECTURE 1

Example. A point B(x) is thrown at random in a segment OA of the


length L of the numeric axis Ox. Find the probability that the smaller
of the segments OB and BA has the length more than L/3. It is assumed
that the probability of hit of a point in the segment is proportional to
the length of the segment and does not depend on its disposition on the
numeric axis.
Solution: Let’s divide the segment OA by points C and D on three equal
parts. The request of the problem will be executed if the point B(x) will
hit in the segment CD of the length L/3. The required probability P =
(L/3)/L = 1/3.
Let a flat figure g be a part of a flat figure G. A point is thrown at random
in the figure G. It means that the following suppositions hold: the thrown
point can appear in any point of the figure G, the probability of hit of the
thrown point in the figure g is proportional to the area of this figure and
does not depend on both its disposition concerning the figure G and the
form of g. In these suppositions the probability of hit of the point in the
figure g is determined by the equality

P = the area of g / the area of G

Example. Two concentric circles of which the radiuses are 5 and 10 cm


respectively are drawn on the plane. Find the probability that the point
thrown at random in the large circle will hit in the ring formed by the
constructed circles. It is assumed that the probability of hit of a point in
a flat figure is proportional to the area of this figure and does not depend
on its disposition concerning the large circle.
Solution: The area of the ring (the figure g)

Sg = πR2 – πr2 = π (102 – 52) = 75π

The area of the large circle (the figure G)

Sg = π∙R2 = π∙102 = 100π.

10
Glossary

The required probability


75r =
P= 0, 75
100r

Glossary
probability theory – теория вероятностей
reliable event – достоверное событие
random event – случайное событие
vessel – сосуд; trial (experiment) – испытание (опыт, эксперимент)
urn – урна; heads or tails? – орел или решка?
at random – наудачу; to land a prize – получить приз
complete group of events – полная группа событий
equally possible events – равновозможные события
uniquely possible events – единственно возможные события
ace – очко (при игре в кости); die – кость (игральная)
dice – игра в кости, кости; hit – попадание; miss – промах
to shuffle – перемешивать; relative frequency – относительная
частота
favorable case – благоприятствующий (благоприятный) случай
mass homogeneous events – массовые однородные события

Exercises for Seminar 1


1.1. There are 50 identical items (and 5 of them are painted) in a box.
Find the probability that the first randomly taken item will be painted.
1.2. A die is tossed. Find the probability that an even number of aces
will appear.
1.3. Participants of a toss-up pull a ticket with numbers from 1 up to 100

11
LECTURE 1

from a box. Find the probability that the number of the first randomly
taken ticket does not contain the digit 5 (toss-up – жеребьевка; to pull
– тянуть; ticket – жетон).
1.4. In a batch of 100 items the quality department has found out 5
non-standard items. What is the relative frequency of appearance of
non-standard items equal to? (batch – партия)
1.5. At shooting by a rifle the relative frequency of hit in a target has
appeared equal to 0,85. Find the number of hits if 120 shots were made
(a rifle – винтовка).
1.6. One die is randomly taken from a carefully hashed full set of 28
dice of domino. Find the probability that the second randomly taken die
can be put to the first if the first die:
a) is a double;
b) is not double (to hash – перемешивать; double – дубль).
The answer: a) 2/9; b) 4/9.
1.7. A point B(x) is randomly put in a segment OA of the length L of the
numeric axis Ox. Find the probability that the smaller of the segments
OB and BA has the length which is less than L/4. It is supposed that the
probability of hit of a point in the segment is proportional to its length
and does not depend on its location on the numeric axis.
1.8. Two persons have agreed to meet in a certain place between 18
and 19 o’clock and have agreed that a person come the first waits for
another person within 15 minutes then leaves. Find the probability of
their meeting if arrival of everyone within the specified hour can take
place at any time and the moments of arrival are independent.
The answer: 7/16.
1.9. Two dice are tossed. Find the probability that:
a) the same number of aces will appear on both dice;
b) two aces will appear at least on one die;
c) the sum of aces will not exceed 6 (to exceed – превышать).

12
Exercises for Homework 1

1.10. A point is randomly taken inside a circle of radius 5. Find the


probability that the point will be inside a proper (equilateral) triangle
entered in the circle (a proper triangle – правильный треугольник;
equilateral – равносторонний).

The answer: 3 3 .
4r

Exercises for Homework 1


1.11. A die is tossed. Find the probability that the upper side of the die
shows:
a) six aces; b) an odd number of aces;
c) no less than four aces; d) no more than two aces.
1.12. An urn contains 12 balls: 3 white, 4 black and 5 red. Find the
probability that a randomly taken ball will be black.
1.13. The first box contains 5 balls with numbers from 1 up to 5, and the
second – 5 balls with numbers from 6 up to 10. It has been randomly
extracted on one ball from each box. Find the probability that the sum
of numbers of the extracted balls will be: 1) no less than 7; 2) equal to
11; 3) no more than 11.
The answer: 2) 0,2; 3) 0,6.
1.14. The relative frequency of workers of an enterprise having a higher
education is equal to 0,18. Determine the number of workers having a
higher education if the total number of workers of the enterprise is 350.
1.15. 78 seeds have germinated of 100 planted seeds. Find the relative
frequency of germination of seeds (seed – семя; to germinate –
прорастать, to plant – сажать).
1.16. Two dice are tossed. Find the probability that:
a) the sum of aces equals 5, and the product equals 6;
b) the product of aces doesn’t exceed 6;
c) the product of aces is divided on 6.

13
LECTURE 1

1.17. A point C is randomly appeared in a segment AB of the length 3.


Determine the probability that the distance between C and B exceeds 1.
The answer: 2/3.
1.18. A point is randomly thrown inside of a circle of the radius R. Find
the probability that the point will be inside the square entered in the
circle. It is supposed that the probability of hit of a point in the square
is proportional to its area and does not depend on its location regarding
the circle.
The answer: 2/π.
1.19. A coin is tossed twice. Find the probability that the coin lands on
heads in both times.
The answer: 1/4.
1.20. A point B(x) is randomly put in a segment OA of the length 6 of
the numeric axis Ox. Find the probability that both the segments OB
and BA have the length which is greater than 2.
The answer: 1/3.

14
Basic Formulas of Combinatorial Analysis

LECTURE 2

Basic Formulas of Combinatorial Analysis


Here is a typical problem of interest involving probability. A
communication system is to consist of n seemingly identical antennas
that are to be lined up in a linear order. The resulting system will then
be able to receive all incoming signals – and will be called functional
– as long as no two consecutive antennas are defective. If it turns out
that exactly m of the n antennas are defective, what is the probability
that the resulting system will be functional? For instance, in the special
case where n = 4 and m = 2 there are 6 possible system configurations
– namely,
0110, 0101, 1010, 0011, 1001, 1100
where 1 means that the antenna is working and 0 that it is defective.
As the resulting system will be functional in the first 3 arrangements
and not functional in the remaining 3, it seems reasonable to take
3/6 = 1/2 as the desired probability. In the case of general n and m,
we could compute the probability that the system is functional in a
similar fashion. That is, we could count the number of configurations
that result in the system being functional and then divide by the total
number of all possible configurations.
From the preceding we see that it would be useful to have an effective
method for counting the number of ways that things can occur. In fact,
many problems in probability theory can be solved simply by counting
the number of different ways that a determinate event can occur. The
mathematical theory of counting is formally known as combinatorial
analysis.
Combinatorial analysis studies quantities of ordered sets subordinate to
determinate conditions, which can be made of elements, indifferent of
a nature, of a given finite set.
Permutations are ordered sets consisting of the same n different elements
and distinguishing only by the order of their disposition (location).

15
LECTURE 2

The number of all possible permutations Pn = n!


where n! = 1∙2∙3∙...∙n
Observe that it is convenient to consider 0!, assuming by definition that

0! = 1

Example. How many three-place numbers can be made of the digits 1,


2, 3 if each digit is included into the image of a number only once?
Solution: P3 = 3! = 1 ∙ 2 ∙ 3 = 6.
Allocations are ordered sets composed of n different elements on m
elements, which are differed by either structure of elements or their
order.
The number of all possible allocations

A nm = n (n - 1) (n - 2) ... (n - m + 1) =
n! , i.e.
(n - m) !
n!
A nm =
(n - m) !
Example. How many signals is it possible to make of 6 flags of different
colour taken on 2?

Solution: A62 = 6 $ (6 - 2 + 1) = 6! = 30
(6 - 2) !
Combinations are ordered sets composed of n different elements on m
elements that are differed by at least one element.
n!
The number of all possible combinations C nm =
m! (n - m) !

Example. How many ways are there to choose two items from a box
containing 10 items?

16
Basic Formulas of Combinatorial Analysis

Solution: The required number of ways C102 = 10! = 45 .


2! (10 - 2) !
Observe that the numbers of allocations, permutations and combinations
are connected by the equality A nm = Pm $ C nm
Remark. It was above supposed that all n elements are different. If some
elements are repeated then in this case ordered sets with repetitions are
calculated by other formulas. For example, if there are n1 elements of
one kind, n2 elements of other kind and et cetera among n elements then
the number of permutations with repetitions
n!
P (n , n , ...) =
n 1 2
n !n !...
1 2

where n1 + n2 + … = n.
Example. How many seven-place numbers consisting of the digits 4, 5
and 6 are there in which the digit 4 is repeated 3 times, and the digits 5
and 6 – 2 times?
Solution: Each seven-place number differs from other by the order of
consecution of the digits (so that n1 = 3, n2 = 2, n3 = 2, and their sum is
equal to 7), i.e. is the permutation with repetitions of 7 elements:
7! =
P7 (3; 2; 2) = 210
3!2!2!
If in allocations (combinations) of n elements on m some of elements
(or all) can appear identical, such allocations (combinations) are said to
be allocations (combinations) with repetitions of n elements on m. For
example, allocations with repetitions of 5 elements a, b, c, d, e on 3 are
abc, cba, bcd, cdb, bbe, ebb, beb, ddd and et cetera; combinations with
repetitions – abc, bcd, bbe, ddd and et cetera.
The number of allocations with repetitions of n elements on m is

Au nm = n m

17
LECTURE 2

The number of combinations with repetitions of n elements on m is


~m
C n = C nm+ m - 1

Example. 10 films participate in a competition on 5 nominations. How


many variants of distribution of prizes are there, if on each nomination
are established:
a) different prizes; b) identical prizes?
Solution: a) Each of variants of distribution of prizes represents a
combination of 5 films from 10 that differs from other combinations
by both the structure of elements and their order on nominations so that
the same films can be repeated some times (a film can receive prizes
on both one nomination and some nominations), i.e. it represents the
allocations with repetitions of 10 elements on 5:
~
A510 = 105 = 100000.

b) If identical prizes are established on each nomination then the order


of following the films in a combination of 5 prizewinners doesn’t have
any significance. Therefore, the number of variants of distribution of
prizes represents the number of combinations with repetitions of 10
elements on 5:
14 $ 13 $ 12 $ 11 $ 10 =
Cu105 = C105 + 5 - 1 = C145 = 2002.
1$2$3$4$5
The following rules are used for solving problems of combinatorial
analysis:
Sum rule. If some object A can be chosen from the set of objects by
m ways, and another object B can be chosen by n ways, then we can
choose either A or B by m + n ways.
Example. There are 300 items in a box. It is known that 150 of them are
items of the first kind, 120 – the second kind, and the rest – the third
kind. How many ways of extracting an item of the first or the second

18
Operations Over Events

kind from the box are there?


Solution: An item of the first kind can be extracted by n1 = 150 ways, of
the second kind – by n2 = 120 ways. By the sum rule there are n1 + n2
= 150 + 120 = 270 ways of extracting an item of the first or the second
kind.
Product rule. If an object A can be chosen from the set of objects by m
ways and after every such choice an object B can be chosen by n ways
then the pair of the objects (A, B) in this order can be chosen by mn
ways.
Example. There are 30 students in a group. It is necessary to choose a
leader, its deputy and head of professional committee. How many ways
of choosing them are there?
Solution: A leader can be chosen from any of 30 students, its deputy –
from any of the rest 29 students, and head of professional committee
– from any of the rest 28 students, i.e. n1 = 30, n2 = 29, n3 = 28. By
the product rule the total number of ways is n1 ∙ n2 ∙ n3 = 30 ∙ 29 ∙ 28 =
=24360.

Operations Over Events


We introduce the following operations over events: sum, product and
negation.
The sum of two events A and B is such third event A + B which consists
in appearance of at least one of these events, i.e. A or B. If A and B are
compatible events then their sum A + B means appearance of either the
event A, or the event B, or both A and B. If A and B are incompatible
events then their sum A + B means appearance of either the event A, or
the event B. For example, if two shots are made by a gun and A is hit at
the first shot, B is hit at the second shot then A + B is either hit at the first
shot or hit at the second shot, or two hits. The event A + B + C consists
of appearance of one of the following events: A; B; C; both A and B;
both A and C; both B and C; all the events A, B and C.
The product of two events A and B is such third event AB which consists
in simultaneous appearance of the events A and B. If A and B are

19
LECTURE 2

incompatible events then their product AB is an impossible event.


The negation of an event A is the event A (not A) which consists in
non-appearance of the event A. Observe that A + A is a reliable event,
and A ∙ A is an impossible event.
Example. A winner of a competition is rewarded: by a prize (the event
А), a money premium (the event В), a medal (the event С).
What do the following events represent: a) A + B; b) ABC; c) ACB ?
Solution: a) The event A + B consists in rewarding the winner by a
prize, or a money premium, or simultaneously both a prize and a money
premium.
b) The event ABC consists in rewarding the winner by a prize, a money
premium and a medal simultaneously.
c) The event ACB consists in rewarding the winner by both a prize and
a medal simultaneously without giving a money premium.

Glossary
seemingly – на вид, по-видимому; arrangement – расположение
allocation – размещение; combination – сочетание
permutation – перестановка; three-place number – трехзначное
число
consecution – следование; repetition – повторение
to extract – извлекать; simultaneous – одновременный
negation – отрицание

Exercises for Seminar 2


2.1. A college planning committee consists of 3 freshmen, 4 sophomores,
5 juniors, and 2 seniors. A subcommittee of 4, consisting of 1 person from
each class, is to be chosen (a freshman – первокурсник; a sophomore –
второкурсник). How many different subcommittees are possible?

20
Exercises for Seminar 2

2.2. How many outcome sequences are possible when a die is rolled
four times, where we say, for instance, that the outcome is 3, 4, 3, 1 if
the first roll landed on 3, the second on 4, the third on 3, and the fourth
on 1?
2.3. There are five disks on the general axis of a lock. Each disk is
subdivided into six sectors on which different letters are written. The
lock opens only in the event that each disk occupies one certain position
regarding the case of the lock. Find the probability that the lock can be
opened at any installation of disks (the case of a lock – корпус замка).
The answer: 0,0001286.
2.4. The order of performance of 7 participants of a competition is
determined by a toss-up. How many different variants of the toss-up
are possible?
2.5. There are 10 cards each of which contains one letter: 3 cards with
letter A, 2 cards with letter S and 5 cards with letters D, R, O, M, B.
A child takes cards in a random order and puts one to another. Find
the probability that the word «AMBASSADOR» will be turned out
(to turn out – оказываться).
2.6. By the conditions of the lottery «Sportloto 6 of 45» a participant
of the lottery who have guessed 4, 5 or 6 numbers from 6 randomly
selected numbers of 45 receives a monetary prize. Find the probability
that the participant will guess: a) all 6 numbers; b) 4 numbers.
2.7. 10 of 30 students have sport categories. What is the probability that
3 randomly chosen students have sport categories?
The answer: 0,03.
2.8. A group consists of 12 students, and 8 of them are pupils with
honor. 9 students are randomly selected. Find the probability that 5
pupils with honor will be among the selected.
The answer: 0,255.

21
LECTURE 2

2.9. Eight different books are randomly placed on one shelf. Find the
probability that two certain books will be put beside (a shelf – полка,
beside – рядом).
The answer: 0,25.
2.10. A box contains 5 red, 3 green and 2 blue pencils. 3 pencils are
randomly extracted from the box. Find the probabilities of the following
events:
A – all the extracted pencils are different color;
B – all the extracted pencils are the same color;
C – one blue pencil among the extracted;
D – exactly two pencils of the same color among the extracted.
The answer: P(A) = 0,25; P(B) = 0,092; P(C) = 0,467; P(D) = 0,658.
2.11. It has been sold 21 of 25 refrigerators of three marks available in
quantities of 5, 7 and 13 units in a shop. Assuming that the probability
to be sold for a refrigerator of each mark is the same, find the probability
of the following events:
a) refrigerators of one mark have been unsold;
b) refrigerators of three different marks have been unsold.
The answer: a) 0,06; b) 0,396.
2.12. A shooter has made three shots in a target. Let Ai be the event «hit
by the shooter at the i-th shot» (i = 1, 2, 3). Express by A1, A2, A3 and
their negations the following events: A – «only one hit»; B – «three
misses»; C – «three hits»; D – «at least one miss»; E – «no less than two
hits»; F – «no more than one hit».

Exercises for Homework 2


2.13. How many different 7-place codes for license plates are possible
if the first 3 places are to be occupied by letters of Latin alphabet and
the final 4 by numbers?
The answer: 175760000.

22
Exercises for Homework 2

2.14. In Ex. 2.13, how many codes for license plates would be possible
if repetition among letters or numbers were prohibited?
2.15. 10 persons participate in competitions, and three of them will
take the first, second and third places. How many different variants are
possible?
The answer: 720.
2.16. How many ways of choosing 3 persons of 10 are possible?
The answer: 120.
2.17. A randomly taken phone number consists of 5 digits. What is the
probability that all digits of the phone number are:
a) identical;
b) odd?
It is known that any phone number does not begin with the digit zero.
The answer: a) 0,0001; b) 0,0347.
2.18. There are 3 cards with letter S, 3 cards with letter T, 2 cards with
letter I, 1 card with letter A and 1 card with letter C. Cards are mixed and
randomly taken out without replacement by one. Find the probability
that cards with letters are taken out by the way of consecution of letters
of the word «STATISTICS».
The answer: 0,0000198.
2.19. A box contains 15 items, and 10 of them are painted. A collector
chooses at random 3 items. Find the probability that the chosen items
are painted (collector – сборщик).
The answer: 0,264.
2.20. Find the probability that from 10 books located in a random order,
3 certain books will be beside.
The answer: 0,0667.

23
LECTURE 2

2.21. Four tickets are distributed among 25 students (15 of them are
girls). Everyone can take only one ticket. What is the probability that
owners of these tickets will be:
a) four girls;
b) four young men;
c) three young men and one girl?
The answer: a) 0,108; b) 0,017; c) 0,142.
2.22. There are 100 products (including 4 defective) in a batch. The
batch is arbitrarily divided into two equal parts which are sent to two
consumers. What is the probability that all defective products will be
got:
a) by one consumer;
b) by both consumers fifty-fifty?
The answer: a) 0,117; b) 0,383.
2.23. A library consists of ten different books, and five books cost on
4 thousands of tenghe each, three books – on one thousand of tenghe
and two books – on 3 thousands of tenghe. Find the probability that two
randomly taken books cost 5 thousands of tenghe.
The answer: 1/3.
2.24. A coin is tossed three times. Let Ai be the event «an appearance
of heads at the i-th tossing» (i = 1, 2, 3). Express by A1, A2, A3 and their
negations the following events: A – «three heads»; B – «three tails»; C
– «at least one heads»; D – «at least one tails»; E – «only one heads»;
F – «only one tails».

24
Theorem of Addition of Probabilities of Incompatible Events

LECTURE 3

Theorem of Addition of Probabilities


of Incompatible Events
Let events A and B be incompatible and let the probabilities of these
events be known. How can we find the probability of A + B?
Theorem. The probability of appearance of any of two incompatible
events is equal to the sum of the probabilities of these events:

P(A + B) = P(A) + P(B)

Corollary. The probability of appearance of any of several pairwise


incompatible events is equal to the sum of the probabilities of these
events:

P(A1 + A2 + …+ An ) = P(A1 ) + P(A2 )+ … + P(An ).

Example. There are 30 balls in an urn: 10 red, 5 blue and 15 white. Find
the probability of appearance of a colour ball.
Solution: An appearance of a colour ball is an appearance of either red
or blue ball. The probability of appearance of a red ball (the event A)
is equal to P(A) = 10/30 = 1/3. The probability of appearance of a blue
ball (the event B) is equal to: P(B) = 5/30 = 1/6. The events A and B
are incompatible (an appearance of a ball of one colour excludes an
appearance of a ball of other colour), therefore the theorem of addition
is applicable. The required probability is:

P(A + B) = P(A) + P(B) = 1/3 + 1/6 = 1/2.

Example. A shooter shoots in a target subdivided into three areas. The


probability of hit in the first area is 0,45 and in the second – 0,35. Find
the probability that the shooter will hit at one shot either in the first area
or in the second area.

25
LECTURE 3

Solution: The events A – «the shooter hit in the first area» and B – «the
shooter hit in the second area» are incompatible (hit in one area excludes
hit in other area). Therefore, the theorem of addition is applicable. The
required probability is:

P(A + B) = P(A) + P(B) = 0,45 + 0,35 = 0,80.

Complete Group of Events


Theorem. The sum of the probabilities of events A1, A2, A3, …, An which
form a complete group is equal to 1:

P(A1 ) + P(A2 ) + P(A3 ) + … + P(An ) = 1

Example. A consulting point of an institute receives packages with


control works from the cities A, B and С. The probability of receiving
a package from the city A is equal 0,7; from the city B – 0,2. Find the
probability that next package will be received from the city С.
Solution: The events «a package has been received from A», «a package
has been received from B» and «a package has been received from C»
form a complete group. Therefore, the sum of probabilities of these
events is equal to 1:

0,7 + 0,2 + p = 1

Then the required probability is equal to p = 1 – 0,9 = 0,1.

Opposite Events
Two uniquely possible events forming a complete group are opposite.
If A denotes one of two opposite events, then the opposite to A event is
denoted by A .
Example. Hit and miss at a shot in a target are opposite events.
Example. An item is randomly taken from a box. The events «a standard

26
Conditional Probability

item has appeared» and «a non-standard item has appeared» are


opposite.
Theorem. The sum of the probabilities of opposite events is equal to 1:

P(A) + P( A ) = 1

Example. The probability that a day will be rainy is p = 0,7. Find the
probability that a day will be clear.
Solution: The events «a day is rainy» and «a day is clear» are opposite,
therefore the required probability is q = 1 – p = 1 – 0,7 = 0,3.
Example. There are n items in a box, and m of them are standard. Find
the probability that there is at least one standard item among k randomly
extracted items.
Solution: The events «there is at least one standard item among the
extracted items» and «there is no standard item among the extracted
items» are opposite. Denote the first event by A, and the second – by
A . Obviously, P(A) = 1 – P( A ). Find P( A ). The total number of ways
by which one can extract k items from n items is Cnk. The number of
non-standard items is n – m. One can extract k non-standard items from
n – m non-standard items by C n-m
k
ways. Therefore, the probability that
there is no standard item among k extracted items is P( A ) = C n-m
k
/ Cnk.
The required probability is:

P(A) = 1 – P( A ) = 1 – C n-m
k
/ Cnk.

Conditional Probability
A random event has been before determined as an event that can take
place or not to take place for holding the set of conditions S. If there are
no any other restrictions except for the conditions S for a calculation
of the probability of an event then such a probability is unconditional;
if there are other auxiliary conditions then the probability of an event

27
LECTURE 3

is said to be conditional. For example, the probability of an event B


is very often calculated with an auxiliary condition that an event A
was happened. Observe that unconditional probability also, strictly
speaking, is conditional since it is supposed that the conditions S hold.
The conditional probability PA(B) is the probability of the event B
calculated in assumption that the event A has already happened.
Example. There are 3 white and 3 black balls in an urn. One takes out
twice on one ball from the urn without replacement. Find the probability
of appearance of a white ball at the second trial (the event B) if a black
ball was extracted at the first trial (the event A).
Solution: There are 5 balls after the first trial, and 3 of them are white.
The required probability is PA(B) = 3/5.
The conditional probability of an event B with the condition that an
event A has already happened is equal to:

P (AB)
PA (B) =
P (A)
where P(A) > 0.
Indeed, return to our example. The probability of appearance of a white
ball at the first trial P(A) = 3/6 = 1/2. Find the probability P(AB) that
a black ball will be appeared at the first trial and a white ball – at the
second trial. The total number of events – a joint appearance of two
balls (indifferently from colour) is equal to the number of allocations
A62 = 6 ∙ 5 = 30. The event AB is favored 3 ∙ 3 = 9 events from the total
number. Consequently, P(AB) = 9/30 = 3/10. Thus,

PA(B) = P(AB)/P(A) = (3/10)/(1/2) = 3/5.

Theorem of Multiplication of Probabilities


Theorem. The probability of joint appearance of two events is equal
to the product of the probability of one of them on the conditional
probability of another event calculated in assumption that the first event
has already happened:

28
Theorem of Multiplication of Probabilities

P(AB) = P(A) · PA(B)

Remark. P(BA) = P(B) · PB(A).


Since the event BA does not differ from the event AB,
P(AB) = P(B) · PB(A).
Consequently, P(A) · PA(B) = P(B) · PB(A).
Corollary. The probability of joint appearance of several events is
equal to the product of the probability of one of them on the conditional
probabilities of all the rest events so that the probability of each
subsequent event is calculated in assumption that all preceding events
have already happened:

P(A1 A2 A3... An ) = P(A1 )·PA (A2 )·PA A (A3 )·...·PA A ...A (An )
1 1 2 1 2 n-1

where PA A ...A (An ) is the probability of the event An calculated in


1 2 n-1
assumption that the events A1, A2, …, An–1 have happened.
In particular, for three events P(ABC) = P(A)·PA(B)·PAB(C)
Observe that the order of location of the events can be chosen any, i.e.
it is indifferently which event is assumed the first, the second and et
cetera.
Example. There are 3 conic and 7 elliptic cylinders at a collector. The
collector has taken one cylinder, and then he has taken the second
cylinder. Find the probability that the first taken cylinder is conic, and
the second – elliptic.
Solution: The probability that the first cylinder will be conic (the event A)
P(A) = 3/10. The probability that the second cylinder will be elliptic
(the event B) calculated in assumption that the first cylinder is conic, i.e.
the conditional probability PA(B) = 7/9. By theorem of multiplication,
the required probability is

P(AB) = P(A) · PA(B) = (3/10) · (7/9) = 7/30.

29
LECTURE 3

Example. There are 5 white, 4 black and 3 blue balls in an urn. Each
trial consists in extracting at random one ball without replacement. Find
the probability that a white ball will appear at the first trial (the event A),
a black ball will appear at the second trial (the event B), and a blue ball
will appear at the third trial (the event C).
Solution: The probability of appearance of a white ball at the first trial
P(A) = 5/12. The probability of appearance of a black ball at the second
trial calculated in assumption that a white ball has appeared at the first
trial, i.e. the conditional probability PA(B) = 4/11. The probability of
appearance of a blue ball at the third trial calculated in assumption that
a white ball has appeared at the first trial, and a black ball has appeared
at the second trial, i.e. the conditional probability PAB(C) = 3/10. The
required probability is
5 4 3 = 1
P (ABC) = P (A) $ PA (B) $ PAB (C) = $ $ .
12 11 10 22

Glossary
opposite events – противоположные события
to extract – извлекать; without replacement – без возвращения
conditional probability – условная вероятность
preceding – предшествующий
conic – конический; elliptic – эллиптический
cylinder – валик; collector – сборщик

Exercises for Seminar 3


3.1. In a cash–prize lottery 150 prizes and 50 monetary winnings are
played on every 10000 tickets. What is the probability of a winning
indifferently monetary or prize for an owner of one lottery ticket
equal to?

30
Exercises for Seminar 3

3.2. The events A, B, C and D form a complete group. The probabilities


of the events are those: P(A) = 0,1; P(B) = 0,4; P(C) = 0,3. What is the
probability of the event D equal to?
3.3. The probability that a shooter will beat out 10 aces at one shot is
equal to 0,1 and the probability to beat out 9 aces is equal to 0,3. Find
the probabilities of the following events: A – the shooter will beat out
8 or less aces; B – the shooter will beat out no less than 9 aces.
3.4. There are 10 items in a box, and 2 of them are non-standard. Find
the probability that in randomly selected 6 items appears no more than
one non-standard item.
Direction: If A is «there is no non-standard items» and B is «there
is one non-standard item» then P(A + B) = P(A) + P(B) =…
3.5. An enterprise produces 95% standard products, and 86% of them
have the first grade. Find the probability that a randomly taken product
made at the enterprise will be standard and the first grade (grade – сорт).
3.6. Two dice are tossing. Find the conditional probability that each die
lands on 5 if it is known that the sum of aces is divided on 5.
3.7. If two dice are rolled, what is the conditional probability that the
first one lands on 4 given that the sum of the dice is 8?
3.8. In a certain community, 36 percent of the families own a dog, and
22 percent of the families that own a dog also own a cat. In addition,
30 percent of the families own a cat. What is
(a) the probability that a randomly selected family owns both
a dog and a cat;
(b) the conditional probability that a randomly selected family
owns a dog given that it owns a cat?
3.9. An ordinary deck of 52 playing cards is randomly divided into
4 piles of 13 cards each. Compute the probability that each pile has
exactly 1 ace (a pile – стопка; a deck – колода; an ace – туз).

31
LECTURE 3

3.10. A coin is tossed until it will not land on the same side 2 times
in succession. Find the probability that the experiment will terminate
before the sixth tossing (in succession – подряд).

Exercises for Homework 3


3.11. By the statistical data of a repair shop 20 stops of a lathe are on
the average: 10 – for change of a cutter; 3 – because of malfunction
of a drive; 2 – because of delayed submission of items. The rest stops
occur for other reasons. Find the probability of stop of the lathe for
other reasons (repair shop – ремонтная мастерская; lathe – токарный
станок; cutter – резец; malfunction – неисправность; drive – привод).
The answer: 0,25.
3.12. There are 30 TVs in a shop, and 20 of them are import. Find
the probability that no less than 3 import TVs will be among 5 TVs
sold for one day, assuming that the probabilities of purchase of TVs
of different marks are identical.
The answer: 0,81.
3.13. If two dice are rolled, what is the conditional probability that
the first one lands on 6 given that the sum of the dice is 11?
The answer: 0,5.
3.14. An urn contains 6 white and 9 black balls. If 4 balls are to be
randomly selected without replacement, what is the probability that
the first 2 selected are white and the last 2 black?
The answer: 0,066.
3.15. Fifty-two percent of the students at a certain university are
females. Five percent of the students in this university are majoring in
computer science. Two percent of the students are women majoring in
computer science. If a student is selected at random, find the conditional
probability that
(a) this student is female, given that the student is majoring
in computer science;

32
Exercises for Homework 3

(b) this student is majoring in computer science, given that


the student is female.
The answer: a) 0,4; b) 0,038.
3.16. Celine is undecided as to whether to take a French course or
a chemistry course. She estimates that her probability of receiving an A
grade would be 1/2 in a French course and 2/3 in a chemistry course.
If Celine decides to base her decision on the flip of a coin, what is
the probability that she gets an A in chemistry (grade – оценка)?
The answer: 0,33.
3.17. Suppose that an urn contains 8 red and 4 white balls. We draw
2 balls from the urn without replacement. If we assume that at each draw
each ball in the urn is equally likely to be chosen, what is the probability
that both balls drawn are red (to draw – тянуть)?
The answer: 0,424.
3.18. An urn contains 10 white, 15 black, 20 blue and 25 red balls. A ball
is taken at random from the urn. Find the probability that the taken
ball is:
a) white or black; b) blue or red.
3.19. Two shooters shoot in a target. Find the probability that the target
will be struck at least one of the shooters if it is known that the probability
of miss by both shooters is equal to 0,27.
3.20. Two cards are randomly selected from a pack of 36 playing
cards. Find the probability that both cards are the same color
(a pack – колода).
The answer: 0,486.

33
LECTURE 4

LECTURE 4

Independent Events
An event B is said to be independent from an event A if appearance
of the event A does not change the probability of the event B, i.e. if
the conditional probability of the event B is equal to its unconditional
probability:

PA (B) = P(B) (*)

Since P(A) · PA(B) = P(B) · PB(A), by using (*) we have


P(A) · P(B) = P(B) · PB(A) and consequently PB(A) = P(A), i.e. the
event A does not depend from the event B.
Thus, if an event B does not depend from an event A then the event
A does not depend from the event B; this means that the property
of independence is mutual.
For independent events theorem of multiplication P(AB) = P(A) ·PA(B)
has the following form:

P(AB) = P(A) · P(B) (**)

i.e. the probability of joint appearance of two independent events is


equal to the product of the probabilities of these events. The equality
(**) is accepted as a definition of independent events.
Two events are independent if the probability of their joint appearance
is equal to the product of the probabilities of these events; otherwise
they are dependent.
At practice one concludes on independence of events on the sense
of a problem. For example, the probability of hit in a target by each
of two guns does not depend on that whether another gun has hit in
the target, therefore the events «the first gun has hit in the target» and
«the second gun has hit in the target» are independent.

34
Independent Events

Example. Find the probability of joint hit in a target by two guns if


the probability of hit in the target by the first gun (the event A) is equal
to 0,8; and by the second gun (the event B) – 0,7.
Solution: The events A and B are independent, therefore by theorem
of multiplication, the required probability
P(AB) = P(A) · P(B) = 0,8 · 0,7 = 0,56.
Remark. If events A and B are independent, the events A and B , A
and B, A and B are also independent.
Proof of Remark:
A = A B + AB ⇒ P(A) = P(A B ) + P(AB) = P(A B ) + P(A) · P(B)
Then we have P(A B ) = P(A) · [1 – P(B)] = P(A) · P( B ), i.e. the events
A and B are independent.
Several events are pairwise independent if each two of them are
independent. For example, events A, B and C are pairwise independent
if the events A and B, A and C, B and C are independent.
Several events are independent in union (or just independent) if each
two of them are independent and each event and all possible products
of the rest events are independent. For example, if events A1, A2 and A3
are independent in union then the events A1 and A2, A1 and A3, A2 and A3,
A1 and A2A3, A2 and A1A3, A3 and A1A2 are independent.
Let’s underline that a pairwise independence of several events does
not imply their independence in union in general, i.e. the demand
of independence of events in union is stronger than the demand of their
pairwise independence.
Example. There are 4 coloured balls in an urn: one ball is coloured
in red colour (A), one ball is coloured in blue colour (B), one ball is
coloured in black colour (C) and one ball – in all these three colours
(ABC). Find the probability that a ball extracted at random from the
urn has red colour. Since two of four balls have red colour, P(A) = 2/4
= 1/2. Reasoning analogously, we have P(B) = 1/2, P(C) = 1/2. Assume
now that the taken ball has blue colour, i.e. the event B has already

35
LECTURE 4

happened. Is the probability that the extracted ball has red colour
changed, i.e. is the probability of the event A changed? One ball of two
balls having blue colour has also red colour, therefore the probability
of the event A is still equal to 1/2. In other words, the conditional
probability of the event A calculated in assumption that the event B
has happened is equal to its unconditional probability. Consequently,
the events A and B are independent. By analogy we have that the
events A and C, B and C are independent. Thus, the events A, B and
C are pairwise independent. Are these events independent in union?
Let the extracted ball have two colours, for example, blue and black.
What is the probability that this ball has also red colour? Since only
one ball is coloured in all three colours, therefore the taken ball has also
red colour. Thus, assuming that the events B and C have happened, we
have the event A will necessarily happen. Consequently, this event is
reliable and its probability is equal to 1. In other words, the conditional
probability PBC(A) = 1 of the event A is not equal to its unconditional
probability P(A) = 1/2. Thus, the pairwise independent events A, B and
C are not independent in union.
Corollary. The probability of joint appearance of several events that
are independent in union is equal to the product of the probabilities
of these events:

P(A1 A2…An ) = P(A1 ) · P(A2 ) · …· P(An ).

Remark. If events A1, A2, …, An are independent in union, then


the opposite to them events A 1, A 2, ... A n are also independent in union.
Example. Find the probability of a joint appearance of heads at tossing
two coins.
Solution: The probability of appearance of heads on the first coin
(the event A) P(A) = 1/2. The probability of appearance of heads
on the second coin (the event B) P(B) = 1/2. The events A and B
are independent; therefore the required probability by theorem
of multiplication is equal to:

36
Probability of Appearance of At Least One Event

P(AB) = P(A) · P(B) = 1/2 · 1/2 = 1/4.


Example. There are 3 boxes containing 10 items each. There
are 8 standard items in the first box, 7 – in the second and 9 – in
the third box. One takes at random on one item from each box. Find
the probability that all three taken items will be standard.
Solution: The probability that a standard item has been taken from
the first box (the event A) P(A) = 8/10 = 0,8. The probability that
a standard item has been taken from the second box (the event B)
P(B) = 7/10 = 0,7. The probability that a standard item has been taken
from the third box (the event C) P(C) = 9/10 = 0,9. Since the events A,
B and C are independent in union, the required probability (by theorem
of multiplication) is equal to:
P(ABC) = P(A) · P(B) · P(C) = 0,8 · 0,7 · 0,9 = 0,504.

Probability of Appearance of At Least One Event


Let as a result of a trial n events independent in union or some
of them (in particular, only one or none) can appear, so that the
probabilities of appearance of each of the events are known. How can
we find the probability that at least one of these events will happen?
For example, if as a result of a trial three events can appear, then an
appearance of at least one of these events means an appearance of either
one, or two, or three events. The answer on the posed question is given
by the following theorem.
Theorem. The probability of appearance of at least one of the events
A1, A2, …, An independent in union is equal to the difference
between 1 and the product of the probabilities of the opposite events
A 1, A 2, ... A n :

P(A) = 1 – q1q2…qn

where A is the appearance of at least one of the events A1, A2, …, An;
P ( A i) = qi, i = 1, n .

37
LECTURE 4

Partial case. If the events A1, A2, …, An have the same probability which
is equal to p then the probability of appearance of at least one of these
events:

P(A) = 1 – qn

where A is the appearance of at least one of the events A1, A2, …, An;
P ( A i) = q = 1 - p, i = 1, n .
Example. The probabilities of hit in a target at shooting by three guns are
the following: p1 = 0,8; p2 = 0,7; p3 = 0,9. Find the probability of at least
one hit (the event A) at one shot by all three guns.
Solution: The probability of hit in the target by each of the guns doesn’t
depend on results of shooting by other guns, therefore the considered
events A1 (hit by the first gun), A2 (hit by the second gun) and A3 (hit
by the third gun) are independent in union. The probabilities of events
which are opposite to the events A1, A2 and A3 (i.e. the probabilities
of misses) are equal respectively:
q1 = 1 – p1 = 1 – 0,8 = 0,2; q2 = 1 – p2 = 1 – 0,7 = 0,3;

q3 = 1 – p3 = 1 – 0,9 = 0,1.
The required probability P(A) = 1 – q1 q2 q3 = 1 – 0,2 · 0,3 · 0,1 = 0,994.
Example. There are 4 flat-printing machines at typography. For each
machine the probability that it works at the present time is equal to 0,9.
Find the probability that at least one machine works at the present time
(the event A).
Solution: The events «a machine works» and «a machine doesn’t work»
(at the present time) are opposite, therefore the sum of their probabilities
is equal to 1: p + q = 1. Consequently, the probability that a machine
doesn’t work at the present time is equal to q = 1 – p = 1 – 0,9 = 0,1.
The required probability P(A) = 1 – q4 = 1 – (0,1)4 = 0, 9999.

38
Probability of Appearance of At Least One Event

Example. A student looks for one formula necessary to him in three


directories. The probability that the formula is contained in the first,
second and third directories, is equal to 0,6; 0,7 and 0,8 respectively.
Find the probability that the formula is contained:
a) only in one directory (the event A);
b) only in two directories (the event B);
c) in all the directories (the event C);
d) at least in one directory (the event D);
e) neither of the directories (the event E).
Solution: Consider elementary events and their probabilities:
А1 – the formula is in the first directory, P(A1) = 0,6;
P( A 1 ) = 1 – 0,6 = 0,4;
А2 – the formula is in the second directory, P(A2) = 0,7;
P( A 2 ) = 1 – 0,7 = 0,3;
А3 – the formula is in the third directory, P(A1) = 0,8;
P( A 3 ) = 1 – 0,8 = 0,2
Express all the events A–E by the elementary events and their negations,
and apply the above-stated theorems:
a) A = A1 $ A 2 $ A 3 + A 1 $ A2 $ A 3 + A 1 $ A 2 $ A3
P (A) = P (A1 $ A 2 $ A 3) + P ( A 1 $ A2 $ A 3) + P ( A 1 $ A 2 $ A3) =
= P (A1) $ P ( A 2) $ P ( A 3) + P ( A 1) $ P (A2) $ P ( A 3) +
+ P ( A 1) $ P ( A 2) $ P (A3) =
= 0, 6 $ 0, 3 $ 0, 2 + 0, 4 $ 0, 7 $ 0, 2 + 0, 4 $ 0, 3 $ 0, 8 = 0, 188.
b) B = A1 $ A2 $ A 3 + A1 $ A 2 $ A3 + A 1 $ A2 $ A3,
P (B) = P (A1 $ A2 $ A 3) + P (A1 $ A 2 $ A3) + P ( A 1 $ A2 $ A3) =
= P (A1) $ P (A2) $ P ( A 3) + P (A1) $ P ( A 2) $ P (A3) +
+ P ( A 1) $ P (A2) $ P (A3) =
= 0, 6 $ 0, 7 $ 0, 2 + 0, 6 $ 0, 3 $ 0, 8 + 0, 4 $ 0, 7 $ 0, 8 = 0, 452.

39
LECTURE 4

c) C = A1 $ A2 $ A3,
P (C) = P (A1 $ A2 $ A3) = P (A1) $ P (A2) $ P (A3) =
= 0, 6 $ 0, 7 $ 0, 8 = 0, 336.
d) P (D) = 1 - P ( A 1) $ P ( A 2) $ P ( A 3) = 1 - 0, 4 $ 0, 3 $ 0.2 = 0, 976.
e) E = A 1 $ A 2 $ A 3, P (E) = 0, 4 $ 0, 3 $ 0, 2 = 0, 024.

Glossary
independent events – независимые события
mutual – взаимный;
independent in union – независимые в совокупности
flat-printing machine – плоскопечатная машина
directory – справочник

Exercises for Seminar 4


4.1. The probability that a shooter hit in a target at one shot is equal to
0,9. The shooter has made 3 shots. Find the probability that all 3 shots
will strike the target.
4.2. A coin and a die are tossed. Find the probability of joint appearance
of the following events: «the coin lands on heads» and «the die lands
on 6».
4.3. What is the probability that at tossing three dice 6 aces will appear
at least on one of the dice (the event А)?
The answer: 0,421.
4.4. There are 8 standard items in a batch of 10 items. Find
the probability that there is at least one standard item among two
randomly taken items.
4.5. Two dice are rolled. What is the conditional probability that at least
one lands on 6 given that the dice land on different numbers?
4.6. The probability of hit in a target by the first shooter at one shot is

40
Exercises for Homework 4

equal to 0,8, and by the second shooter – 0,6. Find the probability that
the target will be struck only with one shooter.
The answer: 0,44.
4.7. The probability to receive high dividends under shares at the first
enterprise – 0,2; on the second – 0,35; on the third – 0,15. Determine
the probability that a shareholder having shares of all the enterprises
will receive high dividends:
a) only at one enterprise;
b) at least on one enterprise (a share – акция).
The answer: a) 0,4265; b) 0,564.
4.8. The first brigade has 6 tractors, and the second – 9. One tractor
demands repair in each brigade. A tractor is chosen at random from
each brigade. What is the probability that:
a) both chosen tractors are serviceable;
b) one of the chosen tractors demands repair
(serviceable – исправный).
The answer: a) 20/27; b) 13/54.

Exercises for Homework 4


4.9. There are items in two boxes: in the first – 10 (3 of them are
standard), in the second – 15 (6 of them are standard). One takes out
at random on one item from each box. Find the probability that both
items will be standard.
The answer: 0,12.
4.10. There are 3 television cameras in a TV studio. For each camera
the probability that it is turned on at present, is equal to p = 0,6.
Find the probability that at least one camera is turned on at present
(the event А).
The answer: 0,936.

41
LECTURE 4

4.11. What is the probability that at least one of a pair of dice lands
on 6, given that the sum of the dice is 8?
The answer: 0,4.
4.12. 10 of 20 savings banks are located behind a city boundary. 5 savings
banks are randomly selected for an inspection. What is the probability
that among the selected banks appears inside the city:
a) 3 savings banks; b) at least one?
The answer: a) 0,348; b) 0,984.
4.13. There are 16 items made by the factory № 1 and 4 items of
the factory № 2 at a collector. Two items are randomly taken. Find the
probability that at least one of them has been made by the factory № 1.
The answer: 92/95.
4.14. Three buyers went in a shop. The probability that each buyer
makes purchases is equal to 0,3. Find the probability that:
a) two of them will make purchases;
b) all three will make purchases;
c) only one of them will make purchases.
The answer: a) 0,189; b) 0,027; c) 0,441.
4.15. Three students pass an exam. The probability that the exam will be
passed on “excellent” by the first student is equal to 0,7; by the second
– 0,6; and by the third – 0,2. What is the probability that the exam will
be passed on “excellent” by:
a) only one student; b) two students;
c) at least one; d) neither of the students?
The answer: a) 0,392; b) 0,428; c) 0,904; d) 0,096.
4.16. Three shots are made in a target. The probability of hit at each shot
is equal to 0,6. Find the probability that only one hit will be in result
of these shots.
The answer: 0,288.

42
Theorem of Addition of Probabilities of Compatible Events

LECTURE 5

Theorem of Addition of Probabilities


of Compatible Events
Two events are compatible if appearance of one of them doesn’t exclude
appearance of another event at the same trial.
Example. A – appearance of four aces at tossing a die; B – appearance
of an even number of aces. The events A and B are compatible.
Let events A and B be compatible, and the probabilities of these events
and the probability of their joint appearance be given. How can we
find the probability of the event A + B consisting in that at least one
of the events A and B will appear?
Theorem. The probability of appearance of at least one of two
compatible events is equal to the sum of the probabilities of these events
without the probability of their joint appearance:
P(A + B) = P(A) + P(B) – P(AB)

Remark 1. Using the obtained formula one should remember that


the events A and B can be both independent and dependent.
For independent events: P(A + B) = P(A) + P(B) – P(A) · P(B)
For dependent events: P(A + B) = P(A) + P(B) – P(A) · PA(B)
Remark 2. If the events A and B are incompatible then their joint
appearance is an impossible event and consequently, P(AB) = 0. Then
for incompatible events A and B, P(A + B) = P(A) + P(B).
Example. The probabilities of hit in a target at shooting by the first
and the second guns are equal to: p1 = 0,7; p2 = 0,8 respectively.
Find the probability of hit at one shot (by two guns) by at least one
of the guns.
Solution: The probability of hit in the target by each of guns doesn’t
depend on result of shooting by another gun, therefore the events A

43
LECTURE 5

(hit by the first fun) and B (hit by the second gun) are independent.
The probability of the event AB (both the first and the second guns gave
hit) P(AB) = P(A) · P(B) = 0,7 · 0,8 = 0,56.
The required probability is:
P(A + B) = P(A) + P(B) – P(AB) = 0,7 + 0,8 – 0,56 = 0,94.

Remark 3. Since in this example the events A and B are independent,


we can use the formula P(A+B) = 1– P( A ) · P( B ) (the probability
of appearance of at least one of the events). In fact, the probabilities of the
events which are opposite to the events A and B, i.e. the probabilities
of misses are:
P( A ) = 1 – P(A) = 1 – 0,7 = 0,3; P( B ) = 1 – P(B) = 1 – 0,8 = 0,2.
The required probability that at least one of guns gives hit at one shot
is equal to
P(A + B) = 1 – P( A ) · P( B ) = 1 – 0,3 · 0,2 = 0,94

Formula of Total Probability


Let an event A can be happened only in case of appearance of one
of incompatible events B1, B2, …, Bn which form a complete group. Let
both the probabilities of these events and the conditional probabilities
PB (A), PB (A), ..., PB (A) of the event A be known. How can we find
1 2 n
the probability of the event A? The answer on this question gives
the following:
Theorem. The probability of an event A which can be happened only
in case of appearance of one of incompatible events B1, B2, …, Bn forming
a complete group is equal to the sum of products of the probabilities
of each of these events on corresponding conditional probability
of the event A:

P(A) = P(B1 )· PB (A) + P(B2 )· PB (A)+... + P(Bn )· PB (A)


1 2 n

44
Formula of Total Probability

This formula is «the formula of total probability».


Example. There are two sets of items. The probability that an item
of the first set is standard is equal to 0,8; and of the second set – 0,9.
Find the probability that a randomly taken item (from a randomly taken
set) is standard.
Solution: Denote by A the event «an extracted item is standard». An
item can be extracted from either the first set (the event B1) or the second
set (the event B2 ). The probability that an item has been extracted
from the first set P(B1 ) = 1/2. The probability that an item has been
extracted from the second set P(B2 ) = 1/2. The conditional probability
that a standard item will be extracted from the first set PB (A) = 0,9.
2
The conditional probability that a standard item will be extracted from
the first set PB (A) The required probability that a randomly extracted
1
item is standard is equal by the formula of total probability to
P(A) = P(B1 )· PB (A) + P(B2 )· PB (A) = 0,5 · 0,8 + 0,5 · 0,9 = 0,85
1 2

Example. There are 20 radio lamps (including 18 standard ones)


in the first box, and 10 radio lamps (including 9 standard ones)
in the econd box. A lamp has been taken randomly from the second box
and placed to the first box. Find the probability that a lamp randomly
extracted from the first box is standard.
Solution: Denote by A the event «a standard item has been extracted
from the first box». One could be extracted from the second box either
a standard item (the event B1) or a non-standard item (the event B2).
The probability that a standard item has been extracted from the second
box P(B1 ) = 9/10. The probability that a non-standard item has been
extracted from the second box P(B2 ) = 1/10. The conditional probability
that a standard item will be extracted from the first box provided
that a standard lamp was placed from the second box to the first one
PB (A) = 19/21. The conditional probability that a standard item will
1
be extracted from the first box provided that a non-standard lamp was
placed from the second box to the first one PB (A) = 18/21. The required
2
probability that a standard lamp will be extracted from the first box is
equal by the formula of total probability to

45
LECTURE 5

P(A) = P(B1 )· PB (A) + P(B2 )· PB (A) = 9 $ 19 + 1 $ 18 = 0, 9.


1 2 10 21 10 21

Probability of Hypotheses. Bayes’s Formulas.


Let an event A can happen only in case of appearance of one
of incompatible events B1, B2, …, Bn forming a complete group. Since
it isn’t known beforehand which of these events will happen, we call
them by hypotheses.
The probability of appearance of the event A is defined by the formula
of total probability:
P(A) = P(B1 )· PB (A) + P(B2 )· PB (A)+... + P(Bn )· PB (A) (*)
1 2 n

Assume that a trial has been made in result of which the event A
was appeared. Pose the problem to determine how the probabilities
of the hypotheses have been changed (in connection with that
the event A has already happened). In other words, we will look for
the conditional probabilities PA (B1 ), PA (B2 ), …, PA (Bn ). Find firstly the
conditional probability PA(B1). By theorem of multiplication we have
P(AB1 ) = P(A) · PA (B1) = P(B1 ) · PB (A).
1
Consequently,
P (B1) $ PB (A)
PA (B1) =
1
.
P (A)
Replacing P(A) by (*), we obtain
P (B1) $ PB (A)
PA (B1) =
1
.
P (B1) $ PB (A) + P (B2) $ PB (A) + ... + P (Bn) $ PB (A)
1 2 n

Formulas determining the conditional probabilities of the rest


hypotheses are deduced analogously, i.e. the conditional probability
of any hypothesis Bi (i = 1, 2, …, n) can be calculated by the formula
P (Bi) $ PB (A)
PA (Bi) =
i
.
P (B1) $ PB (A) + P (B2) $ PB (A) + ... + P (Bn) $ PB (A)
1 2 n

46
Probability of Hypotheses. Bayes’s Formulas

The obtained formulas are the Bayes’s formulas (on name of British
mathematician who deduced them; published in 1764).
The Bayes’s formulas allow overestimating the probabilities
of hypotheses after that a trial has been made in result of which
the required event has appeared.
Example. Items produced by a factory shop are given for checking them
on standard to one of two controllers. The probability that an item will
be given to the first controller is equal to 0,6; and to the second – 0,4.
The probability that a suitable item will be recognized standard by the
first controller is equal to 0,94; and by the second controller – 0,98.
A randomly chosen item that is suitable has been recognized standard
at checking. Find the probability that this item was checked by the first
controller.
Solution: Denote by A the event consisting in that a suitable item has
been recognized standard. One can make two assumptions: 1) The item
has been checked by the first controller (the hypothesis B1); 2) The item
has been checked by the second controller (the hypothesis B2).
Find the required probability that the item has been checked by the first
controller by the Bayes’s formula:
P (B1) $ PB (A)
PA (B1) =
1
.
P (B1) $ PB (A) + P (B2) $ PB (A)
1 2

We have: P(B1 ) = 0,6; P(B2 ) = 0,4; PB (A) =0,94; PB (A) = 0,98


1 2
0, 6 $ 0, 94
The required probability PA (B1) = . 0, 59.
0, 6 $ 0, 94 + 0, 4 $ 0, 98
We see the probability of the hypothesis B1 equals 0,6 before the trial,
and after that a trial result has become known the probability of this
hypothesis (more precise, the conditional probability) has been
changed and become 0,59. Thus, using the Bayes’s formula allowed
overestimating the probability of the considered hypothesis.

47
LECTURE 5

Glossary
compatible events – совместные события; hypothesis – гипотеза
beforehand – заранее; to recognize – признавать, распознавать
a factory shop – цех завода; suitable item – годная деталь
to overestimate – переоценить

Exercises for Seminar 5


5.1. There are 20 skiers, 6 bicyclists and 4 runners in a group
of sportsmen. The probability to fulfil the corresponding qualifying
norm is: for a skier – 0,9, for a bicyclist – 0,8, and for a runner – 0,75.
Find the probability that a randomly chosen sportsman will fulfil the
norm (to fulfil – выполнить).
The answer: 0,86.
5.2. The first box contains 20 items and 15 of them are standard;
the second – 30 items and 24 of them are standard; the third – 10 items
and 6 of them are standard. Find the probability that a randomly
extracted item from a randomly taken box is standard.
The answer: 43/60.
5.3. There are radio lamps in two boxes. The first box contains 12 lamps,
and 1 of them is non-standard; the second box contains 10 lamps, and
1 of them is non-standard. A lamp is randomly taken from the first box
and placed in the second. Find the probability that a randomly extracted
lamp from the second box will be non-standard.
The answer: 13/132.
5.4. At a deviation of an automatic device from the normal operating
mode the signaling device C-1 acts with the probability 0,8, and
the signaling device C-11 acts with the probability 1. The probabilities
that the automatic device is supplied with C-1 or C-11 are equal to 0,6
and 0,4 respectively. A signal about cutting the automatic device has
been received. What is more probable: the automatic device is supplied
with the signaling device C-1 or C-11?

48
Exercises for Homework 5

The answer: The probability that the automatic device is supplied


with C-1, is equal to 6/11, and C-11 – 5/11.
5.5. The probability for products of a certain factory to satisfy the
standard is equal to 0,96. A simplified system of checking on standardness
gives positive result with the probability 0,98 for products satisfying
the standard, and with the probability 0,05 – for products non-satisfying
the standard. A randomly taken product has been recognized as standard
at checking. Find the probability that it really satisfies the standard.
The answer: 0,998.
5.6. A digit is firstly randomly chosen from the digits {1, 2, 3, 4, 5}, and
then the second digit – from the rest four digits. Find the probability that
an odd digit will be chosen:
a) for the first time; b) for the second time; c) in both times.
The answer: a) 3/5; b) 3/5; c) 3/10.
5.7. Three cards are randomly selected without replacement from
an ordinary deck of 52 playing cards. Compute the conditional
probability that the first card selected is a spade, given that the second
and third cards are spades.
The answer: 0,22.
5.8. Urn A contains 2 white balls and 1 black ball, whereas urn B
contains 1white ball and 5 black balls. A ball is drawn at random from
urn A and placed in urn B. A ball is then drawn from urn B. It happens
to be white. What is the probability that the ball transferred was white?
The answer: 0,8.

Exercises for Homework 5


5.9. A collector has received 3 boxes of items made by the factory
№ 1, and 2 boxes of items made by the factory № 2. The probability
that an item of the factory № 1 is standard is equal to 0,8, and the
factory № 2 – 0,9. The collector has randomly extracted an item from
a randomly taken box. Find the probability that a standard item has
been extracted (a collector – сборщик).

49
LECTURE 5

The answer: 0,84.


5.10. There are 4 kinescopes in a television studio. The probabilities
that the kinescope will sustain the warranty period of service are equal
to 0,8; 0,85; 0,9; 0,95 respectively. Find the probability that a randomly
taken kinescope will sustain the warranty period of service (to sustain
– выдержать).
The answer: 0,875.
5.11. A die has been randomly extracted from the full set of 28 dice
of domino. Find the probability that the second randomly extracted die
can be put to the first.
The answer: 7/18.
5.12. For participation in student selective sport competitions 4 students
has been directed from the first group, 6 – from the second, 5 – from the
third group. The probabilities that a student of the first, second and third
group gets in the combined team of institute, are equal to 0,9; 0,7 and
0,8 respectively. A randomly chosen student as a result of competition
has got in the combined team. Which of groups is this student most
likely belonged to (a combined team – сборная)?
The answer: The probabilities that the student has been chosen
from the first, second and third group are equal to 18/59, 21/59,
20/59 respectively.
5.13. English and American spellings are rigour and rigor, respectively.
A man staying at a Parisian hotel writes this word, and a letter taken
at random from his spelling is found to be a vowel. If 40 percent
of the English-speaking men at the hotel are English and 60 percent
are Americans, what is the probability that the writer is an Englishman
(rigour (rigor) – суровость; a vowel – гласная)?
The answer: 5/11.
5.14. Urn A has 5 white and 7 black balls. Urn B has 3 white and
12 black balls. We flip a coin. If the outcome is heads, then a ball from
urn A is selected, whereas if the outcome is tails, then a ball from urn B
is selected. Suppose that a white ball is selected. What is the probability

50
Exercises for Homework 5

that the coin landed on tails (to flip – подбросить)?


The answer: 12/37.
5.15. There are four urns. The first urn contains 1 white and 1 black
ball, the second – 2 white and 3 black balls, the third – 3 white and
5 black balls, and the fourth – 4 white and 7 black balls. The event Hi is
the choosing the i-th urn (i = 1, 2, 3, 4). It is known that the probability
of choosing the i-th urn is equal to i/10. A ball is randomly extracted
from a randomly chosen urn. Find the probability that a randomly
extracted ball is white.
The answer: 0,388.

51
LECTURE 6

LECTURE 6

Repetition (Recurrence) of Trials.


The Bernoulli Formula

If several trials are made and the probability of an event A for each
trial doesn’t depend on outcomes of other trials, such trials are called
independent from the event A.
At various independent trials an event A can have either different
probabilities or the same probability. We will further consider only such
independent trials in which the event A has the same probability.
We use below the notion of complex event meaning by that overlapping
of several events, which are called simple.
Let n independent trials will be made in each of which an event A can
either appear or not to appear. Assume that the probability of the event A
for each trial is the same, namely equals p. Consequently, the probability
of non-happening the event A in each trial is also constant and equals
q = 1 – p.
Let’s pose the problem of calculating the probability that for n trials
the event A will happen exactly k times and consequently will
not happen n – k times. It is important to underline that it is not
required the event A repeated exactly k times in a certain sequence.
For example, if the speech is about appearance of the event A three
times in four trials then the following complex events are possible:
AAAA , AAA A, AA AA, A AAA . The entry AAAA , denotes AAA A, AA thatAA
the, A AAA
event A happened at the first, second and third trials and it didn’t happen
at the fourth trial, i.e. the opposite event A happened; the rest entries
have the corresponding sense.
Denote the required probability by Pn(k). For example, the symbol P5(3)
denotes the probability that the event will happen exactly 3 times for
5 trials and consequently it will not happen 2 times.

52
Repetition (Recurrence) of Trials. The Bernoulli Formula

One can solve the posed problem by means of such-called Bernoulli


formula.
Deduction of the Bernoulli formula: The probability of one complex
event consisting in that for n trials the event A will happen k times and
will not happen n – k times is equal by the theorem of multiplication
of probabilities of independent events to pkqn – k. There can be such
complex events as much as combinations of n elements on k elements
can be composed, i.e. Cnk. Since these complex events are incompatible,
by the theorem of addition of probabilities of incompatible events
the required probability is equal to the sum of the probabilities of
all possible complex events. Since the probabilities of all these
complex events are the same, the required probability (of appearance
of the event A k times for n trials) is equal to the probability of one
complex event multiplied on their number:

Pn (k) = C nk p k q n - k or Pn (k) = n!
pk qn-k
k! (n - k) !

Example. The probability that the expense of electric power during one
day will not exceed the established norm is equal to p = 0,75. Find
the probability that at the closest 6 days the expense of electric power
will not exceed the norm for 4 days.
Solution: The probability of normal expense of electric power during
each of 6 days is constant and equals 0,75. Consequently, the probability
of overexpenditure of electric power for each day is also constant
and equals q = 1 – p = 1 – 0,75 = 0,25. The required probability by
the Bernoulli formula is equal to

6$5
P6 (4) = C64 p4 q2 = $ (0, 75) 4 $ (0, 25) 2 = 0, 30.
1$2

The most probable number k0 of occurrences of an event in independent


trials is determined from the double inequality:

53
LECTURE 6

np – q ≤ k0 ≤ np + p

where n – number of trials, p – probability of occurrence of the event


in one trial, q – probability of non-occurrence of the event in one trial.
Example. 40 boxes of glass products have been delivered on
a warehouse. The probability that all products of a randomly taken box
appear intact is equal to 0,9. Find the most probable number of boxes
in which all products appears intact.
Solution: By the hypothesis n = 40, p = 0,9, q = 0,1. The most probable
number of boxes which are not containing damaged products is
determined by the double inequality: 40 · 0,9 – 0,1 ≤ k0 ≤ 40 · 0,9 +0,9
or 35,9 ≤ k0 ≤ 36,9 Therefore the required most probable number
k0 = 36.

Local Theorem of Laplace


It easy to see that using the Bernoulli formula for great values n is
sufficiently difficult because for example if n = 50, k = 30, p = 0,1
then for finding the probability P50(30) it is necessary to calculate the
expression
P50(30) = 50! / (30! · 20!) · (0,1)30 · (0,9)20.
One arises the question: is it possible to calculate probability interested
for us without using the Bernoulli formula? Yes, it can. Local theorem
of Laplace gives the asymptotic formula which allows approximately to
find the probability of appearance of an event exactly k times for n trials
if the number of trials is sufficiently great.
Local theorem of Laplace: If the probability p of appearance of an
event A for each trial is constant and differs from 0 and 1 then the
probability Pn(k) that the event A will appear for n trials exactly k times
is approximately equal to (the more precise, the greater n) the value of
the function
2
1 1 - x2 = 1
y= $ e $ { (x)
npq 2r npq

54
Local Theorem of Laplace

for x = (k - np) / npq .


There are tables (Appendix 1) with values of the function
2
1 - x2
{ (x) = e corresponding to positive values of the argument x.
2r
We use the same tables for negative values of the argument because the
function φ(x) is even, i.e. φ(–x) = φ(x).
Thus, the probability that the event A will happen for n independent
trials exactly k times is approximately equal to
1
Pn (k) . $ { (x)
npq

where x = (k - np) / npq .


Example. Find the probability that an event A will happen exactly
80 times for 400 trials if the probability of appearance of the event for
each trial equals 0,2.
Solution: By the hypothesis, n = 400; k = 80; p = 0,2; q = 0,8.
Use the asymptotic formula of Laplace:
1 1
P400 (80) . $ { (x) = $ { (x).
400 $ 0, 2 $ 0, 8 8
Calculate the value x determined by data of the problem:
k - np 80 - 400 $ 0, 2
x= = = 0.
npk 8

By the table (Appendix 1) we find


1
{ (0) = 0, 3989 & P400 (80) = $ 0, 3989 = 0, 04986.
8
The Bernoulli formula gives almost the same result: P400(80) = 0,0498.

55
LECTURE 6

Example. The probability of hit in a target by a shooter at one shot


is p = 0,75. Find the probability that the shooter will hit in the target
8 times at 10 shots.
Solution: By the hypothesis, n = 10; k = 8; p = 0,75; q = 0,25.
Use the asymptotic formula of Laplace:
1
P10 (8) . $ { (x) = 0, 7301 $ { (x) .
10 $ 0, 75 $ 0, 25
Calculate the value x determined by data of the problem:
k - np 8 - 10 $ 0, 75
x= = . 0, 36.
npq 10 $ 0, 75 $ 0, 25
By the table (Appendix 1) we find
φ(0,36) = 0,3739 ⇒ P10(8) = 0,7301 · 0,3739 = 0,273.
The Bernoulli formula gives another result: P10(8) = 0,282.
Such big difference of results is explained by that n has small value for our
example (the Laplace formula gives sufficiently good approximations
only for sufficiently great values n).

Integral Theorem of Laplace


Suppose that n trials are made in each of which the probability
of appearance of the event A is constant and equals p (0 < p < 1). How
can we calculate probability Pn(k1, k2) that the event A will appear for
n trials no less than k1 and no more than k2 times (for shortness, we will
say “from k1 up to k2 times”)?
Theorem. If probability p of appearance of an event A for each trial
is constant and differs from 0 and 1 then the probability Pn(k1, k2) that
the event A will appear for n trials from k1 up to k2 times is approximately
equal to the definite integral
xm 2
-z
Pn (k1, k2) .
1
2r
#e 2
dz
xl

56
Integral Theorem of Laplace

where xl = (k1 - np) / npq and x m = (k2 - np) / npq .

For solving of problems requiring an application of the integral theorem


of Laplace special tables are used because the indefinite integral
2

#e -z
2
dz is not expressed by elementary functions.
x 2
1 # - z2
The table (Appendix 1) for the integral U (x) = e dz is given
2r 0
for positive values x and for x = 0; it is used the same table for x < 0
(the function Φ(x) is odd, i.e. Φ(– x) = – Φ(x)). There are values of
the integral only till x = 5 in the table because one can assume Φ(x) = 0,5
for x > 5. The function Φ(x) is often called the Laplace function.
Transform the expression (*) as follows:
0 2 xm 2

Pn (k1, k2) .
1
2r
#e -z
2
dz +
1
2r
#e -z
2
dz =
xl 0
xm 2 xl 2

= 1
2r
#e -z
2
dz -
1
2r
#e -z
2
dz = U (x m ) - U (xl )
0 0

i.e.

Pn (k1 , k2 ) ≈ Φ (x″) – Φ(x′)

where xl = (k1 - np) / npq and x m = (k2 - np) / npq .

Example. The probability that an item has not passed a checking by


the quality department is equal to 0,2. Find the probability that there
will be unchecked from 70 up to 100 items among 400 randomly
selected items.
Solution: By the hypothesis, p = 0,2; q = 0,8; n = 400; k1 = 70; k2 = 100.
Use the integral theorem of Laplace: P400(70, 100) ≈ Φ(x″) – Φ(x′).

57
LECTURE 6

Calculate the lower and the upper limits of integration:


k1 - np 70 - 400 $ 0, 2
xl = = =- 1, 25;
npq 400 $ 0, 2 $ 0, 8
k2 - np 100 - 400 $ 0, 2
xm = = = 2, 5.
npq 400 $ 0, 2 $ 0, 8
Thus, we have P400(70, 100) ≈ Φ(2,5) – Φ(– 1,25) = Φ(2,5) + Φ(1,25)
By the table (Appendix 1) we find: Φ(2,5) = 0,4938; Φ(1,25) = 0,3944.
The required probability P400(70, 100) = 0,4938 + 0,3944 = 0,8882.

Glossary
complex event – сложное событие; overlapping – совмещение
expense – расход; electric power – электроэнергия
overexpenditure – перерасход
quality department – отдел технического контроля
warehouse – товарный склад; intact – целый (неповрежденный)

Exercises for Seminar 6


6.1. There are 6 motors in a shop. For each motor the probability that it is
turned (switched) on at present time is equal to 0,8. Find the probability
that at present:
a) 4 motors are turned on;
b) all motors are turned on;
c) all motors are turned off (a shop – цех).
The answer: a) 0,246; b) 0,26; c) 0,000064.
6.2. Find the probability that an event A will appear in five independent
trials no less than two times if the probability of occurrence of the event
A for each trial is equal to 0,3.

58
Exercises for Seminar 6

The answer: 0,472.


6.3. A coin is tossed 6 times. Find the probability that the coin lands on
heads:
a) less than two times;
b) no less than two times.
The answer: a) 7/64; b) 57/64.
6.4. Find approximately the probability that an event will happen exactly
104 times at 400 trials if in each trial the probability of its occurrence
is equal to 0,2.
The answer: 0,0006.
6.5. The probability of striking a target by a shooter at one shot is equal
to 0,75. Find the probability that at 100 shots the target will be struck:
a) no less than 70 and no more 80 times;
b) no more than 70 times.
The answer: a) 0,7498; b) 0,1251.
6.6. The probability that an event A will appear at least once at two
independent trials is equal to 0,75. Find the probability of appearance
of the event in one trial (it is supposed that the probability of appearance
of the event in both trials is the same).
The answer: 0,5.
6.7. A coming up a potato is equal to 80 %. How many is it necessary
to plant potatoes that the most probable number of came up potatoes
of them was equal to 100 (to come up – всходить (о растении))?
The answer: 124 or 125.
6.8. There are 4000 bees in a bee family. The probability of illness
within a day is equal to 0,002 for each bee. Find the probability that
more than one bee will be ill within a day (a bee – пчела).
The answer: 0,99.

59
LECTURE 6

6.9. A coming up a grain stored in a warehouse is equal to 80%. What is


the probability that the number of came up grains among 100 ones will
make from 68 up to 90 pieces (a grain – зерно)?
The answer: 0,992.
6.10. The probability of receiving an excellent mark at an exam is
equal to 0,2. Find the most probable number of excellent marks and
the probability of this number if 50 students pass the exam.
The answer: k0 = 10 and Pn(k0) = 0,141.

Exercises for Homework 6


6.11. An event B will appear in case when an event A will appear no
less than two times. Find the probability that the event B will happen
if 6 independent trials will be made in each of which the probability
of occurrence of the event A is equal to 0,4.
The answer: 0,767.
6.12. 8 independent trials have been made in each of which the probability
of occurrence of an event A is equal to 0,1. Find the probability that
the event A will appear at least 2 times.
The answer: 0,19.
6.13. A factory has sent 5000 good-quality products. The probability
that one product has been damaged at a transportation is 0,002. Find
the probability that at the transportation it will be damaged:
a) 3 products; b) 1 product; c) no more than 3 products.
The answer: a) 0,0107; b) 0,00218; c) 0,0124.
6.14. A shooter has made 400 shots, and the probability of hit in a target
is 0,8. Find the probability that he hits from 310 up to 325 times.
The answer: 0,6284.
6.15. The number of workers of an enterprise is 500 persons. The
probability of absence on the work because of illness is equal to 0,01
for each worker of the enterprise. Determine the probability that at least

60
Exercises for Homework 6

one of workers will not come to work at the nearest day.


The answer: 0,985.
6.16. How many times is it necessary to toss a die in order that the most
probable number of landing 6 aces was equal to 50?
The answer: 299 ≤ n ≤ 305.
6.17. A shooter hits in a target with the probability 0,6. He is going to
make 10 shots. Find the probability that he hits in the target:
a) three times; b) at least once.
The answer: a) 0,0425; b) 0,9999.
6.18. A coming up seeds makes 80%. What is the probability that from
780 up to 820 seeds will come up of 1000 sown seeds?
The answer: 0,8858.
6.19. There are 70 automobiles in a park. The probability of breakage
of an automobile is equal to 0,2. Find the most probable number
of serviceable automobiles and the probability of this number.
The answer: k0 = 14 and Pn(k0) = 0,119.
6.20. 900 students are studying at a faculty. The probability of birthday
in a given day is equal to 1/365 for each student. Find the probability
that there will be three students with the same birthday.
The answer: 0,24.

61
LECTURE 7

LECTURE 7

Random Variables. The Law of Distribution


of a Discrete Random Variable
It is frequently the case when an experiment is performed that we
are mainly interested in some function of the outcome as opposed to
the actual outcome itself. For instance, in tossing dice we are often
interested in the sum of the two dice and are not really concerned about
the separate values of each die. That is, we may be interested in knowing
that the sum is 7 and not be concerned over whether the actual outcome
was (1, 6) or (2, 5) or (3, 4) or (4, 3) or (5, 2) or (6, 1). Also, in coin
flipping, we may be interested in the total number of heads that occur
and not care at all about the actual head-tail sequence that result. These
quantities of interest, or more formally, these real-valued functions
defined on the outcome space, are known as random variables.
A random variable is understood as a variable which as result of a trial
takes one of the possible set of its values (which namely – it is not
beforehand known).
We denote random variables by capital letters of Latin alphabet
X, Y, Z, …, and their values – by the corresponding small letters
x, y, z, ….
Example. The number of the born boys among hundred newborns
is a random variable which has the following possible values:
0, 1, 2, …, 100.
Example. The distance which will be flied by a shell at shot by a gun is
a random variable. Really, the distance depends not only on installation
of a sight, but also from many other reasons (force and direction
of wind, temperature, etc.) which cannot be completely taken into
account. Possible values of this variable belong to some interval (a, b).
Example. Suppose that our experiment consists of tossing 3 coins. If we
let Y denote the number of heads appearing, then Y is a random variable
taking on one of the values 0, 1, 2, 3 with respective probabilities

62
Random Variables. The Law of Distribution of a Discrete Random Variable

1
P (Y = 0) = P ((T, T, T)) =
8
3
P (Y = 1) = P ((T, T, H), (T, H, T), (H, T, T)) =
8
3
P (Y = 2) = P ((T, H, H), (H, T, H), (H, H, T)) =
8
1
P (Y = 3) = P ((H, H, H)) =
8
A discrete random variable is a random variable which takes on
separate, isolated possible values with certain probabilities. The number
of possible values of a discrete random variable can be finite or infinite.
For a discrete random variable X, we define the probability mass
function p(a) of X by
p(a) = P(X = a)
A continuous random variable is a random variable which can take
all values from some finite or infinite interval. Obviously, the number
of possible values of a continuous random variable is infinite.
The most full, exhaustive description of a random variable is its law
of distribution.
Any ratio establishing connection between possible values of a random
variable and probabilities corresponding to them refers to as the law
of distribution of the random variable.
About a random variable speak that it «is distributed» under the given
law of distribution or «subordinated» to this law of distribution.
For a discrete random variable the law of distribution can be set as
a table, analytically (as a formula) and graphically.
The elementary form of assignment of the law of distribution
of a discrete random variable X is a table (matrix) in which all possible
values of a random variable and the probabilities corresponding to them
are listed in ascending order, i.e.

63
LECTURE 7

x1 x2 … xi … xn
p1 p2 … pi pn

or
x1 x2 ... xn
X=c m
p1 p2 ... pn

Such a table is called the series of distribution of a discrete random


variable.
The events X = x1, X = x2, …, X = xn, consisting in that as a result of trial
the random variable X will take on values x1, x2, …, xn respectively, are
incompatible and uniquely possible (because in the table all possible
values of a random variable are listed), i.e. form a complete group.
Hence, the sum of probabilities is equal to 1. Thus, for every discrete
random variable
n n
/ P (X = x ) = / p = 1
i i
i=1 i=1

(This unit is somehow distributed between values of a random variable,


therefore from here the term “distribution”).
A series of distribution can be represented graphically if values
of a random variable are postponed on the axis of abscissas, and
on the axis of ordinates – their corresponding probabilities. Connecting
the received points forms a broken line named a polygon of distribution
of probabilities.
Example. 100 tickets of a monetary lottery are released. One prize in 50
roubles and ten prizes on 1 rouble are played. Find the law of distribution
of a random variable X – cost of a possible prize for an owner of one
lottery ticket.
Solution: Write the possible values of Х: х1 = 50, х2 = 1, х3 = 0. The
probabilities of these possible values are those: р1 = 0,01; р2 = 0,1; р3
= 1 – (р1 + р2) = 0,89.

64
Mathematical operations over random variables

Let’s write the required law of distribution:

X 50 1 0
P 0,01 0,1 0,89

Mathematical operations over random variables


Two random variables are independent if the law of distribution of one
of them does not vary from that which possible values were taken on by
another variable. So, if a discrete random variable X can take on values
xi (i = 1, 2, …, n), and a random variable Y – values yj (j = 1, 2, .., m)
then the independence of the discrete random variables X and Y means
the independence of the events X = xi and Y = yj for all i = 1, 2, .., n and
j = 1, 2, .., m. Otherwise, the random variables are dependent.
For example, if there are tickets of two different monetary lotteries then
the random variables X and Y expressing respectively a prize under
each ticket (in monetary units) will be independent as at any prize under
a ticket of one lottery the law of distribution of a prize under other
ticket will not be changed. If the random variables X and Y express a
prize under tickets of one monetary lottery then in this case X and Y are
dependent since any prize under one ticket (X = xi) results in change of
probabilities of a prize under other ticket (Y = yj), i.e. changes the law
of distribution Y.
Let two random variables X and Y be given:

65
LECTURE 7

X = e x1 x2 ... xn o, Y = e y1 y2 ... yn o .
p1 p2 ... pn pl1 pl2 ... pln
The product kX of a random variable X on a constant k is the random
variable which takes on values kxi with the same probabilities
pi (i = 1, 2, .., n).
The m-th degree of a random variable X, i.e. Xm is the random variable
which takes on values xim with the same probabilities pi (i = 1, 2, .., n).
-2 1 2
Example. Let a random variable X be given: X = e o . Find
0, 5 0, 3 0, 2
the law of distribution of the random variables: a) Y = 3X; b) Z = X2.
Solution: a) The values of the random variable Y will be: 3 · (–2) = –6;
3 · 1 = 3; 3 · 2 = 6 with the same probabilities 0,5; 0,3; 0,2, i.e.
-6 3 6
Y=e o
0, 5 0, 3 0, 2
b) The values of the random variable Z will be: (– 2)2 = 4, 12 = 1, 22 = 4
with the same probabilities 0,5; 0,3; 0,2. Since the value Z = 4 can be
obtained by squaring the values (– 2) with probability 0,5 and (+ 2) with
probability 0,2, under the theorem of addition: P(Z = 4)= 0,5 + 0,2 = 0,7.
Thus, we have the following law of the random variable Z:
1 4
Z=e o
0, 3 0 , 7
The sum (the difference or the product) of random variables X and
Y is the random variable which takes on all possible values of kind
xi + yj (xi – yj or xi · yj) where i = 1, 2, …, n; j = 1, 2, …, m with the
probabilities pij that the random variable X will take on the value xi, and
Y – the value yj:
py = P[(X = xi)(Y = yj)].
If random variables X and Y are independent, i.e. any events X = xi, Y = yj
are independent, then by theorem of multiplication of probabilities for
independent events

66
(Mathematical) Expectation of a Discrete Random Variable

py = P(X = xi) · (Y = yj) = pi ·p′j.

(Mathematical) Expectation of a
Discrete Random Variable
One of the most important concepts in probability theory is the
expectation of a random variable. If X is a discrete random variable
having a probability mass function p(x), the (mathematical) expectation
(the expected value or the mean) of X, denoted by M(X), is defined by

M (X) = / x $ p (x)
x: p (x) > 0

In words, the expected value of X is a weighted average of the possible


values that X can take on, each value being weighted by the probability
that X assumes it. For instance, if the probability mass function of X is
given by

1 =
p (0) = p (1)
2
then
1 + 1 =1
M (X) = 0 $ 1$
2 2 2

is just the ordinary average of the two possible values 0 and 1 that X can
assume. On the other hand, if

1 2
p (0) = p (1) =
3 3
then
1+ 2 =2
M (X) = 0 $ 1$
3 3 3
is a weighted average of the two possible values 0 and 1.
Remark. The concept of expectation is analogous to the physical

67
LECTURE 7

concept of the center of gravity of a distribution of mass. Consider


a discrete random variable X having probability mass function p(xi),
i ≥ 1. If we now imagine a weightless rod in which weights with mass
p(xi), i ≥ 1, are located at the points xi, i ≥ 1, then the point at which
the rod would be in balance is known as the center of gravity. For those
readers acquainted with elementary statics it is now a simple matter
to show that this point is at M(X).
Example. The laws of distribution of random variables X and Y – the
numbers of points beaten out by 1-st and 2-nd shooters are known:

0 1 2 3 4 5
X=e o
0, 25 0, 21 0, 14 0, 15 0, 09 0, 16

0 1 2 3 4 5
Y=e o
0, 11 0, 13 0, 15 0, 11 0, 21 0, 29

It is necessary to find out who of the shooters shoots better.


Solution: Obviously, a shooter shoots better than another shooter if he
beats out more number of points on the average than another one.
M(X) = 0·0,25 + 1·0,21 + 2·0,14 + 3·0,15 + 4·0,09 + 5·0,16 = 2,10.
M(Y) = 0·0,11 + 1·0,13 + 2·0,15 + 3·0,11 + 4·0,21 + 5·0,29 = 3,05.
Thus, the second shooter shoots better than the first one on the average.
If a discrete random variable X takes on an infinite (countable) set of
values x1, x2, …, xn, … then the mathematical expectation or the expected
value of such a discrete random variable is the sum of the following
3
series (if it absolutely converges): M (X) = / xi pi .
i=1

Property 1. The mathematical expectation of a constant is equal to the


constant:
M(C) = C

68
Dispersion of a Discrete Random Variable

Property 2. A constant multiplier can be taken out for a sign


of mathematical expectation, i.e.
M(kX) = kM(X)
Property 3. The mathematical expectation of the algebraic sum
of initely many random variables is equal to the sum of their
mathematical expectations, i.e.
s s

M (/ Xi) = / M (Xi)
i=1 i=1

Property 4. The mathematical expectation of the product of finitely


many mutually independent random variables is equal to the product
of their mathematical expectations:
s s

M (% Xi) = % M (Xi)
i=1 i=1

Property 5. The mathematical expectation of deviation of a random


variable from its mathematical expectation is equal to zero:

M[X – M(X)] = 0

Dispersion of a Discrete Random Variable


Although M(X) yields the weighted average of the possible values
of X, it does not tell us anything about the variation, or spread, of these
values. For instance, although random variables W, Y, and Z, having
probability mass functions determined by
W = 0 with probability 1

Y *
- 1 with probability 1
= 2
+ 1 with probability 1
2

Z=*
- 100 with probability 1
2
+ 100 with probability 1
2

69
LECTURE 7

All have the same expectation – namely, 0 – there is much greater spread
in the possible value of Y than in those of W (which is a constant) and
in the possible values of Z than in those of Y.
As we expect X to take on values around its mean M(X), it would appear
that a reasonable way of measuring the possible variation of X would
be to look at how far apart X would be from its mean on the average.
In practice it is often required to estimate the dispersion (variation)
of possible values of a random variable around of its average value.
For example, in artillery it is important to know as far as shells will
concentrically lie near to the target which should be struck.
One possible way to measure this would be to consider the quantity
M(|X – a|), where a = M(X). However, it turns out to be mathematically
inconvenient to deal with this quantity, and so a more tractable
quantity is usually considered – namely, the expectation of the square
of the difference between X and its mean. We thus have the following
definition:
If X is a random variable with expectation M(X), then the dispersion
(variance) of X, denoted by D(X), is defined by

D(X) = M[X – M(X)]2

An alternative formula for D(X) is derived as follows:


D (X) = M [(X - M (X)) 2] = / (x - M (X)) 2 p (x) =
x

= / (x2 - 2M (X) $ x + (M (X)) 2) p (x) =


x

= / x2 p (x) - 2M (X)/ xp (x) + (M (X)) 2 / p (x) =


x x x

= M [X2] - 2 (M (X)) 2 + (M (X)) 2 = M [X2] - (M (X)) 2 .

That is,

D(X) = M[X2] – (M(X))2

70
Dispersion of a Discrete Random Variable

In words, the dispersion of X is equal to the expected value of X2 minus


the square of its expected value. This is, in practice, often the easiest
way to compute D(X).
Thus, for our example we have D(W) = 0, D(Y) = 1 and D(Z) = 10000.
Example. Calculate D(X) if X represents the outcome when a die is
rolled.
Solution: We have the following law of distribution:

X 1 2 3 4 5 6
p 1/6 1/6 1/6 1/6 1/6 1/6

Consequently,
1 + 1 + 1 + 1 + 1 + 1 =7
M (X) = 1 $ 2$ 3$ 4$ 5$ 6$
6 6 6 6 6 6 2
Also,
1 + 2 1 + 2 1 + 2 1 + 2 1 + 2 1 = 91
M (X2) = 12 $ 2 $ 3 $ 4 $ 5 $ 6 $
6 6 6 6 6 6 6
Hence,
91 - ` 7 j2 = 35
D (X) = .
6 2 12

Remark. Analogous to the mean being the center of gravity


of a distribution of mass, the dispersion (variance) represents, in
the terminology of mechanics, the moment of inertia.
The mean square deviation (the standard deviation) σ(X) of a random
variable X is the arithmetic value of the square root of its dispersion:

v (X) = D (X)

71
LECTURE 7

Property 1. The dispersion of a constant is equal to zero: D(C) = 0.


Property 2. A constant multiplier can be taken out from the argument
of the dispersion involving it in square:
D(kX) = k2 D(X)
Property 3. The dispersion of a random variable is equal to
the difference between the mathematical expectation of the square
of the random variable and the square of its mathematical expectation:
D(X) = M(X2) – [M(X)]2
Property 4. The dispersion of the algebraic sum of finitely many
mutually independent random variables is equal to the sum of their
dispersions:
s s

D (/ Xi) = / D (Xi) .
i=1 i=1

Observe that the dispersion of both the sum and the difference
of independent random variables X and Y is equal to the sum of their
dispersions, i.e.
D(X + Y) = D(X – Y) = D(X) + D(Y).
The mathematical expectation, the dispersion and the mean square
deviation are numerical characteristics of a random variable.

Glossary
shell – артиллерийский снаряд; aim, sight – прицел
exhaustive – исчерпывающий; assignment – задание
ascending order – возрастающий порядок
mathematical expectation, mean value – математическое ожидание
rod – стержень; on the average – в среднем; deviation – отклонение
concentration – кучность; to strike – поражать
dispersion, variance – дисперсия, рассеяние

72
Exercises for Seminar 7

mean square deviation – среднее квадратическое отклонение


numerical characteristic – числовая характеристика
Exercises for Seminar 7
7.1. Two balls are chosen randomly from an urn containing 8 white,
4 black and 2 orange balls. Suppose that we win $2 for each black
ball selected and we lose $1 for each white ball selected. Let X denote
our winnings. What are the possible values of X, and what are the
probabilities associated with each value?
7.2. The probability of working each of four combines without breakages
during a certain time is equal to 0,9. Compose the law of distribution
of a random variable X – the number of combines working trouble-free.
Find the mathematical expectation, the dispersion and the mean square
deviation of the random variable X.
The answer: M(X) = 3,6; D(X) = 0,36; σ(X) = 0,6.
7.3. The probability of birth of a boy in a family is equal to 0,515.
Compose the law of distribution of a random variable X – the number
of boys in families having four children. Find the mathematical
expectation, the dispersion and the mean square deviation.
The answer: M(X) = 2,06; D(X) = 0,999; σ(X) = 1,0.
7.4. There are 6 masters of sports in a group of 10 sportsmen. One
selects (under the circuit without replacement) 3 sportsmen. Compose
the law of distribution of a random variable X – the number of masters
of sports of the selected sportsmen. Find the mathematical expectation
of the random variable X.
The answer: M(X) = 1,8.
7.5. A shooter makes shots in a target before the first hit. The probability
of hit in the target at each shot is equal to 0,7. Compose the law
of distribution of a random variable X – the number of shots made
by the shooter. Find the most probable number of cartridges (patrons)
given to the shooter.
The answer: k0 = 1.

73
LECTURE 7

7.6. The mathematical expectation of a random variable X is equal to 8.


Find the mathematical expectation of the following random variables:
a) X – 4; b) 3X + 4.
7.7. The dispersion of a random variable X is equal to 8. Find
the dispersion of the following random variables: a) X – 2; b) 3X + 2.
7.8. Independent random variables X and Y have the following
distributions:

X 2 4 6 Y 3 4
p 0,3 0,5 0,2 p 0,4 0,6

Compose the law of distribution of the random variable Z = X + Y.


Find the mathematical expectation, the dispersion and the mean square
deviation of the random variable Z.
The answer: M(Z) = 7,4; D(Z) = 2,2.
7.9. Find the mathematical expectation and the dispersion of random
variable Z = 4X – 2Y if M(X) = 5, M(Y) = 3, D(X) = 4, D(Y) = 6.
The random variables X and Y are independent.
The answer: M(Z) = 14; D(Z) = 88.
7.10. A total of 4 buses carrying 148 students from the same school
arrives at a football stadium. The buses carry, respectively, 40, 33, 25,
and 50 students. One of the students is randomly selected. Let X denote
the number of students that were on the bus carrying this randomly
selected student. One of the 4 bus drivers is also randomly selected. Let
Y denote the number of students on his bus. Which of M(X) or M(Y) do
you think is larger? Why? Compute M(X) and M(Y).

Exercises for Homework 7


7.11. Two dice are rolled. Let X equal the sum of the 2 dice. What are
the possible values of X, and what are the probabilities associated with
each value?

74
Exercises for Homework 7

7.12. The probability that a buyer will make a purchase in a shop is


equal to 0,4. Compose the law of distribution of a random variable
X – the number of buyers who have made a purchase if the shop was
visited by 3 buyers. Find the mathematical expectation, the dispersion
and the mean square deviation of the random variable X.
The answer: M(X) = 1,2; D(X) = 0,72; σ(X) = 0,85.
7.13. A buyer attends shops for purchasing the necessary goods. The
probability that the goods are in a certain shop is equal to 0,4. Compose
the law of distribution of a random variable X – the number of shops
which will be attended by the buyer from four possible. Find the most
probable number of shops which will be visited by the buyer.
The answer: 1 ≤ k0 ≤ 2.
7.14. A sample of 3 items is selected at random from a box containing
20 items of which 4 are defective. Find the expected number
(mathematical expectation) of defective items in the sample.
The answer: 0,6.
7.15. A box contains 5 red and 5 blue marbles. Two marbles are
withdrawn randomly. If they are the same color, then you win $1.10;
if they are different colors, then you win – $1.00 (that is, you lose
$1.00). Calculate the mathematical expectation and the dispersion
of the amount you win (marble – мрамор; to withdraw – извлекать).
The answer: M(X) = – 1/15; D(X) = 49/45.
7.16. The mathematical expectation of a random variable X is equal to 7.
Find the mathematical expectation of the following random variables:
a) X + 6; b) 4X – 3.
7.17. The dispersion of a random variable X is equal to 9. Find
the dispersion of the following random variables: a) X + 6; b) 2X – 7.
7.18. Independent random variables X and Y have the following
distributions:

75
LECTURE 7

X 2 4 6 Y 3 4
p 0,3 0,5 0,2 p 0,4 0,6

Compose the law of distribution of the random variable V = XY. Find


the mathematical expectation, the dispersion and the mean square
deviation of the random variable V.
The answer: M(V) = 13,68; D(V) = 29,3376.
7.19. Find the mathematical expectation and the dispersion of random
variables:
a) Z = 2X – 4Y;
b) Z = 3X + 5Y
if M(X) = 5, M(Y) = 3, D(X) = 4, D(Y) = 6. The random variables
X and Y are independent.
The answer: a) M(Z) = – 2; D(Z) = 112; b) M(Z) = 30; D(Z) = 186.

76
Distribution Function of a Random Variable

LECTURE 8

Distribution Function of a Random Variable.

For a random variable X, the function F defined by

F(x) = P(X < x), –∞<x<∞

is called the cumulative distribution function or, more simply,


the distribution function of X. Thus the distribution function specifies,
for all real values x, the probability that the random variable is less than
x.
Sometimes the distribution function F(x) is said to be the integral
function of distribution or the integral law of distribution.
Geometrically the distribution function is interpreted as the probability
that a random variable X will hit to the left from a given point x.
Example 1. Let a series of distribution of a random variable be given:
1 4 5 7
X=e o
0, 4 0, 1 0, 3 0, 2
Find and represent graphically its distribution function.
Solution: Let’s take different values x and find for them F(x) = P(X < x).
1. If x ≤ 1 then obviously F(x) = 0 (and for x = 1, F(1) = P(X < 1) = 0).
2. Let 1 < x ≤ 4 (for example, x = 2); F(x) = P(X = 1) = 0,4. Obviously,
F(4) = P(X < 4) = 0,4.
3. Let 4 < x ≤ 5 (for example, x = 4,2); F(x) = P(X < x) = P(X = 1)+
+P(X = 4) = 0,4 + 0,1 = 0,5. Obviously, F(5) = 0,5.
4. Let 5 < x ≤7. F(x) = [P(X = 1) + P(X = 4)] + P(X = 5) = 0,5 + 0,3 =
= 0,8. Obviously, F(7) = 0,8.

77
LECTURE 8

5. Let x > 7. F(x) = [P(X = 1) + P(X = 4) + P(X = 5)] + P(X = 7) =


= 0,8 + 0,2 = 1.
Thus, Z
]0 if x # 1,
]0, 4 if 1 < x # 4,
]
F (x) = [0, 5 if 4 < x # 5,
]0, 8 if 5 < x # 7,
]
]
\1, 0 if x > 7.

This example allows making the following claim: the distribution


function of any discrete random variable is a discontinuous step function
of which jumps take place in points corresponding to the possible
values of the random variable and are equal to the probabilities of these
values.
The sum of all jumps is equal to 1.

Properties of a Distribution Function


1. The distribution function of a random variable is a non-negative
function taking on values between 0 and 1: 0 ≤ F(x) ≤ 1.
2. The distribution function of a random variable is a non-decreasing
function for the entire numerical axis, i.e. if x1 < x2 then F(x1) ≤ F(x2).
3. F (- 3) = xlim
"-3
F (x) = 0, F (+ 3) = lim F (x) = 1.
x "+3

78
Continuous Random Variables. Probability Density

4. The probability of hit of a random variable in an interval [x1, x2 )


(including x1) is equal to the increment of its distribution function
on this interval, i.e.

P(x1 ≤ X < x2 ) = F(x2 ) – F(x1 ).

Example 2. The distribution function of a random variable X has


the following form:

F (x) = * x/2 if 0 < x # 2,


0 if x # 0,

1 if x > 2.
Find the probability that the random variable will take on a value in
the interval [1; 3).
Solution: P(1 ≤ X < 3) = F(3) – F(1) = 1 – 1/2 =1/2.

Continuous Random Variables. Probability Density


A random variable X is continuous if its function of distribution is
continuous at each point and differentiable everywhere but possibly
finitely many points.
The distribution function of a continuous random variable X which
is differentiable everywhere but three points of break is shown at
the picture.

79
LECTURE 8

Theorem. The probability of a separately taken value of a continuous


random variable X is equal to zero, i.e. P(X = x1) = 0 for each value x1
of the continuous random variable X.
Corollary. If X is a continuous random variable then probability of hit
of the random variable in an interval (x1, x2) doesn’t depend on whether
the interval is open or closed, i.e.
P(x1 < X < x2 ) = P(x1 ≤ X < x2 ) = P(x1 < X ≤ x2 ) = P(x1 ≤ X ≤ x2 )
The probability density (distribution density or simply density) φ(x)
of a continuous random variable X is the derivative of its distribution
function, i.e.

φ(x) = F′(x)

Sometimes the probability density is said to be the differential function


or the differential law of distribution.
The graph of probability density φ(x) is said to be distribution curve.
Example. Let X be a random variable as in Example 2. Find the
probability density of the random variable X.
Solution: The probability density φ(x) = F′(x) i.e.

{ (x) = )0 if x # 0 or x > 2,
1/2 if 0 < x # 2.

Properties of Probability Density


1. The probability density is a non-negative function, i.e. φ(x) ≥ 0.
2. The probability of hit of a continuous random variable in an interval
[a, b] is equal to the definite integral of its density in limits from a to
b, i.e.
b

P (a # X # b) = # { (x) dx
a

80
Properties of Probability Density

3. The distribution function of a continuous random variable can be


expressed by the probability density:
x

F (x) = # { (t) dt
-3

4. The improper integral in infinite limits of the probability density of


a continuous random variable is equal to 1:
+3

# { (x) dx = 1
-3

Formulas of mathematical expectation and dispersion of a continuous


random variable X have the following form:
+3

a = M (X) = # x{ (x) dx (if the integral converges absolutely)


-3

+3

D (X) = # (x - a) 2
{ (x) dx (if the integral converges).
-3

Using D(X) = M(X2) – [M(X)]2, we have D(X) = M(X2) – a2 or


+3

D (X) = # x { (x) dx - a
2 2

-3

Example. Let a function φ(x) be given:


Find: a) the value of the constant A for which the function is
the probability density of some random variable X; b) the expression
of the distribution function F(x); c) the probability that the random
variable X will take on values in the segment [2; 3]; d) the mathematical
expectation and the dispersion of the random variable X.
Solution: a) φ(x) will be the probability density if φ(x) ≥ 0, i.e. A/x4 ≥ 0

81
LECTURE 8

(and consequently, A ≥ 0) and it satisfies the condition Consequently,


+3 1 +3 b

# { (x) dx = # 0 $ dx + # xA dx = 0 + lim # xA dx =
4
b "+3
4
A
lim c-
1 bm =
3 b " + 3 x3 1
-3 -3 1 1

= A lim `1 - 13 j = A = 1.
3 b "+3 b 3

We have A = 3.
x x

b) Find F(x): If x ≤ 1 then F (x) = # { (x) dx = # 0 $ dx = 0.


-3 -3
x x

If x > 1 then F (x) = 0 + # x3 dx =- x1


4 3 = 1 - 13 . Thus,
x
1 1

F (x) = )0 if x # 1,
3
1 - 1/x if x > 1.
3 3

c) P (2 # X # 3) = # x3 dx =- x1
4 3 = 13 - 13 = 19 .
2 3 216
2 2

The probability P(2 ≤ X ≤ 3) can also be found as the increment


of the distribution function, i.e.

P (2 # X # 3) = F (3) - F (2) = `1 -
1 j - ` - 1 j = 19
1 .
33 23 216

+3 1 +3 b

d) a = M (X) = # x{ (x) dx = # 0 $ dx + # x$
3
x4
dx = 0 + 3 lim # 3 =
b "+3
dx
x
-3 -3 1 1

= 3 lim c- 1 2 m = 3 .
b

b "+3 2x 1 2
Since D(X) = M(X2) – a2,

82
Glossary

+3 1 +3
2
M (X ) = # x { (x) dx =
2
# 2
x $ 0dx + #x 2
$
3
x4
dx =
-3 -3 1

= lim `- 3 j = lim `3 - 3 j = 3.
b

b"3 x 1 b"3 b

Then D (X) = 3 - ` 3 j = 3 .
2

2 4

Glossary
distribution function – функция распределения
jump of a function – скачок функции
step discontinuous function – ступенчатая разрывная функция
increment – приращение; distribution curve – кривая распределения
probability density – плотность вероятности
Exercises for Seminar 8
8.1. Let the law of distribution of a discrete random variable be given:

X 1 4 6 8
P 0,1 0,3 0,4 0,2

Find the integral function of the random variable X and construct its
graph.
8.2. Find the integral function of distribution of the random variable
X – the number of hits in a target if three shots were made with the
probability of hit in the target equal 0,8 for each shot.
8.3. A continuous random variable X is given by the integral function:
Z
]]0 3 if x # 0,
x
F (x) = [ if 0 < x # 5,
]] 125
\1 if x > 5.

83
LECTURE 8

Determine:
a) the probability of hit of the random variable into the interval
(2; 3);
b) the mathematical expectation, the dispersion and the mean
square deviation of the random variable X.
8.4. A random variable X is given by the integral function:
Z
]]0 if x # - 2,
x 1
F (x) = [ + if - 2 < x # 2,
]] 4 2
\1 if x > 2.
Find the probability that in result of a trial the random variable X will
take on the value:
(a) less than 0;
(b) less than 1;
(c) no less than 1;
(d) being in the interval (0; 2).
8.5. The amount of time, in hours, that a computer functions before
breaking down is a continuous random variable with probability density
function given by

f (x) = ) me
-x/100
if x $ 0,
0 if x 1 0.

What is the probability that


(a) a computer will function between 50 and 150 hours before
breaking down;
(b) it will function less than 100 hours?
Direction: Take e equal 2,718281.
The answer: a) 0,3834; b) 0,632.

84
Exercises for Seminar 8

8.6. The lifetime in hours of a certain kind of radio tube is a random


variable having a probability density function given by

0 if x # 100,
f (x) = * 100
if x > 100.
x2

What is the probability that exactly 2 of 5 such tubes in a radio set will
have to be replaced within the first 150 hours of operation? Assume
that the events Ei, i = 1, 2, 3, 4, 5, that the ith such tube will have to be
replaced within this time, are independent.
The answer: 80/243.
8.7. Let X be a random variable with probability density function

f (x) = )C (1 - x ) if - 1 < x < 1,


2

0 otherwise.
(a) What is the value of C?
(b) What is the cumulative distribution function of X?
The answer: a) 3/4.
8.8. Compute M(X) if X has a density function given by
1 -x/2
f (x) = * 4 xe if x > 0,
0 otherwise.
The answer: 4.
8.9. A random variable X is given by the differential function:
Z
]]0 if x # 0,
4a - 2x
f (x) = [ 3 if 0 < x # a,
]] 3a
\0 if x > a.

85
LECTURE 8

Find:
(a) the integral function;
(b) the probability of hit of the random variable into the interval
(a/6; a/3).
The answer: b) 7/36.
8.10. A random variable X is given by the integral function:
Z
]]0 3 if x # 2,
x -8
F (x) = [ if 2 < x # 3,
]] 19
\1 if x > 3.
Find:
(a) the differential function;
(b) the probability of hit of the random variable X into the interval
(2,5; 3);
(c) the mathematical expectation, the dispersion and the mean
square deviation of the random variable X.
The answer: b) 0,599; c) M(X) = 2,566; D(X) = 0,079.

Exercises for Homework 8


8.11. Let the law of distribution of a discrete random variable be given:

X –2 5 7 9
p 0,4 0,3 0,2 0,1
Find the integral function of the random variable X and construct its
graph.
8.12. The probability of passing the first exam by a student is 0,7, the
second exam – 0,6 and the third exam – 0,8. Find the integral function
of the random variable X – the number of exams passed by the student.
Determine M(X).

86
Exercises for Homework 8

The answer: M(X) = 2,1.


8.13. The density function of X is given by

f (x) = )a + bx if 0 # x # 1,
2

0 otherwise.
If M(X) = 3/5, find a and b.
The answer: a = 3/5; b = 6/5.
8.14. A system consisting of one original unit and a spare can function
for a random amount of time X. If the density of X is given (in units
of months) by

f (x) = )Cxe
-x/2
if x > 0,
0 if x # 0.
what is the probability that the system functions for at least 5 months
(a spare – запасной элемент)?
The answer: 0,616.
8.15. Suppose that X is a continuous random variable whose probability
density function is given by

f (x) = )C (4x - 2x ) if 0 < x < 2,


2

0 otherwise.
(a) What is the value of C?
(b) Find P(X > 1).
The answer: a) 3/8; b) 1/2.
8.16. Find M(X) and D(X) when the density function of X is

f (x) = '2x if 0 # x # 1,
0 otherwise.
The answer: M(X) = 2/3; D(X) = 1/18.

87
LECTURE 8

8.17. Let X be a random variable with probability density function


5
f (x) = *C (2x - x ) if 0 < x < 2 ,
3

0 otherwise.
(a) What is the value of C?
(b) What is the cumulative distribution function of X?
The answer: a) – 64/225.
8.18. Compute M(X) if X has a density function given by
50
f (x) = * x3 if x > 5,
0 otherwise.
The answer: 10.
8.19. A random variable X is given by the differential function:

f (x) = * x + x if 0 < x #
0 if x # 0,
3
5 - 1,
0 if x> 5 -1.
Find:
(a) the integral function;
(b) the probability of hit of the random variable into the interval
(1; 1,1).
The answer: b) 0,221.
8.20. A random variable X is given by the integral function:
Z
]]0 2 if x # 1,
x x
f (x) = [ - if 1 < x # 2,
]] 2 2
\1 if x > 2.

88
Exercises for Homework 8

Find:
(a) the differential function;
(b) the probability of hit of the random variable X into the interval
(1; 1,5);
(c) the mathematical expectation, the dispersion and the mean
square deviation of the random variable X.
The answer: b) 0,375; c) M(X) = 19/12; D(X) = 11/144.

89
LECTURE 9

LECTURE 9

Basic Laws of Distribution of


Discrete Random Variables.
Binomial law of distribution.
A discrete random variable X has a binomial law of distribution with
parameters n and p if it takes on values 0, 1, 2, …, m, …, n with
probabilities

P(X = m) = Cnmpmqn-m

where 0 < p < 1, q = 1 – p.


The binomial law of distribution represents the law of distribution
of the number X = m of occurrences of an event A in n independent
trials in each of which it can occur with the same probability.

xi 0 1 2 … m … n
pi qn Cn1p1qn-1 Cn2p2qn-2 … Cnmpmqn-m … pn

The series of distribution of a binomial law has the following form:


n n
Obviously, /p = 1i because / p =is1 the
i sum of all members of
i=0 i=0

decomposition of Newton binomial:

qn + Cn1pqn-1 + Cn2p2qn-2 + Cnmpmqn-m + ... + pn = (q + p)n = 1n = 1

Therefore, the law is said to be binomial.


Theorem. The mathematical expectation of a random variable X
distributed under a binomial law is M(X) = np, and its dispersion
D(X) = npq.

90
2. The law of Poisson Distribution

The binomial law of distribution is widely used in the theory and


practice of statistical control by a production quality, at the description
of functioning of systems of mass service, at modeling the prices
of actives, in the theory of shooting and in other areas.
Example. Footwear has arrived in a shop from two factories in the ratio
2:3. 4 pairs of footwear have been bought. Find the law of distribution
of the number of the bought pairs of footwear made by the first factory.
Find the mathematical expectation and the dispersion of this random
variable.
Solution: The probability that a randomly chosen pair of footwear
has been made by the first factory is p = 2/(2 + 3) = 0,4. The random
variable X – the number of bought pairs of footwear made by the first
factory among 4 selling pairs – has the binomial law of distribution
with parameters n = 4, p = 0,4. The series of distribution of X has
the following form:

xi 0 1 2 3 4
pi 0,1296 0,3456 0,3456 0,1536 0,0256

The values pi = P(X = m), where m = 0, 1, 2, 3, 4, are calculated by


the formula:

P(X = m) = C4m · 0,4m · 0,64-m

Find the mathematical expectation and the dispersion of the random


variable X:
M(X) = np = 4 · 0,4 = 1,6; D(X) = npq = 4 · 0,4 · 0,6 = 0,96.

2. The law of Poisson Distribution


A discrete random variable X has the law of Poisson distribution with
parameter λ > 0 if it takes on values 0, 1, 2, …, m, …(infinite countable
set of values) with probabilities

91
LECTURE 9

mm e- m
P (X = m) =
m!
The series of distribution of the Poisson law has the following form:

xi 0 1 2 … m …
pi e-λ λe-λ λ2e-λ/2! … λme-λ/m! …

Since the sum of the series


3
/p = e
2 -m m -m

i + me- m + m e + ... + m e + ... =


-m

i=1 2! m!
= e- m c1 + m + m + ... + m + ... m = e- m e m = 1,
2 m

2! m!
n
the basic property of distribution series /p = 1 i holds, and
consequently the Poisson law is well-defined. i=0

The Poisson probability distribution was introduced by S.D. Poisson


in a book he wrote regarding the application of probability theory
to lawsuits, criminal trials, and the like.
Theorem. The mathematical expectation and the dispersion of a random
variable distributed under a Poisson law coincide and are equal to
the parameter λ of the law, i.e. M(X) = λ, D(X) = λ.
The Poisson random variable has a tremendous range of applications
in diverse areas because it may be used as an approximation
for a binomial random variable with parameters (n, p) when n is large
and p is small enough so that np is a moderate size. In other words, if
n independent trials, each of which results in a success with probability
p, are performed, then, when n is large and p small enough to make
np moderate, the number of successes occurring is approximately
a Poisson random variable with parameter λ = np.
Some examples of random variables that usually obey the Poisson
probability law follow:

92
2. The law of Poisson Distribution

1. The number of misprints on a page (or a group of pages) of a book.


2. The number of people in a community living to 100 years of age.
3. The number of wrong telephone numbers that are dialed in a day.
4. The number of packages of dog biscuits sold in a particular store
each day.
5. The number of customers entering a post office on a given day.
6. The number of vacancies occurring during a year in the federal
judicial system.
7. The number of α-particles discharged in a fixed period of time from
some radioactive material.
Each of the preceding, and numerous other random variables, are
approximately Poisson for the same reason – namely, because
of the Poisson approximation to the binomial. For instance, we can
suppose that there is a small probability p that each letter typed
on a page will be misprinted. Hence the number of misprints on a page
will be approximately Poisson with λ = np, where n is the number
of letters on a page.
Example. Suppose that the number of typographical errors on a single
page of this book has a Poisson distribution with parameter λ = 1/2.
Calculate the probability that there is at least one error on this page.
Solution: Letting X denote the number of errors on this page, we have
P(X ≥ 1) = 1 – P(X = 0) = 1 – e-1/2 = 0,393.
Example. Suppose that the probability that an item produced
by a certain machine will be defective is 0,1. Find the probability that
a sample of 10 items will contain at most 1 defective item.
Solution: The desired probability is

P (X = 0) + P (X = 1) = C 100 (0,1)0 (0,9)10 + C 10


1
(0,1)1  (0,9)9 = 0,7361,

whereas the Poisson approximation yields the value

93
LECTURE 9

P(X = 0) + P(X = 1) = e-1 + e-1 ≈ 0,7358.

Geometric Distribution
A discrete random variable X has a geometric distribution with the
parameter p if it takes on values 1, 2, …, m, … (infinite countable set
of values) with probabilities

P(X = m) = pqm–1

where 0 < p < 1, q = 1 – p.


The series of a geometric distribution has the following form:
It is easy to see that the probabilities pi form the geometric progression

xi 1 2 3 … m …
pi p pq pq 2
… pq m-1

with the first member p and denominator q (therefore, the law is said
to be geometric).

Since / pi = p + pq + ... + pq m - 1 + ... = p (1 + q + ... + q m - 1 + ...) = p $ 1 = 1,


1-q
the geometric distribution is well-defined.
A random variable X having a geometric distribution represents
the number m of trials which have been carried out under Bernoulli
circuit with probability p of occurrence of the event in each trial till the
first positive outcome.
Theorem. The mathematical expectation of a random variable X having
the geometrical distribution with parameter p is M(X) = 1/p, and its
dispersion D(X) = q/p2 where q = 1 – p.
Example. Testing a big batch of items up to detection of a rejected
item (without restriction of the number of tested items) is carried
out. Compose the law of distribution of the number of tested items.

94
Hypergeometric Distribution

Find its mathematical expectation and dispersion if it is known that


the probability of reject for each item is equal to 0,1.
Solution: The random variable X – the number of tested items
up to detection of rejected item – has geometrical distribution with
the parameter p = 0,1. Therefore, the series of distribution has
the following form:

xi 1 2 3 4 … m …
pi 0,1 0,09 0,081 0,0729 … 0,9m -1 · 0,1 …

M(X) = 1/p = 1/0,1 = 10; D(X) = q/p2 = 0,9/(0,1)2 = 90.

Hypergeometric Distribution
Hypergeometric distribution is widely used in practice of statistical
acceptance control by quality of industrial production, in the problems
connected to the organization of sampling inspections, and other areas.
A discrete random variable X has the hypergeometric distribution with
parameters n, M, N if it takes on values 0, 1, 2, …, m, …, min(n, M)
with the probabilities

C Mm C Nn --mM
P (X = m) =
C Nn

where M ≤ N, n ≤ N; n, M, N are natural numbers.


Let there be M standard items in a batch of N items. n items are
randomly selected from the batch (each item can be extracted with
the same probability), and the selected item is not replaced in the batch
before selecting the next item (therefore the Bernoulli formula here
is not applicable). Then the random variable X which is the number
m of standard items among n selected items has hypergeometric
distribution.

95
LECTURE 9

Theorem. The mathematical expectation of a random variable X having


a hypergeometric distribution with parameters n, M, N is M (X) = n M
N
and its dispersion D (X) = n M `1 - j`1 - j .
M n
N-1 N N
Example. In a lottery «Sportloto 6 of 45» monetary prizes are received
by participants who have guessed 3, 4, 5 and 6 kinds of sports from
randomly selected 6 kinds of 45 (the size of a prize increases with
an increasing the number of guessed kinds of sports). Find the law
of distribution of a random variable X – the number of guessed kinds
of sports among randomly selected 6 kinds. What is the probability
of receiving a monetary prize? Find the mathematical expectation and
the dispersion of the random variable X.
Solution: Obviously, the number of guessed kinds of sports in the lottery
“6 of 45” is a random variable having hypergeometric distribution with
the parameters n = 6, M = 6, N = 45. The series of its distribution has
the following form:

xi 0 1 2 3 4 5 6
pi 0,40056 0,42413 0,15147 0,02244 0,00137 0,00003 0,0000001

The probability of receiving a monetary prize


6

P (3 # X # 6) = / P (X = i) = 0, 02244 + 0, 00137 +
i=3

+ 0, 00003 + 0, 0000001 = 0, 02384 . 0, 024.

M(X) = n · M / N = 6 · 6/45 = 0,8;

D(X) = 6 · 39/44 (1 – 39/45)(1 – 6/45) = 0,6145.

96
Glossary

Glossary
binomial – биномиальный; Poisson – Пуассон
lawsuit – судебный процесс; tremendous – огромный
moderate – небольшой, доступный; diverse areas – разнообразные
области
particle – частица; to discharge – разряжать
circuit – схема; hypergeometric – гипергеометрический

Exercises for Seminar 9


9.1. A die is tossed three times. Write the law of distribution
of the number of appearance of 6.
9.2. Find an average number (mathematical expectation) of typing
errors on page of the manuscript if the probability that the page of the
manuscript contains at least one typing error is 0,95. It is supposed that
the number of typing errors is distributed under the Poisson law (typing
error – опечатка; an average number – среднее число).
The answer: 3.
9.3. The switchboard of an enterprise serves 100 subscribers.
The probability that a subscriber will call on the switchboard within
1 minute is equal 0,02. Which of two events is more probable:
3 subscribers will call or 4 subscribers will call within 1 minute?
(Subscriber – абонент, switchboard – коммутатор).
9.4. A die is tossed before the first landing «six» aces. Find
the probability that the first appearance of «six» will take place:
(a) at the second tossing the die;
(b) at the third tossing the die;
(c) at the fourth tossing the die.
The answer: (a) 5/36.

97
LECTURE 9

9.5. Suppose that a batch of 100 items contains 6 that are defective
and 94 that are non-defective. If X is the number of defective items in
a randomly drawn sample of 10 items from the batch, find (a) P(X = 0)
and (b) P(X > 2).
9.6. There are 7 standard items in a set of 10 items. 4 items are randomly
taken from the set. Find the law of distribution of the random variable X
equal to the number of standard items among the taken items.
9.7. An urn contains 5 white and 20 black balls. 3 balls are randomly
taken from the urn. Compose the law of distribution of the random
variable X equal to the number of taken out white balls.
9.8. At horse-racing competitions it is necessary to overcome four
obstacles with the probabilities equal 0,9; 0,8; 0,7; 0,6 respectively.
At the first failure the sportsman in the further competitions does not
participate. Compose the law of distribution of a random variable
X – the number of taken obstacles. Find the mathematical expectation
of the random variable X (obstacle – препятствие).
The answer: M(X) = 2,4264.
9.9. Two shooters make on one shot in a target. The probability of hit
by the first shooter at one shot is 0,5, and by the second shooter – 0,4.
(а) Find the law of distribution of the random variable X –
the number of hits in the target;
(b) Find the probability of the event X ≥ 1.
The answer: b) 0,7.
9.10. A set of families has the following distribution on number
of children:

xi x1 x2 2 3
pi 0,1 p2 0,4 0,35

Determine x1, x2, p2, if it is known that M(X) = 2, D(X) = 0,9.

98
Exercises for Homework 9

Exercises for Homework 9


9.11. Compose the law of distribution of probabilities of the number
of appearances of the event A in three independent trials if the
probability of appearance of the event is 0,6 for each trial.
9.12. Let X be a random variable equal to the number of boys in families
with five children. Assume that probabilities of births of both boy and
girl are the same. Find the law of distribution of the random variable X.
Find the probabilities of the following events:
(a) there are 2-3 boys in a family;
(b) no more than three boys;
(c) more than 1 boy.
The answer: a) 5/8; b) 13/16; c) 13/16.
9.13. A factory has sent 5000 suitable items to its warehouse.
The probability that an item is broken during a transportation is
0,0002. Find the probability that 3 non-suitable items will be arrived
at the warehouse.
The answer: 0,06.
9.14. Suppose that the number of accidents occurring on a highway
each day is a Poisson random variable with parameter λ = 3.
(a) Find the probability that 3 or more accidents occur today.
(b) Repeat part (a) under the assumption that at least 1 accident
occurs today.
The answer: a) 0,577; b) 0,627.
9.15. A hunter shoots on a game before the first hit, but he managed
to make no more than four shots. The probability of hit by him at one
shot is 0,9.
(а) Find the law of distribution of a random variable X – the number
of misses;
(b) Find the probability of the following events: X < 2, X ≤ 3,

99
LECTURE 9

1 < X ≤ 3 (hunter – охотник; game – дичь).


The answer: b) 0,99; 0,9999; 0,0099.
9.16. There are 11 students in a group, and 5 of them are girls. Compose
the law of distribution of the random variable X – the number of girls
from randomly selected three students.
9.17. There are 8 pencils in a box, and 5 of them are green. 3 pencils are
randomly taken from the box.
(a) Find the law of distribution of a random variable X equal to
the number of green pencils among taken.
(b) Find the probability of the event: 0 < X ≤ 2.
The answer: b) 45/56.
9.18. There are 20 products in a set, and 4 of them are defective.
3 products are randomly chosen for checking their quality. Compose
the law of distribution of a random variable X – the number of defective
products contained in the sample.
9.19. The probability of successful passing an exam by the first student
is 0,7, and by the second – 0,8. Compose the law of distribution
of a random variable X – the number of the students successfully passed
the exam if each of them can retake only once the exam if he didn’t pass
it at the first time. Find the mathematical expectation of the random
variable X.
The answer: M(X) = 1,87.
9.20. A discrete random variable X is given by the following law
of distribution:

xi 1 x2 x3 3
pi 0,1 p2 0,5 0,1
Determine x2, x3, p2, if it is known that M(X) = 4, M(X 2) = 20,2.
The answer: x2 = 2; x3 = 6 or x2 = 7; x3 = 3.

100
Basic Laws of Distribution of Continuous Random Variables

LECTURE 10

Basic Laws of Distribution of


Continuous Random Variables
The Uniform Law of Distribution
A continuous random variable X has a uniform law of distribution
on the segment [a; b] if its probability density φ(x) is a constant on
the segment and equals 0 outside of the segment, i.e.
1
{ (x) = * b - a if a # x # b,
0 if x < a or x > b.
The distribution curve φ(x) and the graph of the distribution function
F(x) of the random variable X are the following:

Theorem. The distribution function of a random variable X distributed


under a uniform law is

F (x) = *(x - a) / (b - a) if a < x # b,


0 if x # a,

1 if x > b.
Its mathematical expectation and its dispersion are the following:

a+b (b - a) 2
M (X) = , D (X) =
2 12

101
LECTURE 10

The uniform law of distribution is used at an analysis of mistakes


of a rounding off at carrying out of numerical calculations (for example,
the mistake of a rounding off of a number up to the whole is uniformly
distributed in the interval [– 0,5; + 0,5], in a series of problems of
mass service, at statistical modeling the observations subordinated to
the given distribution.
Example. Trains of underground go on a regular basis with an interval
of 2 minutes. A passenger leaves on a platform at a random moment
of time. What is the probability that the passenger has to wait no more
than half of minute? Find the mathematical expectation and the mean
square deviation of a random variable X – the time of waiting a train.
Solution: The random variable X – the time of waiting a train on temporal
(in minutes) segment [0; 2] has the uniform law of distribution with
φ(x) = 1/2. Therefore, the probability that the passenger has to wait no
more than half of minute is equal to 1/4 of the area of the rectangle, i.e.

0, 5

# 12 dx = 12 x +
= 1 ; M (X) = 0 2 = 1
0, 5
P (X # 0, 5) =
0
0 4 2
(2 - 0) 2
D (X) = = 1 , v (X) = D (X) . 0, 58 min .
12 3

Exponential Law of Distribution


A continuous random variable X has an exponential law of distribution
with the parameter λ > 0 if its probability density has the following
form:

{ (x) = ) me
- mx
for x $ 0,
0 for x < 0.
The exponential distribution often arises, in practice, as being
the distribution of the amount of time until some specific event occurs.
For instance, the amount of time (starting from now) until an earthquake
occurs, or until a new war breaks out, or until a telephone call you

102
Exponential Law of Distribution

receive turns out to be a wrong number are all random variables that
tend in practice to have exponential distributions.
The distribution curve φ(x) and the graph of function of distribution
F(x) of the random variable X are the following:

Theorem. The function of distribution of a random variable X distributed


under an exponential law is F (x) = )0 for x < 0,
- mx
1-e for x $ 0.
Its mathematical expectation M (X) = 1 and its dispersion D (X) = 12
m m
From the theorem follows that for a random variable distributed under
an exponential law, the mathematical expectation is equal to the mean
square deviation, i.e. M(X) = σ(X) = 1/λ.
The exponential law of distribution plays the big role in the theory
of mass service and the theory of reliability.
Example. It has been established that the time of repairing a TV is
a random variable X distributed under an exponential law. Determine
the probability that it is required no less than 20 days for repairing a TV
if the average time of repairing TVs makes 15 days. Find the probability
density, the distribution function and the mean square deviation of the
random variable X.
Solution: By the hypothesis the mathematical expectation
M(X) = 1/λ = 15, and consequently λ = 1/15, and the probability density
and the distribution function have the following form:

103
LECTURE 10

1 - 151 x -1 x
{ (x) = e ; F (x) = 1 - e 15 (x $ 0) .
15
The required probability P(X ≥ 20) can be found by integrating
the probability density:
+3 +3

P (X $ 20) = P (20 # X < + 3) = # { (x) dx = # 15


1
e
-1 x
15
dx
20 20

But it is simply to do this by using the distribution function:


P (X $ 20) = 1 - P (X < 20) = 1 - F (20) =
- 20 - 20
= 1 - (1 - e 15 ) = e 15 = 0, 264
Find the mean square deviation: σ(X) = M(X) = 15 days.

Normal Law of Distribution


The normal law of distribution is most frequently met in practice.
The main feature allocating it among other laws is that it is the limiting
law to which other laws of distribution are approximated at rather
frequently meeting typical conditions.
The normal distribution was introduced by the French mathematician
Abraham DeMoivre in 1733 and was used by him to approximate
probabilities associated with binomial random variables when
the binomial parameter n is large. This result was later extended
by Laplace and others and is now encompassed in a probability theory
known as the central limit theorem, which is discussed later. The central
limit theorem, one of the two most important results in probability
theory, gives a theoretical base to the often noted empirical observation
that, in practice, many random phenomena obey, at least approximately,
a normal probability distribution. Some examples of this behavior are
the height of a man, the velocity in any direction of a molecule in gas,
and the error made in measuring a physical quantity.
A continuous random variable X has a normal law of distribution
(Gauss law) with the parameters a and σ2 if its probability density has
the following form:

104
Normal Law of Distribution

1 2
( x - a)
{N (x) = e- 2v 2

v 2r
Theorem. The mathematical expectation of a random variable X
distributed under a normal law is M(X) = a, and its dispersion D(X) = σ2.
The normal law of distribution of a random variable with the parameters
a = 0, σ2 = 1, i.e. N(0; 1), is said to be standard or normalized.

A curve of the normal law of distribution is said to be normal or Gauss


curve.
Theorem. The distribution function of a random variable X distributed
under a normal law is expressed by the Laplace function Φ(x):

1 + `x-aj
FN (x) = U
2 v
x
1 # - t2 2

where U (x) = e dt. .


2r 0
Property 1. The probability of hit of a random variable X distributed
under a normal law in the interval [x1; x2] is

P (x1 # X # x2) = U (t2) - U (t1)

where t1 = x1 - a , t2 = x2 - a .
v v
Property 2. The probability that deviation of a random variable X
distributed under a normal law from the mathematical expectation a

105
LECTURE 10

doesn’t exceed the quantity Δ > 0 (on absolute value) is equal to

P(|X – a| ≤ Δ) = 2 Φ(t)

where t = Δ/σ.
The rule of three sigmas: If a random variable X has the normal
distribution with the parameters a and σ2, i.e. N(a; σ2), then it is
practically reliable that its values are in the interval (a – 3σ; a + 3σ),
i.e.

P (| X - a | < 3v) = 2U`


3v j =
2U (3) = 2 $ 0, 4965 = 0, 9973
v
The violation of «the rule of three sigmas», i.e. the deviation of normally
distributed random variable X is more than on 3σ (by absolute value),
is an event which is practically impossible since its probability is rather
small:

P (| X - a | > 3v) = 1 - P (| X - a | # 3v) = 1 - 0, 9973 = 0, 0027.

Example. Assuming that the height of men of a certain age group is


a normally distributed random variable X with the parameters a = 173
and σ2 = 36, find:
1. a) the probability density and the distribution function of the random
variable X;
b) parts of suits of the 4-th height (176–182) and the 3-rd height
(170 –176) which need to be provided in total amount of a factory for
a given age group.
2. Formulate «the rule of three sigmas» for the random variable X.
Solution: 1. a) We have {N (x) = 1 e- 2 $ 36 ;
2
(x - 173)

6 2r
x
1 #e -
(x - 173)2 1 + ` x - 173 j
FN (x) = 2 $ 36 dx = U .
6 2r -3
2 6

106
Normal Law of Distribution

b) The part of suits of the 4-th height (176 – 182 cm) in total amount
of the factory is determined as probability:
P (176 # X # 182) = U (t2) - U (t1) = U (1, 50) - U (0, 50) =
= 0, 4332 - 0, 1915 = 0, 2417,

since t1 = 176 - 173 = 0, 50; t2 = 182 - 173 = 1, 50.


6 6

The part of suits of the 3rd height (170 – 176 cm) can be determined
analogously, but it is simply to make by the following way:

P (170 # X # 176) = P (| X - 173 | # 3) = 2U` j = 2 $ 0, 1915 = 0, 383.


3
6
2. It is practically reliable that the height of men of the given age
group is enclosed in boundaries from a – 3σ = 173 – 3 · 6 = 155 to
a + 3σ = 173 + 3 · 6 = 191, i.e. 155 ≤ X ≤ 191 (cm).
Remark. DeMoivre, who used the normal distribution to approximate
probabilities connected with coin tossing, called it the exponential
bell-shaped curve. Its usefulness, however, became truly apparent
only in 1809, when the famous German mathematician K.F. Gauss
used it as an integral part of his approach to predicting the location
of astronomic entities. As a result, it became common after this time to
call the Gaussian distribution.
During the mid to late nineteenth century, however, most statisticians
started to believe that the majority of data sets would have histograms
conforming to the Gaussian bell-shaped form. Indeed, it came to be
accepted that it was “normal” for any well-behaved data set to follow
this curve. As a result, following the lead of the British statistician Karl
Pearson, people began referring to the Gaussian curve by calling it
simply the normal curve.

107
LECTURE 10

Glossary
uniform – равномерный; exponential – показательный;
sigma – сигма; to round off – округлить; reliability – надежность;
violation – нарушение; part – доля; temporal – временной
bell-shaped – конусообразный; to encompass – заключать
to conform – подчиняться; lead – первенство

Exercises for Seminar 10


10.1. A random variable X is uniformly distributed in the interval
(–2; N). Find:
(a) the differential function of the random variable X;
(b) the integral function;
(c) the probability of hit of the random variable into the interval
(–1; N/2);
(d) the mathematical expectation, the dispersion and the mean
square deviation the random variable X.
The answer: c) 0,5.
10.2. Buses arrive at a specified stop at 15-minute intervals starting
at 7 a.m. That is, they arrive at 7, 7:15, 7:30, 7:45, and so on. If
a passenger arrives at the stop at a time that is uniformly distributed
between 7 and 7:30, find the probability that he waits:
(a) less than 5 minutes for a bus;
(b) more than 10 minutes for a bus.
The answer: a) 1/3; b) 1/3.
10.3. You arrive at a bus stop at 10 o’clock, knowing that the bus will
arrive at some time uniformly distributed between 10 and 10:30.
(a) What is the probability that you will have to wait longer than
10 minutes?

108
Exercises for Seminar 10

(b) If at 10:15 the bus has not yet arrived, what is the probability
that you will have to wait at least an additional 10 minutes?
The answer: a) 2/3; b) 1/3.
10.4. A random variable X is distributed under an exponential law with
parameter λ = 0,5. Find:
(a) the probability density and the distribution function of X;
(b) the probability of hit of the random variable X into the interval
(2; 4);
(c) the mathematical expectation, the dispersion and the mean
square deviation of X.
The answer: b) 0,233.
10.5. The time (in hours) required to repair a machine is an exponentially
distributed random variable with parameter λ = 0,5. What is
(a) the probability that a repair time exceeds 2 hours;
(b) the conditional probability that a repair takes at least 10 hours,
given that its duration exceeds 9 hours?
The answer: a) 0,3679; b) 0,9956.
10.6. Jones figures that the total number of thousands of miles that
an auto can be driven before it would need to be junked is an exponential
random variable with parameter 1/20. Smith has a used car that he
claims has been driven only 10,000 miles. If Jones purchases the car,
what is the probability that she would get at least 20,000 additional
miles out of it? Repeat under the assumption that the lifetime mileage
of the car is not exponentially distributed but rather is (in thousands
of miles) uniformly distributed over (0, 40) (to figure – считать;
to junk – утилизировать).
The answer: 0,7614; 0,75.
10.7. A normally distributed random variable X is given by
2
(x - 5 )
1
the differential function: f (x) = $ e 32 . Determine:
-

4 2r

109
LECTURE 10

(a) the mathematical expectation and the dispersion of the random


variable X;
(b) the probability of hit of the random variable X into the interval
(3; 9).
The answer: b) 0,5328.
10.8. A random variable X is distributed under a normal law with
mathematical expectation a = 20. The probability of hit of the random
variable X into the interval (10; 30) is 0,6826. Determine the probability
of hit of the random variable into the interval (10; 25).
The answer: 0,5328.
10.9. If X is a normal random variable with parameters a = 3 and σ2 = 9,
find (a) P(2 < X < 5); (b) P(X > 0); (c) P(|X – 3| > 6).
The answer: a) 0,3779; b) 0,8413; c) 0,0456.
10.10. Let X be a normal random variable with mathematical expectation
12 and dispersion 4. Find the value of C such that P(X > C) = 0,1.

Exercises for Homework 10


10.11. If X is uniformly distributed over (0, 10), calculate the probability
that
(a) X < 3; (b) X > 6; (c) 3 < X < 8.
The answer: a) 0,3; b) 0,4; c) 0,5.
10.12. Trains headed for destination A arrive at the train station at
15-minute intervals starting at 7 a.m., whereas trains headed for
destination B arrive at 15-minute intervals starting at 7:05 a.m.
(a) If a certain passenger arrives at the station at a time uniformly
distributed between 7 and 8 a.m. and then gets on the first train that
arrives, what proportion of time does he or she go to destination A?
(b) What if the passenger arrives at a time uniformly distributed
between 7:10 and 8:10 a.m.?
The answer: a) 2/3; b) 7/12.

110
Exercises for Homework 10

10.13. A uniformly distributed random variable X is given


by the probability density f(x) = 0,125 in the interval (a – 4; a + 4),
and f(x) = 0 if otherwise. Find:
(a) the distribution function of X;
(b) the probability of hit of X into the interval (a – 3; a + 1);
(c) the mathematical expectation, the dispersion and the mean
square deviation of X.
The answer: b) 0,5.
10.14. A random variable X is distributed under an exponential law with
parameter λ = 2. Find:
(a) the probability density and the distribution function of X;
(b) the probability of hit of the random variable X into the interval
(0; 1);
(c) the mathematical expectation, the dispersion and the mean
square deviation of X.
The answer: b) 0,8647.
10.15. The number of years a radio functions is exponentially distributed
with parameter λ = 1/8. If Jones buys a used radio, what is the probability
that it will be working after an additional 8 years?
The answer: 0,3679.
10.16. Suppose that the length of a phone call in minutes is
an exponential random variable with parameter λ = 1/10. If someone
arrives immediately ahead of you at a public telephone booth, find
the probability that you will have to wait
(a) more than 10 minutes;
(b) between 10 and 20 minutes.
The answer: a) 0,3679; b) 0,2326.

111
LECTURE 10

10.17. A normally distributed random variable X is given


2

by the differential function: f (x) = 1 -x


$ e 2 . Find the interval in
2r
which the random variable X will hit in result of trial with the probability

0,9544.
The answer: (– 2; 2).
10.18. If X is a normal random variable with parameters a = 10 and
σ2 = 36, compute: (a) P(X > 5); (b) P(4 < X < 16); (c) P(|X – 5| > 9).
The answer: a) 0,7967; b) 0,6826; c) 0,2613.
10.19. Suppose that X is a normal random variable with mathematical
expectation 5. If P(X > 9) = 0,2, approximately what is D(X)?
The answer: 22,5625.
10.20. The productivity of a winter wheat on set of allotments is
distributed under a normal law with parameters: a = 50 centner/hectare,
σ = 10 c/h.
Determine:
(a) the percentage of allotments which will have the productivity
more than 40 c/h;
(b) the percentage of allotments with the productivity from 45
up to 60 c/h (productivity – урожайность; winter wheat – озимая
пшеница; allotment – участок).
The answer: a) 84,13%; b) 53,28%.

112
The Law of Large Numbers and Limit Theorems

L E C T U R E 11

The Law of Large Numbers and Limit Theorems


The law of large numbers in a broad sense is understood as the general
principle according to which, under the formulation of academician
A.N. Kolmogorov, cumulative action of the large number of random
factors leads (at the some rather general conditions) to a result almost
independent from a case. In other words, at the large number of random
variables their average result ceases to be random and can be predicted
with the big degree of definiteness.
The law of large numbers in a narrow sense is understood as a series
of mathematical theorems, in each of which for those or other
conditions the fact of approximation of average characteristics
of the large number of trials to some certain constants is established.
Theorem (The Markov inequality). If a random variable X takes only
non-negative values and has a mathematical expectation M(X) then for
every positive number A the following inequality holds:

P(X > A) ≤ M(X)/A

Since the events X > A and X ≤ A are opposite, changing P(X > A) by
1 – P(X ≤ A) we have another form of the Markov inequality:

P(X ≤ A) ≥ 1 – M(X)/A

The Markov inequality is applicable for all non-negative random


variables.
Example. The average amount of calls entering on the switchboard
of a factory within one hour is equal to 300. Estimate the probability
that the number of calls on the switchboard within a randomly chosen
hour: a) will exceed 400; b) will be no more than 500.

113
LECTURE 11

Solution:
a) By the hypothesis M(X) = 300. By the Markov inequality we have
P(X >400) ≤ 300/400, i.e. the probability that the number of calls will
exceed 400 is no more than 0,75.
b) By another form of the Markov inequality P(X ≤ A) ≥ 1 – 300/500
= 0,4, i.e. the probability that the number of calls will be no more than
500 is no less than 0,4.
Example. The sum of all deposits in a branch of bank makes 2 million
roubles, and the probability that a randomly taken deposit will not
exceed 10 thousand roubles is equal to 0,6. What is it possible to tell
about the number of depositors?
Solution: Let X be the size of a randomly taken deposit, and n be
the number of all deposits. Then by the hypothesis the average size
of deposit M(X) = 2000/n (thousand roubles). By the Markov inequality:

P (X ≤ 10) ≥ 1 – M (X) /10 or P(X ≤ 10) ≥ 1 – 2000/10n

Taking in account that P(X ≤ 10) = 0,6 we obtain 1 – 200/n ≤ 0,6; and
consequently n ≤ 500, i.e. the number of depositors is no more than 500.
Theorem (The Chebyshev inequality). For every random variable
X having a mathematical expectation M(X) and a dispersion D(X) the
following inequality holds:

P(|X – a| > ε) ≤ D(X)/ε2

where a = M(X), ε > 0.


Since the events |X – a| > ε and |X – a| ≤ ε are opposite, the Chebyshev
inequality can be written in another form:

P(|X – a| ≤ ε) ≥ 1 – D(X)/ε2

The Chebyshev inequality is applicable for all random variables.

114
The Law of Large Numbers and Limit Theorems

Example. The average expense of water on a cattle-breeding farm


makes 1000 litres in a day, and the mean square deviation of this
random variable does not exceed 200 litres. Estimate the probability
that the expense of water on the farm in a randomly chosen day will not
exceed 2000 litres by using: a) the Markov inequality; b) the Chebyshev
inequality.
Solution: a) Let X be an expense of water on a cattle-breeding farm. By
the hypothesis M(X) = 1000. Using the Markov inequality we obtain
P(X ≤ 2000) ≥ 1 – 1000/2000 = 0,5, i.e. no less than 0,5.
b) The dispersion D(X) = σ2 ≤ 2002. Since the boundaries of the interval
0 ≤ X ≤ 2000 are symmetric according to the mathematical expectation
M(X) = 1000, for estimating the probability of the required event one
can apply the Chebyshev inequality:
2002 =
P (X # 2000) = P (0 # X # 2000) = P (| X - 1000 | # 1000) $ 1 - 0, 96,
10002

i.e. no less than 0,96. In the present example an estimating the probability
of the event found by the Markov inequality (P ≥ 0,5) was made more
precisely by the Chebyshev inequality (P ≥ 0,96).
The Chebyshev theorem. If the dispersions of n independent random
variables X1, X2, …, Xn are bounded by the same constant C then
the arithmetic mean of random variables converges on probability to
the arithmetic mean of their mathematical expectations a1, a2, …, an for
an unbounded increasing the number n, i.e.

lim P` # fj = 1
X1 + X2 + ... + Xn - a1 + a2 + ... + an
n"3 n n
and

P` # fj $ 1 - 2
X1 + X2 + ... + Xn - a1 + a2 + ... + an C
n n nf

115
LECTURE 11

Underline the sense of the Chebyshev theorem. For a large number

n of random variables X1, X2, …, Xn it is practically certain that their


n
arithmetic mean X = c/ Xi m /n – a random variable – very little differs
i=1
n
from non-random quantity c/ ai m /n , i.e. it practically ceases to be
i=1

random.
Corollary. If independent random variables X1, X2, …, Xn have identical
mathematical expectations which equal a, and their dispersions are
bounded by the same constant C then

lim P` a # fj = 1
X1 + X2 + ... + Xn -
n"3 n
and

P` a # fj $ 1 - 2
X1 + X2 + ... + Xn - C
n nf

The Chebyshev theorem and its corollary have the big practical value.
For example, it is necessary for an insurance company to establish
the size of an insurance payment which an insurant should pay; thus
the insurance company undertakes to pay a certain insurance sum at
occurrence of an insurance case. Considering frequency/losses of an
insurant at occurrence of an insurance case as a random variable and
possessing by known statistics of such cases, it is possible to determine
the average number/average losses at happening of insurance cases
which on the basis of the Chebyshev theorem with the big degree
of confidence can be considered as almost non-random quantity. Then
the size of an insurance payment is determined on the basis of these
data and the prospective insurance sum. Without taking into account
action of the law of large numbers (the Chebyshev theorem) essential
losses of the insurance company are possible (at understating the
size of an insurance payment), or loss of appeal of insurance services
(at overestimating the size of a payment).

116
The Law of Large Numbers and Limit Theorems

Example. For determination of the average time of burning electric


lamps in a batch of 200 identical boxes it has been taken on sample
on one lamp from each box. Estimate the probability that the average
duration of burning the selected 200 electric lamps differs from the
average duration of burning lamps in all batch no more than on 5
hours (by absolute value) if it is known that the mean square deviation
of duration of burning lamps in each box is less than 7 hours.
Solution: Let Xi be duration of burning a lamp taken from the i-th box
(hours). By the hypothesis the dispersion D (Xi ) < 72 = 49. Obviously,
the average duration of burning the selected lamps is equal to
(X1 + X2 + … + X200)/200, and the average duration of burning lamps in
all batch is (M (X1 ) + M (X2 ) + … + M (X200 )) / 200 = (a1 + a2 + … +
+a200) / 200. Then the probability of the required event is

P` # 5j $ 1 -
X1 + X2 + ... + X200 - a1 + a2 + ... + a200 49
. 0, 9902
200 n200 200 $ 52

i.e. no less than 0,9902.


The Bernoulli theorem. The relative frequency of an event in n
repeated independent trials, in each of which the event can occur
with the same probability p, converges on probability to probability p
of the event in a single trial for an unbounded increasing the number
n:

lim P` p # fj = 1
m-
n"3 n
and
pq
P` p < fj $ 1 - 2
m-
n nf

The Bernoulli theorem gives a theoretical substantiation of replacement


of unknown probability of an event by its relative frequency obtained in
n repeated independent trials spent at the same complex of conditions.

117
LECTURE 11

So, for example, if the probability of a birth of boy isn’t known, as its
value we can accept the relative frequency of this event which on long-
term statistical data makes approximately 0,515.
Example. A coin is tossed 1000 times. Give a lower bound for
the probability that the absolute value of a deviation of the relative
frequency of appearance of heads from the probability of its appearance
is less than 0,1.
Solution: Here n = 1000, p = q = 0,5; ε = 0,1. Consequently,

1 1
$
P` < 0, 1 j $ 1 -
m - 1 2 2 = 39 = 0, 975
1000 2 1000 $ (0, 1) 2 40

The Central Limit Theorem


The central limit theorem is one of the most remarkable results
in probability theory. Loosely put, it states that the sum of a large
number of independent random variables has a distribution that is
approximately normal. Hence it not only provides a simple method for
computing approximate probabilities for sums of independent random
variables, but it also helps explain the remarkable fact that the empirical
frequencies of so many natural populations exhibit bell-shaped (that is,
normal) curves.
In its simplest form the central limit theorem is as follows:
Theorem (the central limit theorem). Let X1, X2, …, Xn be a sequence
of independent and identically distributed random variables each having
mathematical expectation a and dispersion σ2. Then the distribution of
X1 + X2 + ... + Xn - na
U=
v n
tends to the standard normal as n → ∞, i.e. U is a normally distributed
random variable with M(U) = 0 and D(U) = 1.
Thus, for – ∞ < t < ∞,

118
The Central Limit Theorem

t
X + X2 + ... + Xn - na
Pc 1
v n
# t m " U (t) =
1
2r
#e - x 2 /2
dx
-3

as n → ∞
Example. 100 numbers have been randomly chosen on the segment
[0; 1], more precisely 100 independent random variables X1, X2, …, Xn
uniformly distributed over the segment [0; 1] are considered. Find the
probability that their sum is between 51 and 60, i.e. P (51 ≤ Σ Xi ≤ 60).

Solution: By the central limit theorem U =


/ X - na
i
is a normally
v n
distributed random variable with M(U) = 0 and D(U) = 1.
Then / Xi - na = v n $ U

By uniformity of the random variable Xi we have:

1+0 = 1 (1 - 0) 2
a = M (Xi) = , v2 =
2 2 12

Then: M (/ Xi) = M (na + v n $ U) = na = 100 $ 1 = 50,


2

D (/ Xi) = D (na + v n $ U) = nv2 = 100 $


1 = 100
.
12 12

Thus, / Xi ! N (na, nv2) is a normally distributed random variable with


the mathematical expectation na = 50 and the dispersion nσ2 = 100/12.
Consequently,

119
LECTURE 11

P (51 # / Xi # 60) = U c
60 - na - 51 - na =
m Uc m
n $v n $v
= U 60 50 - U 51 50 = U ( 12 ) - U c
- - 12 m
f p f p
=
10 10 10
12 12
= U (3, 464) - U (0, 3464) . 0, 49971 - 0, 1353 = 0, 3644.

Glossary
to cease – перестать, прекращать; cumulative – совокупный
approximation – приближение; switchboard – коммутатор
cattle-breeding farm – животноводческая ферма
to undertake – обязаться, ручаться
to understate – понижать, преуменьшать
appeal – привлекательность; substantiation – толкование
long-term – многолетний; loosely – свободно

Exercises for Seminar 11


11.1. The amount of electric power consumed by a settlement within
day is a random variable of which the mathematical expectation is
4 thousands of kWt·h. Estimate the probability that for the nearest
day the consumption of energy: a) will exceed 8 thousands of kWt·h;
b) will not exceed 6 thousands of kWt·h (settlement – поселок; electric
power – электроэнергия; consumption – потребление).
The answer: a) 0,5; b) 0,33.
11.2. From past experience a professor knows that the test score of a
student taking his or her final examination is a random variable with
mathematical expectation 75. Give an upper bound for the probability
that a student’s test score will exceed 85. Suppose, in addition, the
professor knows that the dispersion of a student’s test score is equal
to 25. What can be said about the probability that a student will score
between 65 and 85?

120
Exercises for Seminar 11

The answer: 0,882; 0,75.


11.3. A discrete random variable X is given by the following law
of distribution:

X 2 3 6 9
P 0,1 0,4 0,3 0,2
By using the Chebyshev inequality estimate the probability that
|X –  M(X)| > 3.
The answer: 2/3.
11.4. A random variable is given by the integral function:
Z0 if x # a,
]] 2
(x - a)
F (x) = [ 2 if a < x # 2a,
]] a
\1 if x > 2a.
(a) By using the Chebyshev inequality estimate the probability that
|X – M(X)| < a/2; (b) Determine the probability that |X – M(X)| < a/2.
The answer: a) 7/9; b) 35/36.
11.5. By using the Chebyshev inequality estimate the probability
that the number of come up seeds among the sown 5000 will be
from 3750 up to 4250 if the mathematical expectation M(X) = 4000
(seed – семя; to come up – всходить; sown – посеянный).
The answer: 0,9872.
11.6. There are 100 automobiles in an economy. The probability of
non-failure operation of each of them during a certain period is equal
to 0,9. By using the Chebyshev inequality estimate the probability that
the deviation of the number of trouble-free working automobiles for
the certain period from its mathematical expectation will not exceed
5 by absolute value (economy – хозяйство).
The answer: 0,64.

121
LECTURE 11

11.7. By a sampling way the weight of ears of barley is determined.


How many ears need to select in order to assert with probability no less
than 0,99 that the average weight of randomly selected ears will differ
from the average weight of ears in all batch (accepted as a mathematical
expectation) no more than on 0,1 g? It has been established that
the mean square deviation of the weight does not exceed 0,2 g (ear –
колос; barley – ячмень).
The answer: 400.
11.8. Let as a result of 100 independent trials values of a random
variable X have been found: х1, х2, …, х100. Let the mathematical
expectation M(X) = 10 and the dispersion D(X) = 1. Give a lower bound
for the probability that the absolute value of the difference between
the arithmetic mean of observed values of the random variable and
the mathematical expectation will be less than 0,5.
The answer: 0,96.
11.9. There are 100 white and 100 black balls in an urn. 50 balls have
been taken out (with returning). Give a lower bound for the probability
that the amount of white balls from the taken out balls satisfies to
the double inequality: 15 < m < 35.
The answer: 0,875.
11.10. An astronomer is interested in measuring, in light years,
the distance from his observatory to a distant star. Although
the astronomer has a measuring technique, he knows that, because
of changing atmospheric conditions and normal error, each time
a measurement is made it will not yield the exact distance but
merely an estimate. As a result the astronomer plans to make a series
of measurements and then use the average value of these measurements
as his estimated value of the actual distance. If the astronomer believes
that the values of the measurements are independent and identically
distributed random variables having a common mathematical
expectation a (the actual distance) and a common dispersion of 4 (light
years), how many measurements need he make to be guaranteed with
probability 0,95 that the deviation of the arithmetic mean of these
measurements from the mathematical expectation will be no more than

122
Exercises for Homework 11

0,5 light year by absolute value?


The answer: 62.
11.11. Let Xi, i = 1, 2, …, 10, be independent random variables, each
uniformly distributed over (0, 1). Calculate an approximation to
10

P c/ Xi > 6 m by using the central limit theorem.


i=1

The answer: 0,1368.

Exercises for Homework 11


11.12. The amount of forages spent on a farm of large horned livestock
in a day is a random variable of which the mathematical expectation
is 6 tons. Estimate the probability that for the nearest day the expense
of forages on the farm will exceed 10 tons (forage – корм; large horned
livestock – крупный рогатый скот).
The answer: 0,6.
11.13. The number of automobiles sold weekly at a certain dealership
is a random variable with mathematical expectation 16. Give an upper
bound to the probability that next week’s sales exceed 25. Suppose, in
addition, that the dispersion of the number of automobiles sold weekly
is 9. Give a lower bound to the probability that next week’s sales are
between 10 and 22 inclusively (dealership – представительство).
The answer: 0,64; 0,75.
11.14. A discrete random variable X is given by the following law
of distribution:

X –1 0 1 3 5
P 0,1 0,2 0,4 0,2 0,1

123
LECTURE 11

By the Chebyshev inequality estimate the probability that


|X – M(X)| < 2,5.
The answer: 0,5456.
11.15. The probability of ripening seeds of vegetable culture in a given
district is equal to 0,8. By using the Chebyshev inequality estimate
the probability that the number of plants with ripened seeds will make
of 1000 plants from 750 up to 850 (ripening – вызревание; plant –
растение).
The answer: 0,936.
11.16. Suppose that it is known that the number of items produced
in a factory during a week is a random variable with mathematical
expectation 50.
(a) What can be said about the probability that this week’s
production will exceed 75?
(b) If the dispersion of a week’s production is known to equal
25, then what can be said about the probability that this week’s
production will be between 40 and 60?
The answer: a) 2/3; b) 3/4.
11.17. A random variable is given by the integral function:

Z
]]0 2 if x # a,
x
F (x) = [ 2 if a < x # 2a,
]] 4a
\1 if x > 2a.

(a) By the Chebyshev inequality estimate the probability that


|X – M(X)| < a;
(b) Determine the probability that |X – M(X)| < a.
The answer: a) 7/9; b) 35/36.

124
Exercises for Homework 11

11.18. From past experience a professor knows that the test score
of a student taking his or her final examination is a random variable with
mathematical expectation 75 and dispersion 25. How many students
would have to take the examination to ensure, with probability at least
0,9, that the class average would be within 5 of 75?
The answer: 10.
11.19. As a result of 200 independent trials values of a random variable
X: х1, х2, …, х200 have been found, and M(X) = D(X) = 2. Give a lower
bound for the probability that the absolute value of the difference
between the arithmetic mean of the values of the random variable and
the mathematical expectation will be less than 1/5.
The answer: 3/4.
11.20. A die is tossed 10000 times. Estimate the probability
of the deviation of the relative frequency of occurrence of six aces from
the probability of occurrence of the same number of aces will be less
than on 0,01.
The answer: 0,861.
11.21. Use the central limit theorem to solve Exercise 11.18.
The answer: 3.
11.22. Let X1, X2, …, X20 be independent Poisson random variables
with mathematical expectation 1. Use the central limit theorem
20
to approximate P c/ Xi > 25 m
i=1

The answer: 0,1318.

125
LECTURE 12

LECTURE 12

Mathematical Statistics. Variation


series and Their Characteristics
Introduction to Statistics
All life a person has to make decisions: in personal sphere (in which
university or college to enter, with whom to communicate; how
to study); in public (to attend evenings, theatres, meetings, assemblies,
elections); in industrial (determining factors essentially influencing on
productivity, quality of materials and etc.); in scientific (nominating and
checking scientific hypotheses).
Decision-making usually pursues one of the following purposes:
forecasting a future state of a process (an object); controlling (i.e. how
should to change certain parameters of an object in order that other
parameters have taken on a desirable value); an explaining an internal
structure of an object.
Usually decision-making is preceded with an analysis of known
data (based on previous experience, common sense, intuition, and
etc.). Aspiring to see and prove regularities in uncertain processes,
the humanity has developed the whole arsenal of methods that refer to
as mathematical statistics (applied statistics or data analysis).
Mathematical statistics is a section of mathematics in which
mathematical methods of ordering, processing and using statistical data
for scientific and practical conclusions are studied.
Abraham Wald (1902-1950) spoke that «mathematical statistics is
theory of decision-making in conditions of uncertainty».
In essence, mathematical statistics gives a unique, mathematically
proved apparatus for solving problems of control and forecasting at
absence of obvious regularities (presence of randomness) in investigated
processes.
Two kinds of statistical observations are distinguished in practice:
continuous when all objects of a set are studied, and not continuous,

126
Introduction to Statistics

sampling when a part of objects is studied. An example of continuous


observation is the population census covering all population of
the country. A sampling observation is, for example, spent sociological
investigations covering a part of the population of a country, district
and etc.
All set of objects (observations) subject to studying is said to be a parent
population. A part of objects which is selected for direct studying from
a parent population is said to be a sample population or a sample.
Numbers of objects (observations) of a parent or sample population are
said to be their volumes (sizes). A parent population can have both finite
and infinite volume.
The essence of a sampling method is to make a judgment on properties
of a parent population as a whole by some its part (by a sample).
Advantages of sampling:
• It allows to save essentially expenses of resources (material, labor,
time);
• It is uniquely possible in case of an infinite parent population
or in case when the investigation is connected with destruction
of observable objects (for example, investigation of durability
of electric bulbs, limiting operating modes of devices, etc.);
• At the same expenses of resources one enables to conduct a deep
research due to expanding a program of research;
• It allows to lower mistakes of registration, i.e. divergences between
true and registered values of an attribute.
The basic lack of a sampling – mistakes of research named mistakes
of representation. However inevitable mistakes arising at a sampling
of research in connection with studying only a part of objects can
be appreciated beforehand and by correct organizing a sampling are
reduced to practically insignificant quantities.
To have an opportunity to judge on a parent population by sample data,
they should be selected randomly. A sample is representative if it well
enough reproduces a parent population.

127
LECTURE 12

We distinguish the following types of samples:


• Properly random sample formed by a random choice of elements
without a partition on parts or groups;
• Mechanical sample in which elements from a parent population
are selected through a certain interval. For example, if the volume
of sample should make 10 % (10-percentage sample), each 10-th its
element is selected and etc.;
• Typical sample in which one randomly selects elements from
typical groups into which by some attribute a parent population is
partitioned;
• Serial sample in which one randomly selects not elements, and
the whole groups of a population (series), and these series are
subjected to a continuous observation.
One uses two methods of forming a sample:
• Selection with replacement (under the circuit of a replaced ball)
when each element is randomly selected and replaced in a parent
population and can be repeatedly selected;
• Selection without replacement (under the circuit of a non-replaced
ball) when a selected element is not replaced in a parent population.

Variation Series
In real social and economic systems it is impossible to carry out
active experiments; therefore data usually represent observations
over an occurring process, for example: an exchange rate at a stock
exchange within a month, productivity of wheat in a farm for 30 years,
labor productivity of workers for a shift, etc.
Results of observations are generally a series of numbers located in
a disorder which for studying it is necessary to order (rank). The operation
of ordering the values of an attribute by increasing (decreasing) is said
to be ranking of experimental data.
After operation of ranking the experimental data can be grouped so
that in each group the attribute took on the same value which is called

128
Variation Series

a variant (xi), i.e. distinct values of the attribute are variants. The number
of elements in each group is called the frequency (ni) of a variant.
The sum of all frequencies is equal to a certain number n which is called
the volume of population (or sample size):
k
/n = n + n
i 1 2 + ... + nk = n
i=1

The ratio of frequency of the given variant to the volume of population


is the relative frequency (wi) of this variant: wi = ni /n. Frequencies and
relative frequencies are also said to be weights.
A variation series is a ranked series of variants with the corresponding
weights (frequencies and relative frequencies) in increasing
(or decreasing) order.
At studying variation series alongside with the notion of frequency the
notion of cumulative frequency (niсит) is used. Cumulative frequency
shows how many variants with the value of an attribute smaller x
were observed. The ratio of cumulative frequency niсит to the volume
of population n is said to be cumulative relative frequency (wiсит).
For a variation series it is enough to specify variants and frequencies
(relative frequencies) corresponding to them or cumulative frequencies
(cumulative relative frequencies).
Variation series are discrete and continuous (interval). A discrete
variation series is a ranked sequence of variants with the corresponding
frequencies and (or) relative frequencies.
Example 1. As a result of testing a group of 24 persons has obtained the
following points: 4, 0, 3, 4, 1, 0, 3, 1, 0, 4, 0, 0, 3, 1, 0, 1, 1, 3, 2, 3, 1,
2, 1, 2. Construct the discrete variation series.
Solution: Rank the original series, account frequency and relative
frequency of variants:
0, 0, 0, 0, 0, 0, 1, 1, 1, 1, 1, 1, 1, 2, 2, 2, 3, 3, 3, 3, 3, 4, 4, 4.
As a result we obtain the following discrete variation series:

129
LECTURE 12

Point, xi Frequency (the number of Relative frequency, wi


students) ni
0 6 6/24
1 7 7/24
2 3 3/24
3 5 5/24
4 3 3/24
Σ 24 1

If the number of values of an attribute is great, the construction of


a discrete variation series is inexpedient. In this case it is necessary to
construct an interval variation series. For construction of such a series
the interval of variation of an attribute is subdivided into a series of
partitioned intervals, and the amount of values of the quantity in each
of them is counted.

The recommended number of intervals is calculated by the following


formula: m = 1 + 3,322lg n, and the size of an interval (an interval
difference, width of an interval) – k = xmax - xmin where xmax – xmin
1 + 3, 322 lg n
is the difference between the greatest and the least values of an attribute.
Example 2. Let a series of distribution of farms by amount of workers
on 100 hectares of agricultural fields be given (n = 60):

12 6 8 10 11 7 10 12 8 7 7 6 7 8 6
11 9 11 9 10 11 9 10 7 8 8 11 9 8 7
5 9 7 7 14 11 9 8 7 4 7 5 5 10 7
7 5 8 10 10 15 10 10 13 12 11 15 6 6 8

130
Empirical Distribution Function

Find the recommended number of intervals: m = 1 + 3,322 lg 60 ≈ 6,907;


m = 7.
Find the size (length) of partial interval: k = (15 – 4)/7 ≈ 1,6.
Construct an interval variation series using xmin as an initial value. Divide
the interval of variation of the attribute X into m = 7 partial intervals
with step k = 1,6 and count the number of workers on 100 hectares of
agricultural fields in each interval:

Groups of farms Frequency Cumulative Relative frequency,


by amount of (the number frequency wi
workers on 100 of farms in the (cumulative
hectares group) ni number of
farms) niсит
4 – 5,6 5 5 5/60
5,61 – 7,2 17 22 17/60
7,21 – 8,8 9 31 9/60
8,81 – 10,4 15 46 15/60
10,41 – 12,0 10 56 10/60
12,01 – 13,6 1 57 1/60
13,61 – 15,2 3 60 3/60
Σ 60 - 1

Empirical Distribution Function


One of the ways of processing a variation series is to build an empirical
distribution function.
The empirical (statistical) distribution function is called the following
function: F*(x) = nx / n,
where n is the sample size, nx is the number of observations that are less

131
LECTURE 12

than x (x ∈ R).
An empirical function has the following properties:
Property 1. The values of an empirical function belong to the segment
[0; 1].
Property 2. F*(x) is a non-decreasing function.
Property 3. If x1 is the least variant, and xk is the greatest variant then
F*(x) = 0 for x ≤ x1 and F*(x) = 1 for x > xk.

xi 1 4 6
ni 10 15 25

Example 3. Find the empirical function of the following distribution:


Solution: Find the sample size: n = 10 + 15 + 25 = 50. The least variant
is equal to 1, therefore F*(x) = 0 for x ≤ 1. The value x < 4, namely
x1 = 1, was observed 10 times, and consequently F*(x) = 10/50 = 0,2
for 1 < x ≤ 4.
The values x < 6, namely x1 = 1 and x2 = 4, were observed 10 + 15 = 25
times; consequently F*(x) = 25/50 = 0,5 for 4 < x ≤ 6. Since x = 6 is
the greatest variant, F*(x) = 1 for x > 6.
Write the required empirical function:
Z
]0 for x # 1,
]
F * (x) = [0, 2 for 1 < x # 4,
]0, 5 for 4 < x # 6,
]
\1 for x > 6.

Polygon and Histogram


Variation series are presented graphically by a polygon and (or)
a histogram.

132
Numerical Characteristics of Variation Series

Polygon of frequencies is a broken line of which segments connect


points (x1; n1), (x2; n2), …, (xk; nk).
Polygon of relative frequencies is a broken line of which segments
connect points (x1; n1/n), (x2; n2/n), …, (xk; nk /n).
Construct a polygon of frequencies for Example 1:

The figure consisting of rectangles with the basis k and heights ni is called
a histogram of frequencies. For a histogram of relative frequencies ni /n
is considered as a height.
Construct a histogram of frequencies for Example 2.

Numerical Characteristics of Variation Series


Variation series allow receiving a first representation on a studied
distribution. Further it is necessary to investigate numerical
characteristics of distribution.
Arithmetic mean of a variation series is the sum of products of all variants
on the corresponding frequencies divided on the sum of frequencies:

133
LECTURE 12

m
/x n i i m

x= i=1
= / xi wi
n i=1

where xi are variants of a discrete variation series or middles of intervals


of an interval variation series; ni are the corresponding to them
m
frequencies n = / ni ; m is the number of intervals or non-repeating
i=1

variants, wi are the relative frequencies of variants or intervals.

Properties of arithmetic mean


1. If all the results of observations multiply on the same number then
the following holds: z = Cx = Cx .
2. If we add to (subtract from) all the results of observations the same
number then z = x ! C = x ! C
3. If all the frequencies of variants multiply on the same number then
arithmetic mean is not changed.
Mean linear (absolute) deviation of a variation series is the arithmetic
mean of absolute quantities of deviations of variants from their
arithmetic mean:
m
/ x-x i ni
i=1
d=
n
For example, if we have the following series:

xi 1 3 6 16
ni 4 10 5 1

then

134
Properties of arithmetic mean

4 $ 1 + 10 $ 3 + 5 $ 6 + 1 $ 16 = 80 =
x= 4;
4 + 10 + 5 + 1 20
4 $ 1 - 4 + 10 $ 3 - 4 + 5 $ 6 - 4 + 1 $ 16 - 4
d= = 2, 2.
20
The mean linear deviation serves to characterize a variance of a variation
series.
Median Me of a variation series is the value of the attribute falling on
the middle of ranked series of observations. If a discrete variation series
has 2n + 1 members in a ranked set: x1, x2, …, xn, xn+1, …, x2n+1 then
Me = xn+1. If a discrete variation series has 2n members in a ranked set:
x1, x2, …, xn, xn+1, …, x2n then Me = (xn + xn+1)/2.
Mode Mo of a variation series is the variant to which the greatest
frequency is corresponded.
The elementary parameter of a variation is the variation scope R which is
equal to the difference between greatest and least variants of the series:
R = xmax – xmin.
Example 1. Let the following distribution be given:

xi 0 1 2 3 4
ni 6 7 3 5 3

We have: x = 0 $ 6 + 1 $ 7 + 2 $ 3 + 3 $ 5 + 4 $ 3 = 40 = 1, 67;
24 24
1 +1
M0 = 1; Me = ; R = 4 - 0 = 4.
2

We have the following formulas for median and mode of an interval


variation series:
cum
0, 5n - n Me -1
Me = xMe + k $
nMe

135
LECTURE 12

where xMe is the beginning of the median interval, k is the length


of partial interval, n is the set size, n Me
cum
- 1 is the cumulative frequency
of the interval preceding to the median interval, nMe is the frequency
of the median interval.
(nMo - n Mo - 1)
Mo = xMo + k $
(nMo - n Mo - 1) + (nMo - n Mo + 1)
where xMo is the beginning of the modal interval, k is the length of partial
interval, nMo is the frequency of the modal interval, nMo-1 is the frequency
of the pre-modal interval, and nMo+1 is the frequency of the post-modal
interval.
Example 2. Let the following interval series be given:
Intervals with length
Frequency ni
k = 1,6
4 – 5,6 5
5,6 – 7,2 17
7,2 – 8,8 9
8,8 – 10,4 15
10,4 – 12,0 10
12,0 – 13,6 1
13,6 – 15,2 3
Σ 60

We have:
0, 5 $ 60 - 22
Me = 7, 2 + 1, 6 $ = 8, 62;
9

136
Properties of dispersion

(17 - 5)
Mo = 5, 6 + 1, 6 $ = 6, 56.
(17 5) + (17 - 9)
-
Mode and median are used as a characteristic of average position in case
when the boundaries of a series are fuzzy or a series is not symmetrical.
Dispersion D of a variation series is the arithmetic mean of squares
of deviations of variants from their arithmetic mean:
m
/ (x - x )
i
2
ni m

D= i=1
= / (xi - x ) 2 wi .
n i=1

Mean square deviation σ of a variation series is the arithmetic value


of square root of its dispersion:
m
/ (x - x )
i
2
ni
i=1
v= .
n

Coefficient of variation: V = v $ 100% ( x ! 0) .


x
Coefficient of variation characterizes the relative value of the mean
square deviation and usually serves for comparing variance quantities
according to sample mean of two variation series: one of these series
has greater variance if its coefficient of variance is greater.

Properties of dispersion
1. If we add to (subtract from) all the results of observations the same
number C then dispersion is not changed.
2. If all the results of observations multiply on the same number C then
dispersion is increased in C2 times.
3. If all the frequencies of variants multiply on the same number then
dispersion is not changed.

137
LECTURE 12

The rule of addition of dispersions


If we combine several distributions in one “new” distribution,
the general dispersion of the new distribution is equal to the sum of
the intra-group dispersion and the inter-group dispersion:

j
x1 x2 … xm ∑
i
1 n11 n12 n1m n1
2 n21 n22 n2m n2
3 n31 n32 n3m n3
. … … … … .
. … … … … .
. … … … … .
k nk1 nk2 nkm nk
∑ N1 N2 Nm N

/ (x ij - x gen) 2 nij
/x 2
j Nj
i, j
Dgen = Dint ra + Dint er or Dgen = = - ( x gen) 2
N N
where nij is the frequency of the j-th variant of the i-th partial distribution
(j = 1, …, m; i = 1, 2, …, k);
xij is the j-th variant of the i-th partial distribution (j = 1, …, m; i = 1,
2, …, k);
ni is the size of the i-th partial distribution;
N j = / nij is the frequency of the j-th variant of the new distribution;
i
k

N = / ni is the size of the new distribution;


i=1

138
The rule of addition of dispersions

/n x ij i

is the arithmetic mean of the i-th partial distribution


j
xi =
ni
(i = 1, 2, …, k);

x gen =
/ x j N j is the arithmetic mean of the new distribution;
N
/x 2
ij nij
- ( x i) 2 is the dispersion of the i-th partial distribution;
j
Di =
ni

Dint ra =
/D n i i
is the intra-group dispersion;
N

Dint er =
/ (x - x i ) 2 ni
gen
is the inter-group dispersion;
N

Example. Let be given the following distribution consisting of two


groups:

The first group The second group


xi ni xi ni
2 1 3 2
4 7 8 3
5 2

Find the group dispersions, the intra-group dispersion, the inter-group


dispersion and the general dispersion of the distribution.
Solution: We have:
N1 = / ni = 10, N2 = / ni = 5, N = N1 + N2 = 10 + 5 = 15.
Find the group means:

139
LECTURE 12

x 1 = ^/ ni xih // ni = (1 $ 2 + 7 $ 4 + 2 $ 5) /10 = 4;
x 2 = (2 $ 3 + 3 $ 8) /5 = 6.
Find the required group dispersions:

D1 = ^/ ni (xi - x 1) 2h /N1 =
= (1 $ (2 - 4) 2 + 7 $ (4 - 4) 2 + 2 $ (5 - 4) 2) /10 = 0, 6;
D2 = (2 $ (3 - 6) 2 + 3 $ (8 - 6) 2) /5 = 6.

Find the intra-group dispersion:

Dint ra = (N1 D1 + N2 D2) /N = (10 $ 0, 6 + 5 $ 6) /15 = 12/5.

Find the general mean:

x gen =
/n x i i + + + +
= 1 $ 2 7 $ 4 2 $ 5 2 $ 3 3 $ 8 = 14 .
/n i 15 3
Find the required inter-group dispersion:

N1 $ ( x 1 - x gen) 2 + N2 $ ( x 2 - x gen) 2
Dint er = =
N
2 2
10 $ (4 - 14/3) + 5 $ (6 - 14/3)
= = 8.
15 9

Find the general dispersion:


Dgen = Dint ra + Dint er = 12/5 + 8/9 = 148/45.
Initial moment of the k-th order of a variation series is
m
/x n k
i i
i=1
Mk = .
n

140
The rule of addition of dispersions

m
/x n k
i i

In particular, M1 = i=1
= x.
n
Central moment mk of the k-th order of a variation series is
m
/ (x i - x ) k ni
i=1
mk = .
n

m
/ (x i - x ) 2 ni
Obviously, m1 = 0, m2 = i=1
= D.
n
When studying distributions other than normal, there is a need to
quantify this difference. For this purpose special characteristics are
entered, in particular, asymmetry and excess. These characteristics are
equal to zero for the normal distribution. Therefore, if for a studying
distribution the asymmetry and excess have small values, we can
assume the closeness of this distribution to normal. On the contrary,
great values of asymmetry and excess indicate a significant deviation
from the normal.
Coefficient of asymmetry of a variation series is
m

m
/ (x i - x ) 3 ni
i=1
as = 33 = .
v nv3

Excess (or coefficient of excess) of a variation series is


m
/ (x i - x ) 4 ni
m i=1
ek = 4 - 3 = - 3.
v 4
nv4

141
LECTURE 12

Consider Example 2. Construct the following table:

Groups of Frequency
Average
farms on (the number
value of 4

(xi - x ) 2 ni c i v m ni c v m ni
x-x
3
xi - x
amount of of farms in xini
interval
workers on the group)
(xi)
100 hectares ni
4 – 5,6 4,8 5 24 72,708 -18,954 29,554
5,61 – 7,2 6,4 17 108,8 83,28 -12,601 11,404
7,21 – 8,8 8 9 72 3,386 -0,142 0,036
8,81 – 10,4 9,6 15 144 14,603 0,985 0,397
10,41 – 12,0 11,2 10 112 66,908 11,832 12,514
12,01 – 13,6 12,8 1 12,8 17,528 5,017 8,588
13,61 – 15,2 14,4 3 43,2 100,457 39,74 94,03
Σ - 60 516,8 358,869 25,876 156,523

We have: x=
/x n
i i
=
516, 8
= 8, 613;
n 60
D=
/ (xi - x ) ni = 358, 869 = 5, 981;
2

n 60

v=
/ (xi - x ) 2 ni
= 5, 981 = 2, 446;
n
v 2, 446
V = $ 100% = $ 100% = 28, 4%.
x 8, 613
Thus, the average number of employees per 100 ha of farmland on
analyzing set of economies amounted to 8,61 persons. The density of
employees on average varied in the interval x ! v = 8, 61 ! 2, 45 i.e.
from 6,16 to 11,06 persons per 100 ha of farmland. This interval and
the coefficient of variation indicate that there are big differences in the
availability of economies by labor.

142
Glossary

m
/ (x - x ) n
i
3
i
25, 876
Coefficient of asymmetry: as = i=1
3 = = 0, 43.
nv 60
m
/ (x - x )
i
4
ni
156, 523
Excess: ek = i=1
4 -3 = - 3 =- 0, 39.
nv 60
The found value of coefficient of asymmetry (not sufficiently close to
zero) specifies that the distribution is not symmetrical. The excess is
also different from zero indicating a possible difference from normal
distribution.

Glossary
gathering – сбор; gathering data – сбор данных;
nominating – выдвижение
ordering, systematization – систематизация; processing – обработка
data processing – обработка данных; observation – наблюдение
revealing – выявление; attribute – признак;
sampling – выборочный метод
sample – выборка; census – перепись
parent population – генеральная совокупность; essence – сущность
judgment – суждение; destruction – разрушение
inevitable – неизбежный; representative – представительный
properly – собственно; occurring – происходящий;
wheat – пшеница
farm – хозяйство; shift – смена; to rank – ранжировать
frequency – частота; cumulative frequency – накопленная частота
inexpedient – нецелесообразный

143
LECTURE 12

arithmetic mean – среднее арифметическое;


disparate – несоизмеримый
variance – рассеяние; excess – эксцесс
farmland – сельскохозяйственные угодья
economy – хозяйство; labor – рабочая сила

Exercises for Seminar 12


12.1. The sample is given as a distribution of frequencies:

xi 2 5 7
ni 1 3 6

Find the distribution of relative frequencies.


12.2. Find the empirical function of the following distribution:

xi 2 5 7 8
ni 1 3 2 4

12.3. Construct the polygon of frequencies for the following distribution:

xi 2 3 5 6
ni 10 15 5 20

12.4. Construct the polygon of relative frequencies for the following


distribution:

xi 2 4 5 7 10
ni 0,15 0,2 0,1 0,1 0,45

144
Exercises for Seminar 12

12.5. Construct the histogram of frequencies for the following


distribution:
The number of interval The partial interval Frequency
i xi – xi + 1 ni
1 1-5 10
2 5-9 20
3 9-13 50
4 13-17 12
5 17-21 8

12.6. Construct the histogram of relative frequencies for the following


distribution:
The number of interval The partial interval Frequency
i xi – xi + 1 ni
1 10-15 2
2 15-20 4
3 20-25 8
4 25-30 4
5 30-35 2
12.7. Find the group means of a population consisting of two groups:
the first group … xi 0,1 0,4 0,6
ni 3 2 3

the second group xi 0,1 0,3 0,4


ni 10 4 6

145
LECTURE 12

12.8. Find the intra-group, inter-group and general dispersions of


a population consisting of three groups:
the first group … xi 1 2 8
ni 30 15 5

the second group … xi 1 6


ni 10 15

the third group … xi 3 8 2


ni 20 5 3

12.9. There are 100 workers at an enterprise according to the list who
have the following categories:
1, 5, 2, 4, 3, 4, 6, 4, 5, 1, 2, 2, 3, 4, 5, 3, 4, 5, 2, 1, 4, 5, 5, 4, 3, 4, 6, 1, 2,
4, 4, 3, 5, 6, 4, 3, 3, 1, 3, 4, 3, 1, 2, 4, 4, 5, 6, 1, 3, 4, 5, 3, 4, 4, 3, 2, 6, 1,
2, 4, 5, 3, 3, 2, 3, 6, 4, 3, 4, 5, 4, 3, 3, 2, 6, 3, 3, 4, 5, 4, 4, 3, 3, 2, 1, 2, 1,
6, 5, 4, 3, 2, 3, 4, 4, 3, 5, 6, 1, 5.
Compose the series of distribution of workers on categories. Find
cumulative and relative frequencies. Determine the average category
of a worker, the modal and median category, the dispersion and the mean
square deviation.
12.10. There are the following conditional data on size of quotas in
millions dollars of 100 countries – members of the International
currency fund:

353 326 344 324 339 332 324 344 349 352
348 316 329 354 358 302 325 324 351 333
341 312 331 351 304 345 332 382 342 351
396 341 353 318 325 354 338 321 398 359

146
Exercises for Homework 12

376 355 382 342 374 354 358 332 368 343
344 376 324 339 372 366 381 334 369 332
371 312 334 361 304 362 354 366 378 348
352 362 356 364 372 342 344 346 353 334
336 364 352 348 347 368 329 335 363 312
378 342 354 363 361 366 354 364 348 351
It is required:
a) Compose the interval variation series taking the beginning of the first
interval equal 300, and the width of each interval equal 10;
b) Construct the histogram and the polygon of relative frequencies
of distribution;
c) Find the mode and the median of the variation series;
d) Find empirical function of distribution of the variation series and
construct its graph.
12.11. Let the following distribution of employers by an experience
of their work be given:
Distribution of employers on the experience of work

The experience of
Up to 1 1-5 5-10 10-20 20-40 Total
work, years
Number of
8 12 16 14 10 60
employers

Find the average experience of work, the mean square deviation and
the coefficient of variation (experience – стаж).

Exercises for Homework 12


12.12. The sample is given as a distribution of frequencies:

147
LECTURE 12

xi 4 7 8 12
ni 5 2 3 10
Find the distribution of relative frequencies.
12.13. Find the empirical function of the following distribution:
xi 4 7 8
ni 5 2 3

12.14. Construct the polygon of frequencies for the following


distribution:
xi 15 20 25 30 35
ni 10 15 30 20 25

12.15. Construct the polygon of relative frequencies for the following


distribution:
xi 1 4 5 8 9
ni 0,15 0,25 0,3 0,2 0,1

12.16. Construct the histogram of frequencies for the following


distribution:

The number of The partial interval Frequency


interval i xi – xi + 1 ni
1 2-7 5
2 7-12 10
3 12-17 25
4 17-22 6
5 22-27 4

148
Exercises for Homework 12

12.17. Construct the histogram of relative frequencies for the following


distribution:

The number of The partial interval Frequency


interval xi – xi + 1 ni
i
1 2-5 6
2 5-8 10
3 8-11 4
4 11-14 5

12.18. Find the intra-group, inter-group and general dispersions of


a population consisting of two groups:
the first group … xi 2 7
ni 6 4

the second group… xi 2 7


ni 2 8

12.19. By oral interrogation the quality of production released by a firm


and sold in a shop of this firm was studied. Visitors were estimating
the quality by a ten-mark scale. The summary data have been received.
Mark estimation of production of the enterprise

Estimation of quality 1-2 3-4 5-6 7-8 9-10


of production, point
Number of cases 3 8 36 89 45

149
LECTURE 12

Determine the average mark of quality of production, the mean square


deviation, the coefficient of variation, the parameters of asymmetry
and excess (interrogation – опрос; to release – выпускать; visitor –
посетитель).
12.20. Carry out the analysis of the data of annual levels of profit
of three companies:
«Cherry «Lemon «Orange
Year
Computers» Motors» Electronics»
1983 14,2 -6,2 37,5
1984 12,3 13,3 -10,6
1985 -16,2 -8,4 40,3
1986 15,4 27,3 5,4
1987 17,2 28,2 6,2
1988 10,3 14,5 10,2
1989 -6,3 -2,4 13,8
1990 -7,8 -3,1 11,5
1991 3,4 15,6 -6,2
1992 12,2 18,2 27,5

Find the average value and the mean square deviation of the profit for
each of the companies. Compare the received results of their activity for
10 years. Which of the companies, in your opinion, is the activity more
successful for?
12.21. There are the following data on a number of industrial
subdivisions on each of 100 agricultural enterprises:
2, 4, 5, 3, 4, 6, 7, 4, 5, 3, 3, 4, 2, 6, 5, 4, 7, 2, 3, 4, 4, 5, 4, 3, 4, 6, 6, 5, 2,
3, 4, 3, 5, 6, 7, 2, 4, 3, 4, 5, 4, 6, 7, 2, 5, 3, 5, 4, 3, 7, 2, 4, 3, 4, 5, 4, 3, 2,
6, 7, 6, 4, 3, 2, 3, 4, 5, 4, 3, 5, 4, 3, 2, 6, 4, 5, 7, 5, 4, 3, 4, 5, 7, 4, 3, 4, 5,
6, 5, 3, 4, 2, 2, 4, 3, 7, 5, 6, 4, 5.
Compose the series of distribution of the agricultural enterprises by
number of industrial subdivisions for one economy. Find cumulative

150
Exercises for Homework 12

and relative frequencies. Determine the average number of industrial


subdivisions for one economy, the modal and median values
of the number of subdivisions, the dispersion and the mean square
deviation (economy – хозяйство).
12.22. The administration of a supermarket is interested in the optimal
level of stocks of products in a trading hall, and also the monthly average
volume of purchases of the goods which are not subjects of daily
consumption in a family (for example such as soda). For finding-out
of this question the manager of a supermarket within January was
registering the frequency of purchases of 100-gramme packages with
soda and has collected the following data xi:
4, 4, 9, 3, 3, 1, 2, 0, 4, 2, 3, 5, 7, 10, 6, 5, 7, 3, 2, 9, 8, 1, 4, 6, 5, 4, 2, 1,
0, 8
Construct the variation series, determine its numerical characteristics.
Which recommendations would you give to the administration of the
supermarket (stock – запас; to find out – выяснить)?

151
LECTURE 13

LECTURE 13

Statistical Estimations of Parameters of Distribution


Mathematical theory of sampling is based on properly random sample.
Arithmetic means of distribution of an attribute in a parent and sample
populations are parent and sample means respectively, and dispersions
of these distributions – parent and sample dispersions.
The major problem of sampling is estimating parameters (characteristics)
of a parent population by sample data.
Let a quantitative attribute of a parent population be required to
study. Assume we have established that the attribute has a certain
distribution. The task of estimation of parameters of the distribution
arises naturally. For example if it is known that a studied attribute is
normally distributed, it is necessary to estimate (to find approximately)
mathematical expectation and mean square deviation since these
parameters completely determine a normal distribution. And if there
is reason to consider that the attribute has Poisson distribution, it is
necessary to estimate the parameter λ which determines this distribution.
A researcher usually has only data of a sample, for example the
values of a quantitative attribute x1, x2, …, xn received in result of n
observations (hereinafter we assume that observations are independent).
An estimated parameter is expressed by these data. Considering
x1, x2, …, xn as independent random variables X1, X2, …, Xn we can
say that to find statistical estimation of an unknown parameter of a
theoretical distribution is to find function of observed random variables
which gives an approximate value of the estimated parameter.
Formulate the problem of estimation of parameters in a general form:
Let distribution of an attribute X of a parent population be given by
function of probabilities φ(xi, θ) = P(X = xi) (for a discrete random
variable X) or probability density φ(x, θ) (for a continuous random
variable X) which contains an unknown parameter θ. For example, it is
the parameter λ for Poisson distribution or the parameters a and σ2 for
the normal law of distribution and etc.

152
Statistical Estimations of Parameters of Distribution

One tries to judge on a parameter θ by a sample consisting of values


(variants) x1, x2, …, xn. These values can consider as partial values
(realizations) of n independent random variables X1, X2, …, Xn each of
which has the same law of distribution with the random variable X.
An estimator θ* of a parameter θ is a function of results of observations
over a random variable X by means of which one judges on value of
the parameter θ :
θ* = θ*(X1, X2, ..., Xn).
A function of results of observations (i.e. a sample function) is called
statistic. It can be said that an estimator θ* of a parameter θ is statistic
which is close in certain sense to true value of θ.
In order to statistical estimators gave “good” approximations of
estimated parameters it is necessary that these estimators satisfy the
certain requirements.
Let θ* be a statistical estimator of an unknown parameter θ of a
theoretical distribution. Assume that an estimator has been found on
a sample of size n. Repeat the experience, i.e. extract another sample
of the same size from the parent population and find an estimator θ*2
on its data. Repeating the experience many times, we obtain numbers
θ*1, θ*2, ..., θ*k that are in general different between them. Thus, the
estimator θ* can be considered as a random variable, and the numbers
θ*1, θ*2, ..., θ*k as its possible values.
Suppose that the estimator θ* provides an approximate value θ with
a surplus. Then each number θ found on the data of samples is greater
than the true value θ. It is clear that the mathematical expectation (the
mean value) of the random variable θ* is greater than θ, i.e. M(θ*) > θ.
Obviously, if θ* provides an estimation with a lack then M(θ*) < θ.
Thus, an using a statistical estimator of which mathematical expectation
is not equal to an estimated parameter provides systematical mistakes
(In theory of mistakes of measuring non-random mistakes distorting
results of measuring to one certain direction are called systematical).
Therefore it is naturally to require the following: M(θ*) = θ.
An estimator θ* of a parameter θ is unbiased if its mathematical

153
LECTURE 13

expectation is equal to the estimated parameter, i.e. M(θ*) = θ.


Otherwise it is biased.
The requirement of unbiasedness guarantees an absence of regular
mistakes at estimating. Also this requirement is important for a small
number of observations (experiences).
If for a finite volume of sample n, M(θ*) ≠ θ, i.e. the bias (displacement)
of an estimator b(θ*) = M(θ*) – θ ≠ 0, but lim b (i)) = 0 then such an
n"3
estimator, θ* is said to be asymptotically unbiased.
However, it would be a mistake to believe that an unbiased estimator
always provides a good approximation of an estimated parameter.
Indeed, the possible values of θ* can be strongly dispersed around of its
average value, i.e. dispersion D(θ*) can be big. In this case an estimator
θ1* can be too distant from the average value θ*, ͞ consequently from
the estimated parameter θ; therefore if we accept θ1* as an approximate
value of θ then we obtain a big mistake. If we require that dispersion
of θ* be small then the possibility to obtain a big mistake will be
eliminated. Therefore, the requirement of efficiency is added to a
statistical estimator.
A statistical estimator θ* of a parameter θ is efficient if it has the least
dispersion among all possible estimators of the parameter θ calculated
on samples of the same volume n.
At considering samples of a big size (n is very great) the requirement of
consistency is presented to statistical estimators.
An estimator θ* of a parameter θ is consistent if it satisfies the law
of large numbers, i.e. it converges on probability to the estimated
parameter:

lim P (| i) - i | < f) = 1.
n"3

In case of use of consistent estimators an increasing of sample volume


is justified since for this significant mistakes become improbable at
estimating. Therefore, only consistent estimators have a practical sense.

154
Pointwise Estimators of Mathematical Expectation and Dispersion

It is desirable as statistical estimators of parameters of a parent


population to use the estimators satisfying simultaneously requirements
of unbiasedness, consistency and efficiency. However to reach it is not
always possible. One can appear that for simplicity of calculations
it is expedient to use insignificantly biased estimators or estimators
possessing the greater dispersion in comparison with effective
estimators, etc.

Pointwise Estimators of Mathematical


Expectation and Dispersion
Let a random variable X with mathematical expectation a = M(X) and
dispersion D(X) be studied. Statistic used as an approximate value of
unknown parameter of parent population is its pointwise estimator, i.e.
a pointwise estimator of characteristic of parent population is a number
defined by a sample.
Let x1, x2, …, xn be a sample obtained in result of conducting n independent
observations for the random variable X. To underline a random character
of quantities x1, x2, …, xn rewrite them as X1, X2, …, Xn, i.e. by Xi we
mean the value of the random variable X in the i-th experience. The
random variables X1, X2, …, Xn can be considered as n independent
“exemplars” of X. Therefore M(Xi) = M(X) = a, D(Xi) = D(X).
Theorem. Let X1, X2, …, Xn be a sample from a parent population and
M(Xi) = M(X) = a, D(Xi) = D(X) (i =1, …, n). Then the sample mean
m
/x n i i

xs = i=1 is an unbiased and consistent estimator of mathematical


n
expectation (parent mean).
Remark 1. If original variants xi are large numbers then for simplification
of counting it is expedient to subtract the same number C from each
variant, i.e. go to conditional variants ui = xi – C (it is profitable to take
as C the number close to the sample mean; because the sample mean is
unknown, the number C is picked approximately).

155
LECTURE 13

Then 1) x s = C + ^/ ni uih /n.


2) Dispersion is not changed:

Ds (x) = Ds (u) = u2 - [ u ] 2 =
/n u i
2
i
-;
/n u E .
i i
2

n n

Remark 2. If original variants are decimal fractions with k decimal


sings after comma, then in order to avoid any actions with fractions,
we multiply the original variants on a constant number C = 10k, i.e. go
to conditional variants ui = Cxi. For this the dispersion is increased in
C2 times. Therefore at finding the dispersion of conditional variants it
needs divide this dispersion on C2:
Ds (x) = Ds (u) / C 2.

Theorem. The sample dispersion


m
/ (x - x )
i s
2
ni
i=1
Ds =
n
is a biased estimator of the parent dispersion Dp since
n-1
M (Ds) = Dp .
n

Therefore we revise the sample dispersion by multiplying on n/(n – 1)


and obtaining the formula: s2 = n $ Ds .
n-1
Corollary. The revised sample dispersion

s2 =
n
$D =
/ n (x - x )
i i s
2

n-1 s n-1
is an unbiased estimator of the parent dispersion.
It is more convenient the following formula:

156
Pointwise Estimators of Mathematical Expectation and Dispersion

/n x - 6/ ni xi @ /n .
2 2
2 i i
s = x
n-1
In conditional variants it has the following form:
/n u - 6/ ni ui @ /n ,
2 2
i i
su2 =
n-1
and for this if ui = xi – C then s 2x = su2 if ui = Cxi then s 2x = su2 /C2
Example. A sample of size 50 is extracted from a parent population:
variant xi 2 5 7 10
frequency ni 16 12 8 14
Find an unbiased estimator of the parent mean.
Solution: An unbiased estimator of the parent mean is the sample mean:
4
/x n i i
i=1
xs = = (16 $ 2 + 12 $ 5 + 8 $ 7 + 14 $ 10) /50 = 5, 76
n

Example. Find the sample mean for the following distribution of sample
of size n = 10:
xi 1250 1270 1280
ni 2 5 3

Solution: The original variants are large numbers; therefore we go to


conditional variants:
ui = xi – 1270
Thus, we obtain the following distribution of conditional variants:
ui – 20 0 10
ni 2 5 3

157
LECTURE 13

Find the required sample mean:


3
/u n i i
2 $ (- 20) + 5 $ 0 + 3 $ 10
xs = C + i=1
= 1270 + = 1270 - 1 = 1269 .
n 10

Example. For a sample of size n = 41 a biased estimator DS = 3


of the parent dispersion has been found. Find an unbiased estimator
of dispersion of parent population.

Solution: s2 = n 41
$D = $ 3 = 3, 075
n - 1 s 40
Example. As a result of five measurements of length of a rod by one
device (without systematical mistakes) the following results have been
obtained (in mm): 92; 94; 103; 105; 106. Find: a) sample mean of length
of the rod; b) sample and revised dispersions of mistakes of the device
(rod – стержень).
Solution: a) Find sample mean:
b) Find sample dispersion:

Ds =
/ (x - x )
i s
2

= [(92 - 100) 2 + (94 - 100) 2 + (103 - 100) 2 +


n
+ (105 - 100) 2 + (106 - 100) 2] /5 = 34

n
Find the revised dispersion: s2 = 5
$ D = $ 34 = 42, 5
n-1 s 4
Example. Find sample dispersion for the following distribution of
sample of size n = 10:
xi 0,01 0,04 0,08
ni 5 3 2
Solution: In order to avoid any actions with fractions go to conditional
variants ui = 100xi. Thus, we obtain the following distribution:

158
Pointwise Estimators of Mathematical Expectation and Dispersion

ui 1 4 8
ni 5 3 2
Find the sample dispersion of conditional variants:

Ds (u) =
/n u
i
2
i
-;=
/n u E
i i
2

n n
2
= 5 $ 1 3 $ 4 2 $ 8 - c 5 $ 1 3 $ 4 2 $ 8 m = 7, 21
2 2 2
+ + + +
10 10

Find the required sample dispersion of the original variants:

Ds (x) = Ds (u) / 100 2 = 7,21 /1 0000 = 0.0007

Example. Find the revised sample dispersion for the following


distribution of sample of size n = 10:
xi 0,01 0,05 0,09
ni 2 3 5
Solution: In order to avoid any actions with fractions go to conditional
variants ui = 100xi. Thus, we obtain the following distribution:
ui 1 5 9
ni 2 3 5
Find the revised sample dispersion of conditional variants:
- 6/ ni ui @ /n .
/n u 2 2
i i
su2 =
n-1
Substituting the data of the example in this formula we get: su2 = 10,844
Find the required revised dispersion of the original variants:
sx2 = su2 / 1002 = 10,844/10000 ≈ 0,0010844.

159
LECTURE 13

Glossary
sampling – выборочный метод; sample – выборка
census – перепись; parent population – генеральная совокупность
essence – сущность; judgment – суждение; inevitable – неизбежный
representative – представительный; estimator – оценка
surplus – избыток; lack – недостаток; to distort – искажать
unbiased – несмещенная; unbiasedness – несмещенность
consistent – состоятельная

Exercises for Seminar 13


13.1 A sample of size 60 is extracted from a parent population:
variant xi 1 3 6 26
frequency ni 8 40 10 2
Find an unbiased estimator of the parent mean.
The answer: 4
13.2 Find the sample mean for the following distribution of sample
of size n = 20:
xi 2560 2600 2620 2650 2700
ni 2 3 10 4 1
Direction: Go to conditional variants.
The answer: 2621.
13.3. For a sample of size n = 51 a biased estimator Ds = 5 of the parent
dispersion has been found. Find an unbiased estimator of dispersion
of the parent population.
The answer: 5,1.

160
Exercises for Seminar 13

13.4. As a result of four measurements of some physical quantity


by one device (without systematical mistakes) the following results
have been obtained: 8; 9; 11; 12. Find: a) the sample mean of results
of measurements; b) the sample and revised dispersions of mistakes
of the device.
The answer: a) xS = 10; b) DS = 2,5; s2 = 10/3.
13.5. The results of measuring the height (in centimeters) of randomly
selected 100 students have been obtained:

154- 158- 162- 166- 170- 174- 178-


Height
158 162 166 170 174 178 182
The number
10 14 26 28 12 8 2
of students

Find the sample mean and sample dispersion of height of these students.
Direction: Find the middles of intervals and take them as variants.
The answer: ͞xS = 166; DS = 33,44.
13.6. Find the sample dispersion for the following distribution
of sample of size n = 10:
xi 186 192 194
ni 2 5 3
The answer: DS = 8,04.
13.7. Find the sample dispersion for the following distribution
of sample of size n = 50:
xi 0,1 0,5 0,6 0,8
ni 5 15 20 10
The answer: DS = 0,0344.
13.8. Find the revised sample dispersion for the following distribution
of sample of size n = 20:

161
LECTURE 13

xi 0,1 0,5 0,7 0,9


ni 6 12 1 1
The answer: s = 0,0525.
2

Exercises for Homework 13


13.9. Find the sample dispersion for the following distribution of sample
of size n = 100:
xi 340 360 375 380
ni 20 50 18 12
The answer: DS = 167,29.
13.10. Find the sample dispersion for the following distribution
of sample of size n = 50:
xi 18,4 18,9 19,3 19,6
ni 5 10 20 15
Direction: Go to the conditional variants ui = 10xi – 195.
The answer: DS = 0,1336.
13.11. Find the revised sample dispersion for the following distribution
of sample of size n = 10:
xi 23,5 26,1 28,2 30,4
ni 2 3 4 1
The answer: s2 = 4,89.
13.12. Find the revised sample dispersion for the following distribution
of sample of size n = 100:
xi 1250 1275 1280 1300
ni 20 25 50 5
The answer: s = 168,88.
2

162
Methods of Finding of Estimations

LECTURE 14

Methods of Finding of Estimations


Method of moments.
According to the method of moments offered by K. Pearson for finding
pointwise estimators of unknown parameters of a given distribution is
to equate a certain amount of the sampling moments (initial Mk either
central , or those and others) to the corresponding theoretical moments
of distribution (νk or μk) of a random variable of X.
Remind that the sampling moments Mk and mk are determined by
the formulas:
m m
/x n k
i i / (x i - x ) k ni
i=1 i=1
Mk = , mk =
n n
and the corresponding them theoretical moments –

νk = M(Xk), μk = M[X – M(X))2].


n n

ok = / xik pi, nk = / (xi - a) k pi


i=1 i=1

for a discrete random variable with the function of probabilities


pi = φ (xi, θ);
+3 +3

ok = # x { (x, i) dx,
k
nk = # (x - a) { (x, i) dx
k

-3 -3

for a continuous random variable with density of probabilities φ(x, θ)),


where a = M(X).
If a distribution is determined by only one parameter then for its finding
one theoretical moment is equated to one empirical moment of the same
order. For example, we can equate an initial theoretical moment of the

163
LECTURE 14

first order to the initial empirical moment of the first order: ν1 = M1.
Taking in account that ν1 = M(X) and M1 = ͞xS, we obtain: M(X) = ͞xS. (*)
Mathematical expectation is a function of an unknown parameter of
the given distribution, therefore solving the equation (*) with respect to
the unknown parameter we obtain its pointwise estimator.
If a distribution is determined by two parameters then we equate two
theoretical moments to the corresponding empirical moments of the
same order. For example, we can equate the initial theoretical moment
of the first order to the initial empirical moment of the first order and the
central theoretical moment of the second order to the central empirical
moment of the second order: ν1 = M1, μ2 = m2.
Taking in account that ν1 = M(X), M1 = x͞ S , μ2 = D(X), m2 = DS, we have:

)
M) X) = x s,
(**)
D (X) = Ds .
The left parts of these equalities are functions of unknown parameters,
therefore solving the system (**) with respect to the unknown
parameters we obtain their pointwise estimators.
Of course, to calculate sample mean ͞xS and sample dispersion DS we
need a sample x1, x2, …, xn.
Example. A random variable X is distributed under the law of Poisson:
Pm (xi) = mx e- m /xi !
i

where m is the number of trials made in one experience; xi is the number


of appearances of an event in the i-th experience. Find by method
of moments on a sample x1, x2, …, xn the pointwise estimator of the
unknown parameter λ determining the Poisson distribution.
Solution: It is required to estimate one parameter; therefore it is
sufficiently to have one equation with respect to this parameter. Equate
the initial theoretical moment of the first order ν1 to the initial empirical
moment of the first order M1: ν1 = M1.
Taking in account that ν1 = M(X) and M1 = ͞xS we obtain M(X) = ͞xS.

164
Method of maximal (the greatest) plausibility

Also taking in account that the mathematical expectation of the Poisson


distribution is equal to the parameter λ of this distribution, we finally
have: λ = x͞ S.
Thus, the pointwise estimator of the parameter λ of the Poisson
distribution is the sample mean:

λ* = ͞xS.

Estimators of the method of moments are usually consistent; however


they are not “best” on efficiency, their efficiencies e(θ*) are often less
than 1. Nevertheless, the method of moments is frequently used in
practice since it results in rather simple calculations.

Method of maximal (the greatest) plausibility.


The basic method of obtaining estimations of parameters of parent
population on sample data is the method of maximal plausibility
offered by R. Fisher.
The method of maximal plausibility of pointwise estimator of unknown
parameters of a given distribution is reduced to finding maximum of
a function of one or several estimated parameters.
A. Discrete random variables. Let X be a discrete random variable
which in result of n experiences has taken on possible values x1, x2, …, xn.
Assume that the form of law of distribution is given, but a parameter
θ determining this law is unknown; it is required to find its pointwise
estimator θ* = θ*(x1, x2, …, xn)
Denote the probability that in result of trial the variable X take on
a value xi by p(xi; θ).
The function of plausibility of a discrete random variable X is called
the function of argument θ:
L(x1, x2, …, xn;θ) = p(x1, θ) · p(x2, θ) · ... · p(xi, θ) · ... · p(xn, θ).
or
n

L (x1, x2, ..., xi, ..., xn; i) = % p (xi, i)


i=1

165
LECTURE 14

An estimator of the greatest plausibility of parameter θ is called such


its value θ* at which the function of plausibility reaches a maximum.
Functions L and ln L reach a maximum for the same value θ, therefore
instead of finding a maximum of the function L we find (it is frequently
convenient) a maximum of the function ln L.
The logarithmic function of plausibility is called the function ln L.
A point of maximum of a function ln L of argument θ can be found, for
example, as follows:
1. Find the derivative d ln L
di
2. Equate the derivative to zero and find the critical point θ* – the root
of the obtained equation (it is called an equation of plausibility).
2
3. Find the second derivative d ln2L ; if the second derivative for θ = θ*
di
is negative then θ* is a point of maximum.

The found point of maximum θ* is taken as an estimator of the greatest


plausibility of the parameter θ.
B. Continuous random variables. Let X be a continuous random
variable which in result of n trials has taken on the values x1, x2, …, xn.
Assume that the form of density of distribution – the function f(x) – is
given, but the parameter θ determining this function is unknown.
The function of plausibility of a continuous random variable X is called
the function of argument θ:
L(x1, x2, …, xi ..., xn;θ) = f(x1, θ) · f(x2, θ) · ... · f(xi, θ) · ... · f(xn, θ).
An estimator of the greatest plausibility of an unknown parameter of
distribution of a continuous random variable is found analogously as in
case of a discrete random variable.
If density of distribution f(x) of a continuous random variable is
determined by two unknown parameters θ1 and θ2 then the function
of plausibility is a function of two independent arguments θ1 and θ2:
L(x1, x2, …, xi ..., xn;θ1, θ2) = f(x1, θ1, θ2) · f(x2, θ1, θ2) · ... · f(xn, θ1, θ2)

166
Method of maximal (the greatest) plausibility

Further the logarithmic function of plausibility is found, and for finding


its maximum the following system is composed and solved:

2 ln L =
* 22lni1L
0,

= 0.
2i2

Example. Find by method of maximal plausibility a pointwise estimator


of unknown parameter p (probability of appearance of an event in one
trial) of a binomial distribution:

Pm (xi) = C mx p x (1 - p) m - x
i i i

where xi is the number of appearances of an event in the i-th experience


(i = 1, 2, …, n), m is the number of trials in one experience, n is
the number of experiences.
Solution: Compose the function of plausibility:

L = p(x1; θ) · p(x2; θ) · ... · p(xn; θ).

Taking in account that θ = p and Pm (X = xi) = C mx p x (1 - p) m - x we i i i

obtain:

L = 6C mx p x (1 - p) m - x @ $ 6C mx p x (1 - p) m - x @ $ ... $ 6C mx p x (1 - p) m - x @
1 1 1 2 2 2 n n n

or

L = ^C mx C mx ...C mx h $ p x + x + ... + x $ (1 - p) nm - (x + x + ... + x )


1 2 n 1 2 n 1 2 n

Write the logarithmic function of plausibility:

ln L = ln 6C mx C mx ...C mx @ + c/ xi m ln p + (nm - / xi) ln (1 - p)


n n
1 2 n

i=1 i=1

167
LECTURE 14

Find the first derivative on p:


d ln L = /x i
- (nm - / xi) 1
dp p 1-p
Equating the first derivative to zero and solving the obtained equation,
we find the critical point:

p = ^/ xih / (nm)

Find the second derivative on p:

d2 ln L =- / xi - nm - / xi
dp2 p2 (1 - p) 2
It is easily to see that for p = ^/ xih / (nm) the second derivative is
negative; consequently this point is a point of maximum, and we take it
as an estimator of the greatest plausibility of unknown probability p of
binomial distribution:

p * = ^/ xih / (nm)

Obviously if xi appearances of the event were observed in ni experiences


then

p * = ^/ ni xih / (nm)

Example. Find by method of maximal plausibility on a sample


x1, x2, …, xn the pointwise estimator of unknown parameter λ of
an exponential distribution of which density is f (x) = me- mx (x $ 0)
Solution: Compose the function of plausibility

L = f(x1; θ) · f(x2; θ) · ... · f(xn; θ).

168
Method of maximal (the greatest) plausibility

Taking in account that θ = λ and consequently f(x; θ) = f(x; λ) = λe-λx we


have:

L = (me- mx ) $ (me- mx ) $ ... $ (me- mx ) = mn $ e- m/ x


1 2 n i

Find the logarithmic function of plausibility:

ln L = n ln m - m/ xi

Find the first derivative on λ:

n/m - / xi
d ln L =
dm
Write the equation of plausibility by equating the first derivative to zero:

n/m - / xi = 0

Find the critical point by solving the obtained equation with respect to
λ:
1
m= = 1/ x s
(/ xi) /n

Find the second derivative on λ:


d2 ln L = -
( n) /m2
dm2
It is easily to see that for m = 1/x s the second derivative is negative;
consequently this point is point of maximum, and thus we accept
the quantity which is inverse to sample mean as the estimator
of maximal plausibility: m* = 1/x s
Importance of the method of maximal plausibility is connected to its
optimal properties. So, if for parameter θ there is an effective estimator
θ*e then an estimator of maximal plausibility is unique and equal to θ*e.

169
LECTURE 14

Moreover, under enough general conditions the estimators of maximal


plausibility are consistent, asymptotically unbiased, and asymptotically
effective and have asymptotically normal distribution.
The basic lack of the method of maximal plausibility is difficulty
of calculation of estimators connected to solving the equations
of plausibility, more often nonlinear. Essentially as well that for
construction of estimators of maximal plausibility and providing
their “good” properties the exact knowledge such as the analyzed law
of distribution φ(x,θ) is necessary, that it appears practically unreal in
many cases.

Exercises for Seminar 14


14.1 A random variable X (the number of weed seeds in a trial of grain)
is distributed under the Poisson law. We have the following distribution
of weed seeds in n = 1000 trials of grain (the number xi of weeds in one
trial is indicated in the first raw; the frequency ni – the number of trials
containing xi weed seeds – in the second raw):
xi 0 1 2 3 4 5 6
ni 405 366 175 40 8 4 2
Find by method of moments the pointwise estimator of unknown
parameter of the Poisson distribution (weed – сорняк; seed – семя;
trial – проба; grain – зерно).
The answer: 0,9.
14.2 Find by method of moments on a sample x1, x2, …, xn the
pointwise estimator of the parameter p of the binomial distribution
Pm (xi) = C mx p x (1 - p) m - x , where xi is the number of appearances of an
i i i

event in the i-th experience (i = 1, 2, …, n), m is the number of trials in


one experience.
Direction: 1) Take in account that M(X) = mp. 2) Equate the initial
theoretical moment of the first order to the initial empirical moment
of the first order.

170
Exercises for Seminar 14

The answer: p* =
/x i

nm
14.3 A random variable X (the number of appearances of the event A in
m independent trials) is subordinated to a binomial law with unknown
parameter p. We have the following empirical distribution of number
of appearances of the event in 10 experiences on 5 trials each (the
number xi of appearances of the event A in one experience is indicated
in the first raw; the frequency ni – the number of experiences in which xi
appearances of the event A were observed – in the second raw):
xi 0 1 2 3 4
ni 5 2 1 1 1
Find by method of moments the pointwise estimator of unknown
parameter p of the binomial distribution.
Direction: Use the solution of Exercise 14.2.
The answer: 0,22.
14.4 Find by method of moments the pointwise estimator of parameter
p (probability) of a geometrical distribution P (X = xi) = (1 - p) x $ p
i-1

where xi is the number of trials made before an appearance of an event;


p is probability of appearance of the event in one trial.
Direction: Take in account that M(X) = 1 / p.
The answer: p* = 1
xs
14.5 Find by method of moments on a sample x1, x2, …, xn the
pointwise estimator of parameter p of a geometrical distribution
P (X = xi) = (1 - p) x $ p if for four experiences an event was appeared
i-1

after two, four, six and eight trials respectively.


The answer: 0,2.
14.6 A random variable X (the number of appearances of the event A in
m independent trials) is subordinated to a binomial law with unknown
parameter p. We have the following empirical distribution of number of

171
LECTURE 14

appearances of the event A in 1000 trials (the number xi of appearances


of the event A in one experience of m = 10 trials is indicated in the
first row; the frequency ni – the number of experiences in which xi
appearances of the event A were observed – in the second row):
xi 0 1 2 3 4 5 6 7
ni 2 3 10 22 26 20 12 5
Find by method of the greatest plausibility the pointwise estimator of
unknown parameter p of the binomial distribution.
The answer: 0,4.
14.7 A random variable X (the number of appearances of an event A in
m independent trials) is subordinated to a law of Poisson distribution
with unknown parameter λ:

Pm (X = xi) = mx $ e- m /xi !
i

where m is the number of trials in one experience, xi is the number of


appearances of the event in the i-th experience (i = 1, 2, …, n).
Find by the method of greatest plausibility on a sample x1, x2, …, xn the
pointwise estimator of unknown parameter λ of Poisson distribution.
The answer: m* = x s
14.8. A random variable X (the number of damaged glass items in one
container) is distributed under a Poisson law with unknown parameter
λ. We have the following empirical distribution of number of damaged
items in 500 containers (the number xi of damaged items in one container
is indicated in the first raw; the frequency ni – the number of containers
containing xi damaged items – in the second raw):
xi 0 1 2 3 4 5 6 7
ni 199 169 87 31 9 3 1 1
Find by method of the greatest plausibility the pointwise estimator of
unknown parameter λ of the Poisson distribution.

172
Exercises for Seminar 14

Direction: Use Exercise 14.7.


The answer: 1.
14.9. Find by the method of maximal plausibility on a sample x1, x2,
…, xn the pointwise estimators of parameters a and σ of a normal
distribution of which density is
1 2 2
f (x) = e-(x - a) /(2v )
v 2r
Direction: Compose and solve the following system:
2 ln L = 2 ln L =
0, 0
2a 2v
The answer: a* = x s, v* = Ds
14.10. Find by the method of greatest plausibility on a sample
x1, x2, …, xn the pointwise estimator of parameter β of gamma distribution
(parameter α is known) of which density is
1
f (x) = x a e-x/b (a > - 1, b > 0, x $ 0)
ba + 1 C (a + 1)

The answer: b* = xs
a+1
14.11. The device consists of elements of which the trouble-free
working time is subordinated to the gamma distribution. Tests of five
elements gave the following results (the element working time in hours
before failure): 50, 75, 125, 250, 300. Find by the method of greatest
plausibility the pointwise estimator of one unknown parameter β of
gamma distribution if the second parameter α = 1,12.
Direction: Use Exercise 14.10.
The answer: 75,47.

173
LECTURE 14

Exercises for Homework 14


14.12 A random variable X (the number of non-standard items in a batch)
is distributed under the Poisson law. We have the following distribution
of non-standard items in n = 200 batches (the number xi of non-standard
items in one batch is indicated in the first raw; the frequency ni – the
number of batches containing xi non-standard items – in the second
raw):
xi 0 1 2 3 4
ni 132 43 20 3 2
Find by method of moments the pointwise estimator of unknown
parameter of the Poisson distribution (batch – партия).
The answer: 0,5.
14.13 Find by method of moments on a sample x1, x2, …, xn the pointwise
estimator of unknown parameter λ of an exponential distribution of
which the density is f (x) = me- mx (x $ 0)
Direction: Take in account that M (X) = 1
m
The answer: m =* 1
xs
14.14 A random variable X (time of working an element) has an
exponential distribution f (x) = me- mx (x $ 0) We have the following
empirical distribution of average time of working n = 200 elements (the
average time xi of working an element in hours is indicated in the first
raw; the frequency ni – the amount of elements worked in average xi
hours – in the second raw):
xi 2,5 7,5 12,5 17,5 22,5 27,5
ni 133 45 15 4 2 1
Find by method of moments the pointwise estimator of unknown
parameter of the exponential distribution.
Direction: Use the solution of Exercise 14.13.

174
Exercises for Homework 14

The answer: 0,2.


14.15 Find by method of moments on a sample x1, x2, …, xn the pointwise
estimators of unknown parameters a and σ of a normal distribution of
which the density f (x) = 1 e-(x - a) /(2v )
2 2

v 2r
Direction: Equate the initial theoretical moment of the first order and
the central theoretical moment of the second order to the corresponding
empirical moments.
The answer: a* = x s, v* = Ds
14.16 A random variable X (the deviation of controllable size of an item
from nominal) is subordinated to a normal distribution with unknown
parameters a and σ. We have the following empirical distribution of
deviation from nominal of n = 200 items (the deviation xi (mm) is
indicated in the first raw; the frequency ni – the amount of items having
the deviation xi – in the second raw):
xi 0,3 0,5 0,7 0,9 1,1 1,3 1,5 1,7 1,9 2,2 2,3
ni 6 9 26 25 30 26 21 24 20 8 5
Find by method of moments the pointwise estimators of unknown
parameters a and σ of normal distribution.
Direction: Use the solution of Exercise 14.15.
The answer: a* = 1,26; σ* = 0,5.
14.17 A random variable X (the time of fail-safe working an element)
has an exponential distribution f (x) = me- mx (x $ 0) . We have the
following empirical distribution of average time of working 1000
elements (the average time xi of fail-safe working one element in hours
is indicated in the first raw; the frequency ni – the number of elements
worked in average xi hours – in the second raw):
xi 5 15 25 35 45 55 65
ni 365 245 150 100 70 45 25

175
LECTURE 14

Find by method of the greatest plausibility the pointwise estimator of


unknown parameter λ of the exponential distribution.
The answer: 0,05.
14.18. Find by the method of greatest plausibility on a sample
x1, x2, …, xn the pointwise estimator of parameter p of a geometric
distribution:

P (X = xi) = (1 - p) x $ p i-1

where xi is the number of trials made before an appearance of an event;


p is probability of appearance of the event in one trial.
The answer: p* = 1
xs
14.19 Find by the method of greatest plausibility on a sample x1, x2, …, xn
the pointwise estimator of parameter a of Captain distribution (parameter
σ is known) of which density is
gl (x) -[g(x) - a] /(2v )
f (x) = e
2
, 2

v 2r
where g(x) is a differentiable function.
The answer: a* = ;/ g (xi) E/n
n

i=1

14.20 Find by the method of greatest plausibility on a sample x1, x2, …, xn


the pointwise estimator of parameter σ of Captain distribution (parameter
a is known).
n
The answer: v* = / [g (x ) - a]
i
2
/n
i=1

176
Interval Estimators of Parameters. Testing of Statistical Hypotheses

LECTURE 15

Interval Estimators of Parameters.


Testing of Statistical Hypotheses
Interval Estimators of Parameters
Pointwise estimators of an unknown parameter are good as initial results
of processing observations. Their disadvantage is that it is unknown
with which precision they give an estimated value. For samples of small
size the question on precision of estimators is very significant, since it
can be a great discrepancy between θ and θ* in this case. In addition,
at solving practical tasks it is often required to determine the reliability
of these estimators. Therefore the problem on approximation of the
parameter θ by a whole interval (θ1*, θ2*) (not only by one number)
arises.
An estimator of an unknown parameter is interval if it is determined by
two numbers – endpoints of an interval.
The task of interval estimation can be formulated as follows: construct
a numeric interval (θ1*, θ2*) on sample data on which we can say that
the exact value of estimated parameter is within this interval with
probability chosen beforehand.
An interval is said to be confident (or confidence interval) if it covers
a given parameter with a given reliability (confidence probability) γ.
The quantity γ is chosen beforehand, its choice depends on specifically
solved task. For example, the confidence level of an air passenger
to the reliability of an aircraft, obviously, should be greater than the
buyer's confidence level to the reliability of a TV set, lamp, toy and etc.
The reliability γ is chosen equal to 0,9; 0,95; 0,99 or 0,999. Then it is
practically reliably finding the parameter θ in the confidence interval.
1. An interval estimator (with reliability γ) of mathematical expectation
a of a normally distributed quantitative attribute X on sample mean ͞xS at
known mean square deviation σ of a parent population is the confidence
interval

177
LECTURE 15

x s - t $ (v/ n) < a < x s + t $ (v/ n)


where t $ (v/ n) = d is the accuracy of estimation, n is sample size, t
is value of argument of Laplace function Φ(t) (Appendix 1) for which
Φ(t) = γ/2.
If σ is unknown (and sample size n < 30) then

x s - tc $ (s/ n) < a < x s + tc $ (s/ n)

where s is revised sample mean square deviation, tγ can be found by


Table (Appendix 2) on given n and γ.
2. An interval estimator (with reliability γ) of mean square deviation
σ of a normally distributed quantitative attribute X on revised sample
mean square deviation s is the confidence interval
s $ (1 - q) < v < s $ (1 + q) for (q < 1)
0 < v < s $ (1 + q) for (q > 1)
where q is found by Table (Appendix 3) on given n and γ.
3. An interval estimator (with reliability γ) of unknown probability p
of a binomial distribution on relative frequency w is the confidence
interval (with approximated endpoints p1 and p2)

p1 < p < p2,


where
$ ;w + + ` t j E,
n t2 - w (1 - w) 2
p1 = 2 t
t +n 2n n 2n

$ ;w + + ` t j E,
n t2 + w (1 - w) 2
p2 = 2 t
t +n 2n n 2n

where n is the general number of trials; m is the number of appearances


of an event; w is the relative frequency that is equal to ratio m/n; t is

178
Interval Estimators of Parameters

the value of argument of Laplace function (Appendix 1) for which


Φ(t) = γ/2 (γ is a given reliability).
Remark. For large values n (order of hundreds) we can take as
approximated boundaries of the confidence interval
w (1 - w) w (1 - w)
p1 = w - t , p2 = w + t
n n

Example. Find the confidence interval for an estimator with


reliability 0,95 of unknown mathematical expectation a of a normally
distributed attribute X of a parent population if the parent mean square
deviation σ = 5, sample mean and the sample size n = 25.
Solution: It is required to find the confidence interval
v v (*)
xs - t $ < a < xs + t $
n n
All the quantities but t are known. Find t from the following formula:
Φ(t) = 0,95/2 = 0,475.
By Table (Appendix 1) we find t = 1,96.
Replacing t = 1,96, , ͞xS = 5, n = 25 in (*) we finally obtain the required
confidence interval: 12,04 < a < 15,96.
Example. Find minimal size of a sample at which with reliability 0,975
the accuracy of an estimator of mathematical expectation a of a parent
population on sample mean is equal to δ = 0,3 if it is known that the
mean square deviation of a normally distributed parent population
σ = 1,2.
Solution: We will use the formula determining the accuracy of estimator
of mathematical expectation of a parent population on sample mean:
d = t $ (v/ n)
We have the following: n=t2σ2/δ2 (*)
By the hypothesis, γ = 0,975; consequently, Φ(t) = 0,975/2 = 0,4875. By

179
LECTURE 15

Table (Appendix 1) we find t = 2,24. Replacing t = 2,24, σ = 1,2 and


δ = 0,3 in (*), we obtain the required size of a sample: n = 81.
Example. A sample of size n = 10 has been extracted from a parent
population:
variant xi –2 1 2 3 4 5
frequency ni 2 1 2 2 2 1
Estimate with reliability 0,95 the mathematical expectation a of a
normally distributed attribute of a parent population on sample mean
by the confidence interval.
Solution: Find the sample mean and revised mean square deviation by
the formulas:

xs =
/n x ,i i
s=
/ n (x - x )
i i s
2

n n-1
Replacing the data of the example in these formulas, we obtain: ͞xS = 2,
s = 2,4.
Find tγ. By using Table (Appendix 2) on γ = 0,95 and n = 10 we find
tγ = 2,26.
Find the required confidence interval:
x s - tc $ (s/ n) < a < x s + tc $ (s/ n)
Replacing ͞xS = 2, s = 2,4, tγ = 2,26, n = 10, we obtain the required
confidence interval
0,3 < a < 3,7
covering the unknown mathematical expectation a with reliability 0,95.
Example. By data of a sample of size n = 16 from a parent population
the following has been found: the revised mean square deviation s = 1
of a normally distributive quantitative attribute. Find the confidence
interval covering parent mean square deviation σ with reliability 0,95.
Solution: The task is reduced to finding the confidence interval

180
Interval Estimators of Parameters

s · (1 – q) < σ < s·(1 + q) (for q < 1),

or 0 < σ < s·(1 + q) (for q > 1) (*)


Using the data γ = 0,95 and n = 16 by Table (Appendix 3) we find
q = 0,44. Since q < 1, replacing s = 1, q = 0,44 in (*), we obtain the
required confidence interval
0,56 < σ < 1,44.
Example. 12 measurements of some physical quantity have been made
by one device (without systematic mistakes), and the revised mean
square deviation s of random mistakes appeared equal to 0,6. Find the
accuracy of the device with reliability 0,99. It is supposed that results
of measurements are normally distributed.
Solution: The accuracy of the device is characterized by mean square
deviation of random mistakes of measuring. Therefore the task is
reduced to finding the confidence interval covering σ with the given
reliability γ = 0,99:
s · (1 – q) < σ < s·(1 + q) (for q < 1), (*)
Using the data γ = 0,99 and n = 12 by Table (Appendix 3) we find q = 0,9.
Since q < 1, replacing s = 0,6; q = 0,9 in (*), we obtain the required
confidence interval
0,06 < σ < 1,14.
Example. Independent trials are made with identical but unknown
probability p of appearance of an event A in each trial. Find the
confidence interval for estimation of probability p with reliability 0,95
if the event A has appeared 15 times in 60 trials.
Solution: By the hypothesis n = 60, m = 15, γ = 0,95. Find the relative
frequency of appearance of the event A: w = m/n = 15/60 = 0,25.
Find t from the formula Φ(t) = γ/2 = 0,95/2 = 0,475. By Table of Laplace
function (Appendix 1) we find t = 1,96.

181
LECTURE 15

Find boundaries of the required confidence interval:

$ ;w + + ` t j E,
n t2 - w (1 - w) 2
p1 = 2 t
t +n 2n n 2n

$ ;w + + ` t j E,
n t2 + w (1 - w) 2
p2 = 2 t
t +n 2n n 2n

Replacing n = 60, w = 0,25; t = 1,96 in these formulas, we obtain


p1 = 0,16; p2 = 0,37.
Thus, the required confidence interval 0,16 < p < 0, 37.
Example. An experimental slot machine providing an appearance
of a winning only once of 100 tossings of a coin in the machine has
been made. 400 trials have been made to check suitability of the slot
machine, and the winning has appeared 5 times in these trials. Find
the confidence interval covering unknown probability of appearance of
the winning with reliability γ = 0,999 (slot machine – игровой автомат;
winning – выигрыш; suitability – пригодность).
Solution: Find the relative frequency of appearance of
the winning: w = m/n = 5/400 = 0,0125. Find t from the formula
Φ(t) = γ/2 = 0,999/2 = 0,4995. By Table of Laplace function (Appendix 1)
we find t = 3,3.
Taking in account that n = 400 is great, we use the approximate formulas
for finding boundaries of the confidence interval:
w (1 - w) w (1 - w)
p1 = w - t , p2 = w + t
n n
Replacing w = 0,0125; t = 3,3; n = 400 in these formulas, we obtain
p1 = – 0,0058, p2 = 0,0308. Thus, the required confidence interval
0 < p < 0,0308.

Testing of Statistical Hypotheses


A hypothesis on a kind of unknown distribution or on parameters

182
Testing of Statistical Hypotheses

of known distributions is said to be statistical. A tested hypothesis


H0 is called null (basic). The hypothesis H1 that contradicts the null
hypothesis is called competing (alternative).
We differ hypotheses that contain only one and more than one
assumption. A hypothesis containing only one assumption is called
simple.
A hypothesis containing of finitely many or infinitely many simple
hypotheses is called complex.
Mistakes of two sorts can be admitted in result of testing a hypothesis.
A mistake of the first sort consists of that the correct null hypothesis
will be rejected. The probability of mistake of the first sort is called
a significance level and is denoted by α. A mistake of the second
sort consists of that the incorrect null hypothesis will be accepted.
The probability of mistake of the second sort is denoted by β.
A random variable K that serves for testing a hypothesis is called
statistical criterion (or just criterion). The value of criterion that has
been calculated on samples is called observed (empirical) value Kobs.
The set of values of criterion at which the null hypothesis is rejected is
called the critical area. The set of values of criterion at which the null
hypothesis is accepted is called the area of acceptance of the hypothesis
(the area of allowable values).
The basic principle of testing statistical hypotheses: if an observed
value of criterion belongs to the critical area then the null hypothesis
is rejected; if an observed value of criterion belongs to the area
of acceptance of the hypothesis then the hypothesis is accepted.
Points separating the critical area from the area of acceptance of
a hypothesis are called critical points (boundaries) kcr. The critical area
defined by the inequality K > kcr, where kcr is a positive number, is
called right-handed. The critical area defined by the inequality K < kcr,
where kcr is a negative number, is called left-handed. The critical area
defined by the inequalities K < k1 and K > k2, where k2 > k1, is called two-
handed. In particular, if critical points are symmetric regarding to zero

183
LECTURE 15

then two-handed critical area is defined by the following inequalities


(in assumption that kcr > 0):
K < – kcr, K > kcr, or by identity inequality |K| > kcr.
A significance level α is given for finding the critical area, and after
critical points is found taking in account the following:
a) For right-handed critical area P(K > kcr) = α (kcr > 0).
b) For left-handed critical area P(K < kcr) = α (kcr < 0).
c) For two-handed symmetric area
P(K > kcr) = α / 2 (kcr > 0) P(K < – kcr) = α / 2
The probability of hit of criterion in the critical area provided that the
competing hypothesis is true is called the power of criterion. In other
words, the power of criterion is the probability that the null hypothesis
will be rejected if the competing hypothesis is true.

Comparing Two Dispersions of


Normal Parent Populations
The revised sample dispersions sX2 and sγ2 have been found on
independent samples with sizes n1 and n2 extracted from normal parent
populations. It is required to compare these dispersions.
Rule 1. To test the null hypothesis H0: D(X) = D(Y) on equality of parent
dispersions of normal populations at the competing hypothesis H1:
D(X) > D(Y) for given significance level α, it is necessary to calculate
an observed value of criterion (the ratio of greater revised dispersion
2
to smaller one) Fobs = sG2 and by Table of critical points of Fisher-
ss
Snedecor distribution, on given significance level α and numbers
of degrees of freedom k1 = n1 – 1, k2 = n2 – 1 (k1 is the number
of degrees of freedom of greater revised dispersion) to find critical
point Fcr(α; k1, k2). If Fobs < Fcr then there is no reason to reject the null

184
Comparing Two Dispersions of Normal Parent Populations

hypothesis. If Fobs > Fcr, the null hypothesis is rejected.


Rule 2. If the competing hypothesis H1 is D(X) ≠ D(Y), the critical point
Fcr(α/2; k1, k2) is found on significance level α/2 (less than in two times
from the given) and numbers of degrees of freedom k1 and k2 (k1 is the
number of degrees of freedom of greater dispersion). If Fobs < Fcr then
there is no reason to reject the null hypothesis. If Fobs > Fcr, the null
hypothesis is rejected.
Example. The revised sample dispersions and have been found on
two independent samples with sizes n1 = 11 and n2 = 14 extracted
from normal parent populations X and Y. Test the null hypothesis H0:
D(X) = D(Y) on equality of parent dispersions for the significance level
α = 0,05 at the competing hypothesis H1: D(X) > D(Y).
Solution: Find the ratio of greater revised dispersion to smaller one:

sG2 = 0, 76 =
Fobs = 2
s s2 0, 38
By the hypothesis of the example the competing hypothesis has the
following form: D(X) > D(Y). Therefore the critical area is right-handed.
By Table (Appendix 5) on the significance level α = 0,05 and
numbers of degrees of freedom k1 = n1 – 1 = 11 – 1 = 10 and
k2 = n2 – 1 = 14 – 1 = 13 find the critical point:
Fcr(0,05; 10; 13) = 2,67.
Since Fobs < Fcr, there is no reason to reject the hypothesis on equality
of parent dispersions. In other words, the revised sample dispersions are
non-significantly differed.
Example. The revised sample dispersions sX2 = 0,84 and sγ2 = 2,52
have been found on two independent samples with sizes n1 = 14 and n2
= 10 extracted from normal parent populations X and Y. Test the null
hypothesis H0: D(X) = D(Y) on equality of parent dispersions for the

185
LECTURE 15

significance level α = 0,1 at the competing hypothesis H1: D(X) ≠ D(Y).


Solution: Find the ratio of greater revised dispersion to smaller one:
s G2 = 2, 52 =
Fobs = 3
s s2 0, 84
By the hypothesis of the example the competing hypothesis has the
following form: D(X) ≠ D(Y). Therefore the critical area is two-handed.
With regarding to Rule 2 at finding a critical point it should take the
significance level that is less than in two times from the given level.
By Table (Appendix 5) on the significance level α/2 = 0,1/2 = 0,05 and
numbers of degrees of freedom k1 = n1 – 1 = 10 – 1 = 9 and k2 = n2 – 1
= 14 – 1 = 13 find the critical point:
Fcr(0,05; 9; 13) = 2,72.
Since Fobs > Fcr, we reject the null hypothesis on equality of parent
dispersions.
Example. Measurements of the same physical quantity have been made
by two methods. The following results have been obtained:
a) By the first method: x1 = 9,6; x2 = 10,0; x3 = 9,8; x4 = 10,2; x5 = 10,6;
b) By the second method: y1 = 10,4; y2 = 9,7; y3 = 10,0; y4 = 10,3.
Can we consider that both methods provide (guarantee) an identical
accuracy of measuring if the significance level is α = 0,1? It is supposed
that the results of measuring are normally distributed and the samples
are independent.
Solution: We will judge on accuracy of the methods by quantities of
dispersions. Thus, the null hypothesis has the form H0: D(X) = D(Y). We
take H1: D(X) ≠ D(Y) as the competing hypothesis.
Find the sample dispersions. To simplify our calculations, we pass to
conditional variants:
ui = 10xi – 100, νi = 10yi – 100.

186
Comparing Two Dispersions of Normal Parent Populations

We obtain the following conditional variants:


ui –4 0 –2 2 6
vi 4 –3 0 3
Find the revised sample dispersions:

su2 =
/u 2
i - ^/ uih /n1
2

=
(16 + 4 + 4 + 36) - 22 /5
= 14, 8;
n1 - 1 5-1

s v2 =
/v 2
i - ^/ vih /n2
2

=
(16 + 9 + 9) - 42 /4
= 10.
n2 - 1 4-1
Compare the dispersions. Find the ratio of greater revised dispersion to
smaller one (each of dispersions has been increased in 100 times, but
their ratio has not been changed):
s 2X = su2 = 14, 8 =
Fobs = 1, 48
sY2 s v2 10
By the hypothesis of the example the competing hypothesis has the
following form: D(X) ≠ D(Y). Therefore, the critical area is two-handed.
With regarding to Rule 2 at finding a critical point it should take the
significance level that is less than in two times from the given level.
By Table (Appendix 5) on the significance level α/2 = 0,1/2 = 0,05
and numbers of degrees of freedom k1 = n1 – 1 = 5 – 1 = 4 and
k2 = n2 – 1 = 4 – 1 = 3 find the critical point:
Fcr(0,05; 4; 3) = 9,12.
Since Fobs < Fcr, there is no reason to reject the hypothesis on equality
of parent dispersions. In other words, the revised dispersions are non-
significantly differed, and consequently both methods provide an
identical accuracy of measuring.

187
LECTURE 15

Comparing a Revised Sample Dispersion


with a Hypothetical Parent Dispersion of a Normal Population
Denote the sample size by n on which the revised dispersion s2 has been
found.
Rule 1. To test the null hypothesis H0: σ 2 = σ 20 on equality of unknown
parent dispersion σ 2 to the hypothetical (supposed) value σ 20 for a
given significance level α at the competing hypothesis H1: σ 2 > σ 20 it is
necessary to calculate an observed value of criterion
(n - 1) s2
|2obs =
v20
and by Table of critical points of distribution χ2, on the given significance
level α and the number of degrees of freedom k = n – 1 to find the
critical point χ2cr (α; k). If χ2obs < χ2cr , there is no reason to reject the null
hypothesis. If χ2obs > χ2cr, the null hypothesis is rejected.
Rule 2. If the competing hypothesis H1 is σ 2 ≠ σ 20, the left
|χ2left c (1 – α / 2; k) and the right χ2right cr(α / 2; k) critical points are found.
If left χ2left c < χ2obs < χ2right cr there is no reason to reject the null hypothesis.
If χ2obs < χ2left cr or χ2obs > χ2right cr the null hypothesis is rejected.
Rule 3. If the competing hypothesis H1 is σ 2 < σ 20, the critical point
χ2cr(1 - α; k) is found. If χ2obs > χ2cr(1 - α; k) there is no reason to reject
the null hypothesis. If χ2obs < χ2cr(1 - α; k) the null hypothesis is rejected.
Remark. If the number of degrees of freedom k > 30, the critical point
χ2cr(α; k) can be found from the Wilson-Gilferti equality:

|2cr (a, k) = k $ [1 - 2/9k + za 2/ (9k)] 3

where zα is found using Laplace function (Appendix 1) from the


equality Φ(zα) = (1 – 2α) / 2.

188
Comparing a Revised Sample Dispersion

Example. A sample of size n = 21 has been extracted from a normal


parent population and the revised sample dispersion s2 = 16,2 has been
found. It is required to test the null hypothesis H0: σ 2 = σ 20 = 15, at the
significance level 0,01 taking H1: σ 2 > 15 as the competing hypothesis.
Solution: Find the observed value of criterion:
(n - 1) s2 (21 - 1) $ 16, 2
|2obs = = = 21, 6
v20 15
By the hypothesis of the example the competing hypothesis has the
following form: σ 2 > 15. Therefore the critical area is right-handed (Rule
1). By Table (Appendix 4), the significance level 0,01 and the number
of degrees of freedom k = n – 1 = 21 – 1 = 20 find the critical point
χ2cr(0,01;20) = 37,6. Since χ2obs < χ2cr, there is no reason to reject the null
hypothesis on equality of parent dispersion to the hypothetical value
σ 2 = 15. In other words, the difference between the revised dispersion
(16,2) and hypothetical parent dispersion (15) is not significant.
Example. A batch of items is accepted if the dispersion of controllable
size does not exceed 0,2 significantly. The revised sample dispersion
found on a sample of size n = 121 is sX2 = 0,3. Can we accept the batch
at the significance level 0,01?
Solution: The null hypothesis H0: σ 2 = σ 20 = 0,2. The competing
hypothesis H1: σ 2 > 0,2.
Find the observed value of criterion:
(n - 1) s 2X (121 - 1) $ 0, 3
|2obs = = = 180
v20 0, 2
The competing hypothesis has the form: σ 2 > 0,2. Consequently the
critical area is right-handed. Since there is no the number of degrees
of freedom k = 120 in Table (Appendix 4), find the critical point
approximately from the Wilson-Gilferti equality. Find firstly (taking in
account that α = 0,01) that zα = z0,01 from the equality:
Φ(z0,01) = (1 – 2α) / 2 = (1 – 2·0,01) / 2 = 0,49.

189
LECTURE 15

By Table of Laplace function (Appendix 1) using linear interpolation


we find z0,01 = 2,326. Replacing k = 120, zα = 2,326 in the Wilson-
Gilferti formula, we obtain χ2cr(0,01; 120) = 158,85. Since χ2obs > χ2cr the
null hypothesis is rejected, i.e. the batch can’t be accepted.

Glossary
discrepancy – расхождение; reliability – надежность
confidence interval – доверительный интервал
accuracy – точность; slot machine – игровой автомат
competing hypothesis – конкурирующая гипотеза; batch – партия
significance level – уровень значимости

Exercises for Seminar 15


15.1. Find the confidence interval for an estimator with reliability 0,99
of unknown mathematical expectation a of a normally distributed
attribute X of a parent population if the parent mean square deviation
σ, sample mean and sample size n are known: σ = 4, ͞xS = 10,2; n = 16.
The answer: 7,63 < a < 12,77.
15.2. Five equally exact measurements of distance between a gun and
a target have been made by the same device with the mean square
deviation of random mistakes of measurements σ = 40 m. Find the
confidence interval for an estimator of true distance a to the target
with reliability γ = 0,95, knowing that the arithmetic mean of results of
measurements ͞xS = 2000 m. It is supposed that results of measurements
are normally distributed.
The answer: 1964,94 < a < 2035,06.
15.3. An automatic machine stamps cylinders. The sample mean
of diameters of made cylinders has been calculated on a sample n =
100. Find the accuracy δ with reliability 0,95 on which the sample
mean estimates the mathematical expectation of diameters of making

190
Exercises for Seminar 15

cylinders, knowing that their mean square deviation σ = 2 mm. It is


supposed that diameters of cylinders are normally distributed.
The answer: δ = 0,392 mm.
15.4. Find minimal size of a sample at which with reliability 0,925
the accuracy of estimator of mathematical expectation of a normally
distributed parent population on sample mean is equal to 0,2 if the mean
square deviation of the parent population σ = 1,5.
The answer: n = 179.
15.5. A sample of size n = 12 has been extracted from a parent population:
variant xi –0,5 –0,4 –0,2 0 0,2 0,6 0,8 1 1,2 1,5
frequency ni 1 2 1 1 1 1 1 1 2 1
Estimate with reliability 0,95 the mathematical expectation a of a
normally distributed attribute of a parent population on sample mean
by the confidence interval.
The answer: – 0,04 < a < 0,88.
15.6. By data of nine independent equally exact measurements of
some physical quantity the following has been found: the arithmetic
mean of results of measurements ͞xS = 30,1 and revised mean square
deviation s = 6. Estimate true value of the measured quantity by the
confidence interval with reliability γ = 0,99. It is supposed that results
of measurements are normally distributed.
The answer: 25,38 < a < 34,82.
15.7. By data of a sample of size n from a parent population of a
normally distributed quantitative attribute the following has been
found: the revised mean square deviation s. Find the confidence interval
covering the parent mean square deviation σ with reliability 0,999 if n
= 10, s = 5,1.
The answer: 0 < σ < 14,28.
15.8. 10 measurements of some physical quantity have been made
by one device (without systematic mistakes), and the revised mean

191
LECTURE 15

square deviation s of random mistakes appeared equal to 0,8. Find the


accuracy of the device with reliability 0,95. It is supposed that results
of measurements are normally distributed.
The answer: 0,28 < σ < 1,32.
15.9. Independent trials are made with identical but unknown probability
p of appearance of an event A in each trial. Find the confidence interval
for estimation of probability p with reliability 0,99 if the event A has
appeared 60 times in 100 trials.
The answer: 0,47 < p < 0,71.
15.10. An event A has appeared 270 times in 360 trials in each of which
the probability of appearance of the event is constant and unknown.
Find the confidence interval covering the unknown probability p with
reliability 0,95.
The answer: 0,705 < p < 0,795.
15.11. 100 refusals have been registered when 1000 elements were
testing. Find the confidence interval covering an unknown probability p
of refusal of an element with reliability 0,95.
The answer: 0,083 < p < 0,119.
15.12. The revised sample dispersions sX2 = 34,02 and sγ2 = 12,15 have
been found on two independent samples with sizes n1 = 9 and n2 =
16 extracted from normal parent populations X and Y. Test the null
hypothesis H0: D(X) = D(Y) on equality of parent dispersions for the
significance level α = 0,01 at the competing hypothesis H1: D(X) >
D(Y).
The answer: There is no reason to reject the null hypothesis.
15.13. Two probes (samples) with sizes n1 = 10 and n2 = 8 have
been taken for comparing an accuracy of two automatic machines.
The following results have been obtained in result of measuring a
controllable size of selected items:
xi 1,08 1,10 1,12 1,14 1,15 1,25 1,36 1,38 1,40 1,42
yi 1,11 1,12 1,18 1,22 1,33 1,35 1,36 1,38

192
Exercises for Seminar 15

Can we consider that the machines have an identical accuracy [H0: D(X)
= D(Y)] if we take the significance level α= 0,1 and H1: D(X) ≠ D(Y) as
the competing hypothesis?
Direction: For simplifying calculations to pass to the following
conditional variants:
ui = 100xi – 124, νi = 100yi – 126.

The answer: There is reason to think that the accuracy of the


machines is different.
15.14. A sample of size n = 31 has been extracted from a normal parent
population:
variants xi 10,1 10,3 10,6 11,2 11,5 11,8 12,0
frequencies ni 1 3 7 10 6 3 1
It is required to test the null hypothesis H0: σ = σ = 0,18, at the 2 2
0
significance level 0,05 taking H1: σ 2 > 0,18 as the competing hypothesis.

Direction: Take conditional variants ui = 10xi – 100; calculate firstly


/n u - 6/ ni ui @ /n and then s 2 = s 2 / 102.
2 2
i i
su2 = x u
n-1

The answer: The null hypothesis is rejected.


15.15. It has been found in result of long time-study of assembling a
node by different collectors that the dispersion of this time is σ20 = 2 min2
(time-study – хронометраж; node – узел). The results of 20 observations
for an inexperienced collector working are the following:
the time of assembling
of one node in minutes xi 56 58 60 62 64
frequency ni 1 4 10 3 2
Can we consider that the inexperienced collector works rhythmically
at the significance level 0,05 (in the sense that the dispersion of time

193
LECTURE 15

expended by him is not essentially differed from the dispersion of time


of the rest collectors)?
Direction: The null hypothesis H0 is σ 2 = σ 20 = 2; the competing
hypothesis H1 is σ 2 ≠ σ 20.Take ui = xi – 60 as conditional variants and
calculate
The answer: The null hypothesis is rejected. The inexperienced
collector works non-rhythmically.

Exercises for Homework 15


15.16. Find the confidence interval for an estimator with reliability
0,99 of unknown mathematical expectation a of a normally distributed
attribute X of a parent population if the parent mean square deviation σ,
sample mean ͞xS and sample size n are known: σ = 5, ͞xS = 16,8; n = 25.
The answer: 14,23 < a < 19,37.
15.17. A sample of big butch of electro lamps contains 100 lamps. The
average durability of burning a lamp of the sample is 1000 hours. Find
the confidence interval with reliability 0,95 for average durability a of
burning a lamp of all the butch if it is known that the mean square
deviation of durability of burning a lamp σ = 40 h. it is supposed that
the durability of burning lamps is normally distributed.
The answer: 992,16 < a < 1007,84.
15.18. By data of 16 independent equally exact measurements of some
physical quantity the following has been found: the arithmetic mean of
results of measurements ͞xS = 42,8 and revised mean square deviation
s = 8. Estimate true value of the measured quantity with reliability
γ = 0,999.
The answer: 34,66 < a < 50,94.
15.19. By data of a sample of size n from a parent population of a
normally distributed quantitative attribute the following has been
found: the revised mean square deviation s. Find the confidence interval
covering the parent mean square deviation σ with reliability 0,999 if
n = 50, s = 14.

194
Exercises for Homework 15

The answer: 7,98 < σ < 20,02.


15.20. 300 trials have been made in each of which an unknown
probability p of appearance of an event A is constant. The event A has
appeared 250 times in these trials. Find the confidence interval covering
the unknown probability p with reliability 0,95.
The answer: 0,78 < p < 0,87.
15.21. 32 non-standard items have appeared in a batch of 250 items
made by an automatic machine. Find the confidence interval covering
with reliability 0,99 an unknown probability p of making a non-standard
item by the machine.
The answer: 0,07 < p < 0,18.
15.22. 100 refusals have been registered when 1000 elements were
testing. Find the confidence interval covering an unknown probability p
of refusal of an element with reliability 0,99.
The answer: 0,076 < p < 0,124.
15.23. The sample dispersions Ds(X) = 14,4 and Ds(Y) = 20,5 have been
found on two independent samples with sizes n1 = 9 and n2 = 6 extracted
from normal parent populations X and Y. Test the null hypothesis H0:
D(X) = D(Y) on equality of parent dispersions for the significance level
α = 0,1 at the competing hypothesis H1: D(X) ≠ D(Y).
Direction: It is necessary firstly to find the revised dispersions by the
following formulas:
n1 n2
s 2X = $ D (X), sY2 = $ D (Y)
n1 - 1 s n2 - 1 s
The answer: There is no reason to reject the null hypothesis.
15.24. A sample of size n = 17 has been extracted from a normal
parent population and the revised sample dispersion s2 = 0,24 has been
found. It is required to test the null hypothesis H0: σ 2 = σ 20 = 0,18,
at the significance level 0,05 taking H1: σ 2 > 0,18 as the competing
hypothesis.

195
LECTURE 15

The answer: There is no reason to reject the null hypothesis.


15.25. The accuracy of working an automatic machine is tested by
dispersion of controllable size of items that doesn’t exceed σ 20 = 0,1.
A probe of 25 randomly selected items has been taken and the following
results of measuring have been obtained:
controllable size
of items of the probe xi 3,0 3,5 3,8 4,4 4,5
frequencies ni 2 6 9 7 1
It is required to test the following: Provide the machine does the required
accuracy at the significance level 0,05?
Direction: The null hypothesis H0 is σ 2 = σ 20 = 0,1 the competing
hypothesis H1 is σ 2 > 0,1. Take ui = 10xi – 39 as conditional variants.
The answer: The null hypothesis is rejected.
15.26. A batch of items is accepted if the dispersion of controllable size
does not exceed 0,2 significantly. The revised sample dispersion found
on a sample of size n = 121 is sX2 = 0,3. Can we accept the batch at the
significance level 0,05?
Direction: The null hypothesis H0 is σ 2 = σ 20 = 0,2 the competing
hypothesis H1 is σ 2 > 0,2.
The answer: The batch is rejected.

196
Variant 1

INDIVIDUAL HOMEWORKS

Variant 1
1. A die is tossed. What is the probability that the die lands on 2? What
is the probability that the die lands on a number greater than 3?
2. A student after lectures at a university can come back home either by
a trolleybus or by a tram. He goes differently: he chooses a trolleybus
for 2/5 cases and a tram for 3/5 cases. If he goes by a trolleybus, he
comes back home to four o’clock in the afternoon in 70 % of cases, and
if he goes by a tram, he comes back – only in 65 % of cases. What is the
probability that he will come back home by four o’clock for a randomly
taken day?
3. A standard production makes on the average 97% at some factory. A
randomly selected batch of products consisting of 200 units is checked.
If 7 or more non-standard products will be among them, the batch
is rejected. Find the probability that: a) there will be 4 non-standard
products in the batch; b) the batch of products will be accepted.
4. Discrete independent random variables X and Y are given by the
following laws of distribution:

X 0 3 Y –2 –1 2
P 0,3 0,7 P 0,2 0,4 0,4
Find M (X + Y) by two ways: 1) composing the law of distribution of X
+ Y; 2) using the property: M (X + Y) = M (X) + M (Y).
5. The density of distribution of a continuous random variable X is
given:

f (x) = *2 (x - 2) if 2 < x # 3,
0 if x # 2,

1 if x > 3.
Find: a) the distribution function F(x); b) the probability of hit of the
random variable X into the interval (2,5; 7).

197
INDIVIDUAL HOMEWORKS

6. A chemist is making a weighing a certain chemical substance without


regular mistakes. Random errors of weighing are subordinated to a
normal law with mathematical expectation and dispersion equal 10
and 225 respectively. Find the probability that a weighing will be made
with a mistake which is not exceeding 5 by absolute value (weighing –
взвешивание; substance – вещество).
7. The probability of occurrence of a certain event is equal to 0,6 in each
of independent trials. 1000 trials have been made. Find the probability
that the relative frequency of occurrence of the event deviates from its
probability less than on 0,05.
8. The following data on contents of copper in ore (in grammes per one
kg of ore) have been obtained after inspecting 80 samples (ore – руда):

487 324 755 651 268 162 278 845 749 577
546 445 735 754 169 243 656 812 837 569
632 536 666 577 455 361 569 754 471 751
551 669 558 122 352 457 828 643 425 652
448 839 663 142 779 446 465 642 163 755
673 849 369 252 666 778 347 848 102 452
759 335 349 264 846 675 374 796 232 817
792 394 453 365 567 358 213 377 475 166
1) Compose the interval and the discrete variation series taking the
beginning of the first interval equal 100, and the width of each interval
equal 50.
2) Construct the histogram and the polygon of relative frequencies of
distribution.
3) Find the mode and the median (using the discrete series).
4) Find empirical functions of distribution of continuous and discrete
variation series; and construct their graphs.

198
Variant 2

Variant 2
1. Two shooters make shots in a target. The probability of hit in the
target at one shot by the first shooter is 0,7; and by the second shooter –
0,9. Find the probability that at one shot: a) both shooters will hit in the
target; b) only one of the shooters will hit in the target.
2. Two automatic devices make plastic hangers. The probability of
making a non-standard hanger by the first automatic device is 0,08; and
by the second – 0,1. The productivity of the first automatic device is
twice more than the second. Find the probability that a randomly taken
item will be non-standard.
3. The probability that a receipt written out by a shop assistant will be
paid by a buyer is equal to 0,8. The shop assistant has written out 16
receipts. Find the most probable number of paid receipts and calculate
its probability (receipt – чек).
4. Two independent random variables X and Y are given by the following
tables of distribution:
X 1 2 Y 1 3 4
P 0,7 0,3 P 0,5 0,3 0,2

Compose the law of distribution of their sum Z = X + Y and check the


property: D(X + Y) = D(X) + D(Y).
5. The random variable X is given by the distribution function:

f (x) = * x/7 if 0 < x # 7,


0 if x # 0,

1 if x > 7.
Find: a) the mathematical expectation and the dispersion; b) the
probability of hit of the random variable X into the interval (3; 8).
6. It is supposed that the strength of a let out party of a parachute fabric
is a normally distributed random variable X with the mathematical
expectation a = 200 kg/cm2 and the mean square deviation σ = 12 kg/ m2

199
INDIVIDUAL HOMEWORKS

(to let out – выпускать; parachute fabric – парашютная ткань).


Find: 1) the differential function of distribution f(x) and the integral
function of distribution F(x); 2) the probability that X will take on a
value from 175 up to 225 kg/cm2.
7. A factory makes products, and 80% of them are the first grade.
Estimate the probability that the part of products of the first grade
among 10000 made products will differ from the probability of making

227 512 405 292 630 315 424 222 377 596
318 601 132 115 518 293 488 103 294 518
402 326 305 217 298 585 312 302 418 298
603 505 292 285 422 215 518 651 503 258
195 497 612 306 192 465 142 686 320 202

a product of the first grade no more than on 0,03 by absolute value.


8. There are the following data on lifetime of a separate parachute
produced by a certain factory after studying 50 samples:
1) Compose the interval and the discrete variation series taking the
beginning of the first interval equal 100, and the width of each interval
equal 50.
2) Construct the histogram and the polygon of relative frequencies of
distribution.
3) Find the mode and the median (using the discrete series).
4) Find empirical functions of distribution of continuous and discrete
variation series; and construct their graphs.

200
Variant 3

Variant 3
1. A die is tossed once. Find the probabilities of the following events:
a) appearance of an odd number of aces;
b) appearance of no less than 4 aces;
c) appearance of no more than 2 aces.
2. Products of a certain kind are delivered in a shop from three factories.
The first factory delivers 45% of all products, the second – 40%, and the
third – 15%. The probability that a product of the first factory is non-
defective is equal to 0,6, of the second – 0,7, and of the third – 0,85.
One product has been bought. It appeared non-defective. Determine the
probability that the bought product has been made by the third factory.
3. A coming up seeds of a given grade of cucumber is 0,8. Find the
probability that the number of grown seeds will be between 775 and
810 from 1000 planted seeds.
4. Two independent random variables X and Y are given by the following
tables of distribution:

X 1 2 4 Y 2 3
P 0,4 0,3 0,3 P 0,7 0,3

Compose the table of distribution of the random variable X – Y and


check the property: D(X – Y) = D(X) + D(Y).
5. A random variable X is given by the density of distribution
1
f (x) = x in the interval (0; 4), and f(x) = 0 outside of the interval.
8
Find: a) the dispersion of the random variable X; b) the probability of
hit of the random variable X into the interval (2; 5).
6. A fat content of milk of cows in collective farms of a region (in
percentage) is a normally distributed random variable with mathematical
expectation 4 and mean square deviation 1. Find the probability that the

201
INDIVIDUAL HOMEWORKS

random variable differs from its mathematical expectation no more than


0,3 (collective farm – колхоз).
7. The dispersion of each of pairwise independent random variables
does not exceed 15. Estimate the probability that the deviation of the
arithmetic mean of 2000 these variables from the arithmetic mean of
their mathematical expectations will not exceed 0,5.
8. There are the following data on the size of annual revenues of 60
joint-stock companies:

812 521 1222 1140 1282 1244 698 1009 565 1396
723 701 755 592 1195 983 772 1125 638 1226
618 859 205 677 1086 1349 845 1250 732 1122
749 829 912 789 578 1310 926 1368 948 1085
632 918 545 872 909 1219 925 1294 608 720
502 1012 615 1228 1128 769 552 836 542 828

1) Compose the interval and the discrete variation series taking the
beginning of the first interval equal 500, and the width of each interval
equal 50.
2) Construct the histogram and the polygon of relative frequencies of
distribution.
3) Find the mode and the median (using the discrete series).
4) Find empirical functions of distribution of continuous and discrete
variation series; and construct their graphs.

202
Variant 4

Variant 4
1. Assume that the probability to destroy a tank by one grenade is 0,6.
What is the probability that 3 grenades will destroy a tank if it is enough
one hit in the target for destroying a tank? (to destroy – уничтожить;
grenade – граната).
2. A buyer can direct to one of three cash desks to pay for milk. The
probabilities of directing to each cash desk depend on their location and
are equal 0,3; 0,6; 0,1 respectively. The probability that milk available
in a cash desk will be sold out to the moment of arrival of the buyer
equal for cash desks 0,7; 0,6; 0,5 respectively. The buyer has directed
to one of cash desks and bought milk. Find the probability that he or she
has bought milk in the second cash desk (cash desk – касса).
3. The probability of striking a target by a shooter from a rifle at one
shot is 0,8. Find the probability that the shooter will hit in the target at
200 shots no less than 150 and no more than 175 times.
4. Two independent random variables X and Y are given by the following
tables of distributions:
X 3 5 7 Y 2 3
P 0,2 0,4 0,4 P 0,7 0,3

Compose the table of distribution of the random variable Z = X + Y and


check the property: M (X + Y) = M (X) + M (Y).
5. The probability density of distribution of a random variable X is

f (x) = )0 if - 3 < x < 2,


3
C/x if 2 # x < 3.
Find: a) the parameter C; b) the distribution function F(x); c) the
probability that the random variable X will take on a value from the
interval (3; 9).
6. The size of diameter of a steel wire letting out by an automatic
device is distributed under a normal law with mathematical expectation
a = 4 mm and dispersion 0,64. Find the probability that the diameter of

203
INDIVIDUAL HOMEWORKS

a randomly taken steel wire: a) will be from 3,6 up to 4,3 mm; b) differs
from the mathematical expectation no more than on 0,1 mm.
7. The discrete random variable X is given by the following law of
distribution:
X 1 2 3
Р 0,3 0,1 0,6

By using the Chebyshev’s inequality estimate the probability that the


random variable X differs from its mathematical expectation no more
than on 2,5 by absolute value.
8. There are the following data on the size of monthly salaries of
70 workers of service sphere:
157 265 343 452 367 254 199 251 357 456
271 353 241 361 477 658 345 168 255 647
565 463 274 271 363 354 252 388 462 637
381 364 458 151 452 463 271 564 268 587
453 186 447 369 647 366 461 463 372 553
662 333 358 638 357 352 439 449 249 448
354 175 553 249 462 451 633 372 655 151
1) Compose the interval and the discrete variation series taking the
beginning of the first interval equal 150, and the width of each interval
equal 50.
2) Construct the histogram and the polygon of relative frequencies of
distribution.
3) Find the mode and the median (using the discrete series).
4) Find empirical functions of distribution of continuous and discrete
variation series; and construct their graphs.

204
Variant 5

Variant 5
1. A student knows 25 of 30 questions of a program. Determine the
probability that he or she will answer both of two randomly chosen
questions at an exam.
2. There are 50% of the first-year students (60% of them are girls), 35
% of the second-year (50% – girls), and 15 % of the third-year (40%
– girls) in a hostel. All girls are on duty at the entrance in the hostel by
turns. Find the probability that a girl of the second year is on duty at
the entrance for a randomly chosen day (hostel – общежитие; to be on
duty – дежурить; by turns – по очереди).
3. A batch of products of some factory has been delivered in a warehouse.
It is known that the factory lets out on the average 90% of production of
the first grade. 4 products are randomly selected. What is the probability
that no less than two products of the first grade will be among them?
4. Two independent random variables X and Y are given by the following
tables of distribution:
X 0 3 5 Y 3 7
P 0,4 0,2 0,4 P 0,7 0,3

Compose the table of distribution of the random variable Z = X + Y and


check the property: M (X + Y) = M (X) + M (Y).
8
5. A random variable X is given by the density of distribution f (x) = 9 x
in the interval (0; 3/2), and f(x) = 0 outside of the interval. Find:
a) the mathematical expectation and the dispersion of the random
variable X;
b) the probability of hit of the random variable X into the interval
(1; 4).
6. Let the weight of a stick of smoked sausage of a given batch is a
random variable X distributed under a normal law of distribution with
mathematical expectation 400 g and mean square deviation 10 g.
Determine the probability that the weight of a randomly taken stick

205
INDIVIDUAL HOMEWORKS

from the batch: a) is in limits from 390 g up to 420 g; b) differs from


the mathematical expectation no more than on 2,8 g by absolute value
(stick of smoked sausage – палка копченой колбасы).
7. Let P(|X – M(X)|) < C) ≥ 0,95, D(X) = 5. By using the Chebyshev’s
inequality find С.
8. There are the following data on cost in tenghe of 100 available goods
152 116 150 142 162 164 176 112 112 154
148 154 152 124 182 134 156 130 144 131
142 124 151 142 162 148 163 164 140 118
178 158 126 172 172 161 104 147 170 132
152 166 142 168 152 102 146 140 154 161
176 125 138 181 144 154 129 154 169 124
196 144 181 131 165 134 163 145 133 125
171 136 175 142 182 149 153 120 108 137
144 126 152 168 186 184 162 178 178 198
136 134 160 132 112 119 148 151 165 153

in a shop:
1) Compose the interval and the discrete variation series taking the
beginning of the first interval equal 100, and the width of each interval
equal 10.
2) Construct the histogram and the polygon of relative frequencies of
distribution.
3) Find the mode and the median (using the discrete series).
4) Find empirical functions of distribution of continuous and discrete
variation series; and construct their graphs.

206
Variant 6

Variant 6
1. There are 100 tickets in a lottery, and 10 of them are winning. A
participant of the lottery buys two tickets. Determine the probability
that at least one of them will be winning.
2. At a factory making toys the first machine produces 60 %, the second
– 30 %, and the third – 10 % of all toys. The probability that a randomly
chosen toy produced by the first machine is defective equals 0,07, by
the second – 0,03, and by the third – 0,01. What is the probability that a
randomly chosen toy will be defective?
3. The probability that a coin lands on heads at one tossing is equal to
0,5. Calculate the probability that the coin lands on heads exactly two
times at four tossings.
4. Two independent random variables X and Y are given by the following
tables of distribution:

X 2 3 Y 0 2 3
P 0,6 0,4 P 0,2 0,6 0,2

Compose the table of distribution of the random variable Z = X + Y and


check the property: M (X + Y) = M(X) + M (Y).
5. A random variable X is given by the integral function:
Z
]]0 2 if x # 0,
x
f (x) = [ if 0 < x # 2 ,
2
]]
\1 if x > 2.
Find a) the differential function f(x); b) the mathematical expectation
and the dispersion of X; c) the probability of hit of X into the interval
(3/5; 6/5).
6. It is supposed that the durability limit of a let out party of a parachute
fabric is a normally distributed random variable X with the dispersion
equal 100 kg/cm2. What maximal deviation of the durability limit of a

207
INDIVIDUAL HOMEWORKS

tested sample of a parachute fabric from the mathematical expectation


can be guaranteed with probability 0,925 by absolute value?
7. The probability of appearance of an event is 1/6 in each trial. By using
the Chebyshev’s inequality estimate the probability that the number of
appearances of the event differs from the mathematical expectation less
than on 30 if 900 trials will be made.
8. There are the following data on the size of monthly salaries of
bookkeepers of 100 commercial firms:

359 365 343 352 367 354 399 351 352 356
371 353 341 361 377 358 345 368 355 347
365 363 374 371 363 354 352 388 362 337
381 361 358 351 352 363 371 364 368 387
353 386 347 369 347 366 361 363 372 353
362 335 358 338 357 352 339 349 349 348
354 375 353 349 362 351 333 372 355 351
372 343 378 347 352 354 338 378 364 361
364 353 357 375 349 346 352 339 356 352
343 346 356 357 384 375 366 392 385 362

1) Compose the interval and the discrete variation series taking the
beginning of the first interval equal 330, and the width of each interval
equal 10.
2) Construct the histogram and the polygon of relative frequencies of
distribution.
3) Find the mode and the median (using the discrete series).
4) Find empirical functions of distribution of continuous and discrete
variation series; and construct their graphs.

208
Variant 7

Variant 7
1. A buyer has bought a TV set and a video recorder. The probability
that a TV set will sustain its warranty period is 0,9 and a video recorder
– 0,85. Find the probability that: a) both devices will sustain their
warranty periods; b) at least one of them will sustain the own warranty
period (to sustain a warranty period – выдержать гарантийный срок).
2. Assume that 40% of all men and 30% of all women are short-sighted
persons. A randomly chosen person is short-sighted. Assume that the
numbers of men and women are equal. What is the probability that this
person is a woman? (short-sighted – близорукий).
3. A coming up seeds of a given grade of rice is estimated with the
probability equal 0,9. What is the probability that no less than five seeds
will come up from the sown six ones? (to come up – всходить; seed –
семя; grade – сорт; rice – рис; to sow – сеять).
4. Two independent random variables X and Y are given by the following
tables of distribution:
X 1 2 3 Y 3 4
P 0,3 0,5 0,2 P 0,6 0,4

Compose the table of distribution of the random variable Z = X · Y and


check the property: M(X · Y) = M(X) · M(Y).
5. The probability density of distribution of a random variable is given
by: f (x) = )C if - 3 < x < 3,
0 if x < - 3 or x > 3.
Find the parameter C, the mathematical expectation, the dispersion, the
distribution function of the random variable X, and the probability of
the event: –2 < X < 2.
6. A normally distributed random variable X is given by the following
density of distribution:
1 2
(x - 1 )
f (x) = e- 32 .
4 2r

209
INDIVIDUAL HOMEWORKS

Find the probability of hit of the random variable X into the interval
(2; 4).
7. The average value of the weight of a product is 300 grammes, and
the dispersion is 20. By using the Chebyshev’s inequality estimate the
probability that the weight of a randomly taken product will be no less
than 280 g and no more than 320 g.
8. There are the following data on the size of revenues of 100 small
banks:

212 221 222 240 232 224 230 209 225 222
223 201 255 232 235 238 222 253 238 226
218 259 205 237 208 249 245 250 232 228
249 229 212 217 212 210 226 222 218 225
232 218 245 222 209 229 225 242 228 220
202 212 215 228 228 229 252 225 242 228
218 220 212 215 243 218 242 229 218 236
206 226 240 216 233 205 223 242 228 232
212 240 203 222 218 225 233 224 211 216
219 223 244 238 230 231 226 234 232 216

1) Compose the interval and the discrete variation series taking the
beginning of the first interval equal 200, and the width of each interval
equal 10.
2) Construct the histogram and the polygon of relative frequencies of
distribution.
3) Find the mode and the median (using the discrete series).
4) Find empirical functions of distribution of continuous and discrete
variation series; and construct their graphs.

210
Variant 8

Variant 8
1. There are 13 green and 19 red balls in an urn. One ball has been
randomly taken out from the urn and put aside. This ball was green.
After that one more ball is randomly taken out from the urn. Find the
probability that this ball will be also green.
2. Two persons have typed an identical quantity of pages of a text. The
probability that the first person will make a mistake is 0,08, and the
second – 0,12. One mistake has been found out at verification of the
text. Find the probability that the first person has mistaken.
3. The probability of hit in a target at each shot by a rifle is 0,7. How
many shots should make in order that the most probable number of hits
was equal to 30?
4. Two independent random variables X and Y are given by the following
tables of distributions:
X 2 3 5 Y 1 6
P 0,5 0,2 0,3 P 0,4 0,6

Compose the table of distribution of the random variable Z = X + Y and


check the property: M (X + Y) = M(X) + M (Y).
5. The density of distribution of a continuous random variable X is
given by: Z
]]0 if x # 1,
-
x 1
f (x) = [ if 1 < x # 3,
]] 2
\1 if x > 3.
Find: a) the distribution function F(x); b) the probability of hit of the
random variable X into the interval (2; 4).
6. A normally distributed random variable X is given by the following
density of distribution:
1 2
( x - 3)
f (x) = e- 50 .
5 2r

211
INDIVIDUAL HOMEWORKS

Find the probability of hit of the random variable X into the interval
(– 2; 4).
7. The probability of occurrence of a certain event is 0,6 in each trial.
By using the Chebyshev’s inequality estimate the probability that the
number of occurrences of the event will be in limits from 100 up to 140
if 200 independent trials will be made.
8. There are the following data on weight of an item produced by an
automatic device after studying 100 samples:
312 312 305 292 330 315 324 322 312 296
318 301 332 315 318 312 288 303 294 318
302 326 305 317 298 285 312 302 318 298
303 305 292 285 322 315 318 315 303 318
295 297 312 306 292 305 342 286 320 302
312 312 328 315 328 329 299 318 303 302
302 315 295 318 325 298 325 342 323 305
306 303 324 292 316 295 326 318 299 303
318 322 318 306 315 325 322 303 312 316
290 334 325 332 319 335 329 338 344 325
1) Compose the interval and the discrete variation series taking the
beginning of the first interval equal 280, and the width of each interval
equal 10.
2) Construct the histogram and the polygon of relative frequencies of
distribution.
3) Find the mode and the median (using the discrete series).
4) Find empirical functions of distribution of continuous and discrete
variation series; and construct their graphs.

212
Variant 9

Variant 9
1. A student came to pass an exam knowing only 38 questions of 50.
What is the probability that he or she pass the exam if the student
can’t answer a randomly chosen question, the teacher asks one more
question?
2. Two shooters make shots in a target simultaneously. It is known that
the probability of hit by the first shooter is 0,7, and by the second – 0,8.
The target has been struck once. What is the probability that the target
will not be struck by the first shooter?
3. A plane has made 25 shots in an object. The probability of hit at one
shot is 0,5. Find the most probable number of hits and the probability
of this number of hits.
4. Two independent random variables X and Y are given by the following
laws of distribution:

X 3 7 Y 0 1 3
P 0,7 0,3 P 0,1 0,5 0,4
Compose the law of distribution of the random variable Z = X · Y and
check the property: M(X · Y) = M (X) · M (Y).
5. A random variable X is given by the integral function:

F (x) = * x - 1/3 if 1/3 < x # 4/3,


0 if x # 1/3,

1 if x > 4/3.
Find: а) the differential function f(x); b) the mathematical expectation
and the dispersion of X; c) the probability of hit of the random variable
X into the interval (5/6; 6/5).
6. A normally distributed random variable X is given by the following
density of distribution:
1 2
(x - 2)
f (x) = e- 32 .
4 2r

213
INDIVIDUAL HOMEWORKS

Find the probability of hit of the random variable X into the interval
(– 3; 5).
7. There are 1500 green and 2500 red balls in an urn. 400 balls have been
taken out from the urn (with returning). Estimate the probability that
the number m of extracted green balls satisfies the double inequality:
120 < m < 180.
8. At inspecting 100 products of a given batch the following data on
weight of a separate product (in grammes) have been obtained:

887 824 855 851 868 862 878 845 849 877
846 845 835 854 869 843 856 812 837 869
832 836 866 877 855 861 869 854 871 851
851 869 858 822 852 857 828 843 825 852
848 839 863 842 879 846 865 842 863 855
873 849 869 852 866 878 847 848 802 852
859 835 849 864 846 875 874 896 832 817
892 894 853 865 867 858 813 877 875 866
848 886 868 888 832 862 844 846 852 859
858 854 831 851 859 858 867 849 853 849

1) Compose the interval and the discrete variation series taking the
beginning of the first interval equal 800, and the width of each interval
equal 10.
2) Construct the histogram and the polygon of relative frequencies of
distribution.
3) Find the mode and the median (using the discrete series).
4) Find empirical functions of distribution of continuous and discrete
variation series; and construct their graphs.

214
Variant 10

Variant 10
1. Three dice are tossed. Find the probability that the sum of landed
aces of the dice is divided on 5.
2. A collector has received 4 boxes of items made by the first factory,
and 3 boxes of items made by the second factory. The probability that
an item of the first factory is standard is equal to 0,75; the second
factory – 0,85. Find the probability that a randomly extracted item from
a randomly chosen box will be standard (collector – сборщик).
3. There is a group of 60 persons born at April. Find the probability that
the birthday for three persons will be the first of April. Assume that the
probability of a birth in a fixed day of April is equal to 1/30.
4. The probability that a book necessary for a student is available in a
library is equal to 0,4. Compose the law of distribution of the number
of libraries which will be visited by the student if there are five libraries
in the city.
5. The distribution function of a continuous random variable X is given
by:
Z0 if x # 0,
]
]
F (x) = 1 - cos 2x if 0 < x # ,
r
[ 4
]] r
1 if x> .
\ 4

Find: 1) the density of distribution; 2) the mathematical expectation and


the dispersion of X; 3) the probability of hit of the random variable X
into the interval (0; π/8).
6. A random variable X is normally distributed with mathematical
expectation M(X) = 12 and dispersion D(X) = 4. Find the probability
that:
а) X will take on a value belonging to the interval (7; 14);
b) X will differ from the mathematical expectation less than
on 1,5.

215
INDIVIDUAL HOMEWORKS

7. The dispersion of each of 1000 independent random variables does


not exceed 4. Estimate the probability that the arithmetic mean of
these random variables will differ from the arithmetic mean of their
mathematical expectations less than on 0,15.
8. There are the following data on monthly volume of cigarettes (in
thousand packs) of 100 supermarkets:

32 87 52 61 52 64 79 67 58 46
58 53 83 31 56 50 65 47 58 77
53 47 51 70 40 54 53 26 13 55
6 86 73 56 1 69 62 39 49 77
48 66 53 51 78 66 52 63 53 88
68 16 31 74 45 49 66 80 95 93
69 14 56 41 76 60 42 51 49 74
28 45 62 55 43 51 54 66 67 63
68 52 48 72 34 40 64 17 56 69
21 25 35 37 54 33 45 37 45 28

1) Compose the interval and the discrete variation series taking the
beginning of the first interval equal 0, and the width of each interval
equal 10.
2) Construct the histogram and the polygon of relative frequencies
of distribution.
3) Find the mode and the median (using the discrete series).
4) Find empirical functions of distribution of continuous and discrete
variation series; and construct their graphs.

216
Variant 11

Variant 11
1. The average number of rainy days at March is equal to 13 for some
district. What is the probability that at least one rainy day will be in first
three days of March?
2. Thirty automatic devices are working in a shop. 15 of them have
mark A, 10 – mark B and the rest 5 – mark C. The probability that the
quality of an item will be excellent for these devices is equal to 0,92;
0,86 and 0,75 respectively. What is the percentage of excellent items is
made by the shop on the whole (on the whole – в целом)?
3. A shop assistant selects at random 10 packs of goods. The probability
that a single pack of goods has no defect is 0,9. Find the most probable
number of defect-free packs among ten selected packs and calculate the
probability of this number.
4. Two independent random variables X and Y are given by the following
tables of distribution:
X 3 6 Y 1 4 7
P 0,7 0,3 P 0,1 0,7 0,2

Compose the table of distribution of the random variable Z = X · Y and


check the property: M(X · Y) = M(X) · M(Y).
5. A random variable X is given by the following function of distribution:

F (x) = * x if 0 < x # 1,
0 if x # 0,
3

1 if x > 1.

Find: a) the differential function f(x); b) the mathematical expectation


and the dispersion of X; c) the probability of hit of the random variable
X into the interval (1/2; 3).
6. A random variable X is normally distributed with mathematical
expectation equal 16 and mean square deviation equal 4. Find the

217
INDIVIDUAL HOMEWORKS

probability that as a result of trials the random variable X will take on


a value belonging to the interval (12; 20).
7. The mean square deviation of each of 100000 independent random
variables does not exceed 15. Estimate the probability that the absolute
value of deviation of the arithmetic mean of these variables from the
arithmetic mean of their mathematical expectations will not exceed 0,4.
8. There are the following data on cost (in tenghe per one kilogram) of
40 sausage products in a supermarket:

1650 516 750 1420 962 464 376 1312 712 854
348 654 852 924 882 734 176 1530 944 1031
442 824 1051 542 1062 648 163 1464 440 1618
778 958 1216 372 572 1161 804 1647 370 1332

1) Compose the interval and the discrete variation series taking the
beginning of the first interval equal 300, and the width of each interval
equal 100.
2) Construct the histogram and the polygon of relative frequencies of
distribution.
3) Find the mode and the median (using the discrete series).
4) Find empirical functions of distribution of continuous and discrete
variation series; and construct their graphs.

218
Variant 12

Variant 12
1. There are 6 winning lottery tickets of 40. Find the probability that
from three randomly chosen tickets: a) no ticket will be winning; b)
only one ticket will be winning; c) two tickets will be winning.
2. One of two shooters makes two shots. The probability of hit in a
target at one shot for the first shooter is 0,7, and for the second – 0,9.
Find the probability that the target will be struck twice.
3. The probability of hit in a target at each shot is 0,002. Find the
probability that the target will be struck at least once if 1000 shots were
made.
4. Two independent random variables X and Y are given by the following
tables of distribution:
X 2 3 Y 1 5
P 0,4 0,6 P 0,7 0,3

Compose the law of distribution of the random variable Z = X · Y and


check the property: M (X · Y) = M (X) · M (Y).
5. A random variable X is given by the integral function:

F (x) = *0, 1x - 2 if 5 < x # 7, 5,


0 if x # 5,

1 if x > 5.
Find: a) the differential function f(x); b) the mathematical expectation
and the dispersion of X; c) the probability of hit of X into the interval
(6; 10).
6. A random variable is normally distributed with dispersion equal
0,25. Find the probability that the random variable will differ from its
mathematical expectation less than on 0,4.
7. Determine the least number of items which should be taken in order
to guarantee with the probability exceeding 0,9 that the part of defective

219
INDIVIDUAL HOMEWORKS

items among them will differ from the probability of making a defective
item, equal 0,02, no more than on 0,01 by absolute value.
8. There are the following data on the size of annual bonuses of
70 senior managers of a national company:

512 521 522 540 532 524 530 509 525 522
523 501 555 532 535 538 522 553 538 526
518 559 505 537 508 549 545 550 532 528
549 529 512 517 512 510 526 522 518 525
532 518 545 522 509 529 525 542 528 520
502 512 515 528 528 529 552 525 542 528
518 520 512 515 543 518 542 529 518 536

1) Compose the interval and the discrete variation series taking the
beginning of the first interval equal 500, and the width of each interval
equal 10.
2) Construct the histogram and the polygon of relative frequencies of
distribution.
3) Find the mode and the median (using the discrete series).
4) Find empirical functions of distribution of continuous and discrete
variation series; and construct their graphs.

220
Variant 13

Variant 13
1. There is a box containing identical packs of tea of two kinds in a
warehouse. The probability that a randomly taken out pack from the
box is «Assam» equals 0,6. What is the probability that at least one
pack from two randomly taken out packs will be «Assam»?
2. Devices of a certain kind are produced by three factories: the first
factory delivers 1/3 of all devices, the second – 1/4, and the third – 5/12.
The probability of non-failure working a device produced by the first
factory is 0,7, by the second – 0,5, and by the third – 0,8. Determine the
probability that a randomly chosen device will trouble-free work.
3. A factory is making products, and 75% of them are the first grade.
80 products have been randomly selected. Find the probability that the
number of products having the first grade among the selected will be: a)
no less than 55 and no more than 65; b) exactly 58.
4. Two independent random variables X and Y are given by the following
tables of distribution:
X 2 4 Y 1 3 4
P 0,6 0,4 P 0,5 0,3 0,2

Compose the table of distribution of the random variable Z = X + Y and


check the property: D (X + Y) = D(X) + D (Y).
5. Find the distribution function of the number of hits in a target if five
shots have been made by a shooter, and the probability of hit at one shot
is 0,4. Calculate the probability that the target will be struck no less than
two times and no more than four times.
6. A normally distributed random variable X is given by the following
density of distribution:

1 2
(x - 4)
f (x) = e- 72 .
6 2r
Find the probability of hit of the random variable X into the interval
(6; 13).

221
INDIVIDUAL HOMEWORKS

7. The probability that a student of some university has a sport category


is 0,4. Estimate the probability that the number of students having a
sport category in a group of 300 students differs from the mathematical
expectation less than on 25.
8. The following data on monthly salary (in thousands of tenghe) of a
separate manager have been received after inspecting 30 commercial
banks:

271 253 362 151 164 174 166 272 185 164
351 361 252 286 259 161 287 252 166 361
175 264 249 211 356 339 178 193 343 255

1) Compose the interval and the discrete variation series taking the
beginning of the first interval equal 150, and the width of each interval
equal 30.
2) Construct the histogram and the polygon of relative frequencies of
distribution.
3) Find the mode and the median (using the discrete series).
4) Find empirical functions of distribution of continuous and discrete
variation series; and construct their graphs.

222
Variant 14

Variant 14
1. There are two prizes on 1000 tenghe, three prizes on 600 tenghe, five
prizes on 300 tenghe, and ten prizes on 100 tenghe in a lottery of 200
tickets. Somebody buys one ticket. Find the probability of: a) winning
no less than 300 tenghe; b) winning no more than 100 tenghe.
2. There are 3 green and 7 red balls in the first urn, and 4 green and 8
red balls in the second urn. One ball has been randomly extracted from
each urn, and the rest balls have been put in the third urn. Find the
probability that a randomly extracted ball from the third urn will be red.
3. The probability that an item of a given batch will be defective is 0,05.
Calculate the probability that among randomly selected 100 items:
a) exactly 10 items will be defective; b) from 3 up to 9 items will be
defective.
4. Two independent random variables are given by the following tables
of distribution:
X 0 2 3 Y 1 2
P 0,4 0,5 0,1 P 0,7 0,3

Compose the law of distribution of the random variable Z = X · Y and


check the property: M (X · Y) = M (X) · M (Y).
5. A random variable X is given by the integral function:

F (x) = *0, 5 (1 - sin x) if r/2 < x # 3r/2,


0 if x # 0,

1 if x > 3r/2.

Find: a) the differential function f(x); b) the mathematical expectation


and the dispersion of X.
6. A random variable X is normally distributed with mathematical
expectation equal 1 and dispersion equal 4. Find: a) the probability that
the random variable X will take on a value belonging to the interval

223
INDIVIDUAL HOMEWORKS

(0; 5); b) the probability that the random variable X will differ from the
mathematical expectation M(X) less than on 3.
7. The weekly need of water for a certain apartment is a random
variable with mathematical expectation 1000 litres and dispersion 900.
Estimate the probability that the nearest week the expense of water in
this apartment will be from 800 up to 1200 litres.
8. There are the following data on the size of monthly salaries of bus
drivers of 60 transport organizations:

119 165 143 152 167 154 199 151 152 156
121 153 141 161 107 158 145 168 155 147
165 163 174 101 163 154 152 128 162 137
181 161 158 171 152 163 171 164 118 187
153 186 147 169 147 166 111 163 172 123
162 135 158 138 157 152 139 149 149 148

1) Compose the interval and the discrete variation series taking the
beginning of the first interval equal 100, and the width of each interval
equal 10.
2) Construct the histogram and the polygon of relative frequencies of
distribution.
3) Find the mode and the median (using the discrete series).
4) Find empirical functions of distribution of continuous and discrete
variation series; and construct their graphs.

224
Variant 15

Variant 15
1. At a dialing a phone number a subscriber has forgotten two last
digits and typed them at random, remembering only that these digits
are even and different. Find the probability that the number has been
typed correctly.
2. There are 130 items made by the first factory, 90 – by the second
factory and 70 – by the third factory among 290 items. The probability
that an item of the first factory is defective is equal to 0,03, of the second
factory – 0,015, and of the third factory – 0,025. One item is randomly
taken. Determine the probability that the item is non-defective.
3. The probability of hit in a target at one shot by a shooter is equal
to 0,6. Determine the probability that in a series from 600 shots appears:
а) exactly 372 hits; b) from 366 up to 378 hits.
4. Two independent random variables are given by the following tables
of distribution:

X 1 3 Y 1 2 4
P 0,4 0,6 P 0,3 0,5 0,2

Compose the table of distribution of the random variable Z = X · Y and


check the property: M (X · Y) = M (X) · M (Y).
5. A random variable X is given by the integral function:

F (x) = *0, 5 (1 + cos x) if - r < x # 0,


0 if x # - r,

1 if x > 0.
Find: a) the differential function f(x); b) the mathematical expectation
and the dispersion of X.
6. A boy fishes in a small river. Assume that the weight of a caught fish
is subordinated to a normal law with mathematical expectation 300 g
and mean square deviation 20 g. Find the probability that the weight of

225
INDIVIDUAL HOMEWORKS

one caught fish will make: 1) from 250 up to 330 g, 2) no more than 340
g, 3) more than 270 g.
7. How many times is it necessary to measure a given parameter
of which the true value is equal to a in order to assert with the
probability no less than 0,975 that the arithmetic mean of these
measurements differs from a less than on 2 by absolute value if the
dispersion of each measurement is less than 24.
8. There are the following data on cost (in thousands of dollars)
of 50 vailable cars in an auto salon:

42 16 50 42 62 64 76 12 12 54
78 54 52 24 15 34 56 30 44 31
52 24 51 42 62 48 63 64 40 18
48 58 26 72 72 61 7 47 70 32
52 66 42 68 52 6 46 40 11 61

1) Compose the interval and the discrete variation series taking the
beginning of the first interval equal 0, and the width of each interval
equal 10.
2) Construct the histogram and the polygon of relative frequencies
of distribution.
3) Find the mode and the median (using the discrete series).
4) Find empirical functions of distribution of continuous and discrete
variation series; and construct their graphs.

226
Variant 16

Variant 16
1. Two dice are tossed. What is the probability that the sum of the dice
will be no more than 7 aces?
2. There are two identical boxes with balls, and 3 green and 2 red
balls in the first box, 2 green and 5 red balls in the second box. One
ball is randomly extracted from a randomly chosen box. What is the
probability that the extracted ball is red?
3. The probability of at least one hit at three shots is equal to 0,936.
What is the probability of two hits in a series of four shots?
4. Two independent random variables X and Y are given by the following
tables of distribution:
X 1 2 5 Y 0 1 4
P 0,6 0,1 0,3 P 0,7 0,2 0,1

Compose the law of distribution of the random variable Z = X + Y and


check the property: M(X + Y) = M(X) + M (Y).
5. The density of distribution of a continuous random variable X is
given by:
Z
]]0 if x # 2,
x
F (x) = [ - 1 if 2 < x # 4,
]] 2
\1 if x > 4.

Find: a) the distribution function F(x); b) the probability of hit of the


random variable X into the interval (3; 5).
6. The percent of contents of ashes in coal is a normally distributed
random variable with mathematical expectation 20% and dispersion
9%. Find: a) the differential and integral functions of distribution; b)
the probability that a randomly taken test of coal will contain from 18
up to 28% of ashes (ashes – зола; coal – уголь).

227
INDIVIDUAL HOMEWORKS

7. The weekly expense of water in an apartment is a random variable


with dispersion 1000 litres. Estimate the probability that the expense of
water in the apartment within one week will differ from the mathematical
expectation no more than on 200 litres by absolute value.
8. There are the following data on weight of a ball making by an
automatic device after studying 80 samples:

112 112 105 92 130 115 124 122 112 96


118 101 132 115 118 112 88 103 94 118
102 126 105 117 98 85 112 102 118 98
103 105 92 85 122 115 118 115 103 118
95 97 112 106 92 105 142 86 120 102
112 112 128 115 128 129 99 118 103 102
102 115 95 118 125 98 125 142 123 105
106 103 124 92 116 155 126 118 99 103

1) Compose the interval and the discrete variation series taking the
beginning of the first interval equal 80, and the width of each interval
equal 10.
2) Construct the histogram and the polygon of relative frequencies of
distribution.
3) Find the mode and the median (using the discrete series).
4) Find empirical functions of distribution of continuous and discrete
variation series; and construct their graphs.

228
Variant 17

Variant 17
1. A wardrobe mistress has given out simultaneously tickets to three
persons given their coats in a wardrobe. After that she has mixed up
all coats and hung up them at random. Find the probabilities of the
following events: a) the wardrobe mistress will give out to each of
three persons his own coat; b) only one person will receive its coat
(wardrobe mistress – гардеробщица; to mix up – перепутать; to hang
up – повесить что-либо; ticket – номерок).
2. Items from two automatic devices are delivered for an assemblage.
It is known that the first automatic device gives 0,5 % of spoilage, and
the second – 0,6 %. Find the probability that a randomly taken item for
an assemblage will be defective if 200 items have been delivered from
the first automatic device, and 300 – from the second (assemblage –
сборка).
3. 3 of 100 products on the average have a defect at a given technological
process. Determine the probability that among randomly chosen 20000
products: a) exactly 562 products will be defective; b) from 545 up to
605 products will be defective.
4. Two independent random variables X and Y are given by the following
tables of distribution:

X 2 3 Y 1 2 3
P 0,7 0,3 P 0,4 0,3 0,3

Compose the table of distribution of the random variable Z = X + Y and


check the property: M(X + Y) = M(X) + M(Y).
5. A random variable X is given by the integral function:

F (x) = * x + 1 if - 1 < x # 0,
0 if x # - 1,

1 if x > 0.

229
INDIVIDUAL HOMEWORKS

Find: a) the differential function f(x); b) the mathematical expectation


and the dispersion of X; c) the probability of hit of the random variable
X into the interval (–1/2; 1/2).
6. The dispersion of a random variable distributed under a normal law
is equal to 9 m, and the mathematical expectation is equal to 25 m. Find
boundaries in which one should expect a value of the random variable
with probability 0,9.
7. It has been established by checking the quality of produced electric
bulbs that 97% of them have no less than a guaranteed lifetime (in
hours). Estimate the probability that the part of bulbs (in a batch of
1000 electric bulbs) with the lifetime less than the guaranteed lifetime
will differ from the probability of such an electric bulb no more than
on 0,05.
8. At inspecting 60 fish packings of a given batch the following data on
weight of a separate packing (in grammes) have been obtained:

248 239 263 242 279 246 265 242 263 255
273 249 209 252 266 278 247 248 202 252
259 235 249 264 246 275 274 296 232 217
292 294 223 265 267 258 213 277 275 266
248 286 268 288 232 262 244 246 252 259
258 254 231 251 229 258 267 249 253 249

1) Compose the interval and the discrete variation series taking the
beginning of the first interval equal 200, and the width of each interval
equal 10.
2) Construct the histogram and the polygon of relative frequencies of
distribution.
3) Find the mode and the median (using the discrete series).
4) Find empirical functions of distribution of continuous and discrete
variation series; and construct their graphs.

230
Variant 18

Variant 18
1. A coin is tossed four times. Find the probability that a) the coin lands
on heads three times; b) the coin lands on tails twice.
2. There are radio tubes made by two factories on a warehouse. 60
% of them are made by the first factory and 40 % – by the second.
It is known that 85 of 100 tubes made by the first factory satisfy the
standard, and 75 of 100 tubes made by the second factory satisfy the
standard. Determine the probability that a randomly taken tube from the
warehouse will satisfy the standard.
3. Assuming that the probability of landing six aces at one tossing a die
is equal to 1/6, determine the probability that in a series of 100 tossings
the die lands on six aces: a) exactly 20 times; b) from 18 up to 30 times.
4. The laws of distribution of two independent random variables are
given by the following tables of distribution:
X –2 –1 0 Y –1 0 1
P 0,1 0,6 0,3 P 0,4 0,1 0,5

Compose the law of distribution of their product and check the following
property: M(X · Y) = M(X) · M(Y).
5. A random variable X is given by the integral function:

F (x) = *(1/2) x - 1/2


0 if x # 1,
if 1 < x # 3,
1 if x > 3.
Find: a) the differential function f(x); b) the mathematical expectation
and the dispersion of X; c) the probability of hit of the random variable
into the interval (2; 5).
6. The mathematical expectation and the dispersion of a normally
distributed random variable X are equal to 26 and 9 respectively. Find:
a) the probability that the random variable X will take on a value from
the interval (20; 29); b) the probability that the deviation X – M(X) will
be no more than 1 by absolute value.

231
INDIVIDUAL HOMEWORKS

7. The discrete random variable X is given by the law of distribution:


X 2 3 4
P 0,6 0,1 0,3
By using the Chebyshev’s inequality estimate the probability that the
absolute value of deviation of the random variable from its mathematical
expectation less than 3.
8. The following data on monthly loading (in hours) of a separate
worker have been received after inspecting 100 commercial banks:
1) Compose the interval and the discrete variation series taking the
beginning of the first interval equal 730, and the width of each interval
equal 10.
771 753 762 751 761 735 772 743 746 756
751 761 752 786 752 775 756 797 771 772
775 764 749 767 763 758 762 762 747 763
764 774 766 772 777 767 773 761 746 757
759 761 787 752 799 761 798 752 761 787
756 739 778 793 768 771 778 788 781 771
759 787 775 747 753 748 754 761 762 764
785 764 763 765 753 755 758 786 761 775
766 761 752 745 771 754 799 769 738 749
743 755 777 758 774 778 763 789 747 754
2) Construct the histogram and the polygon of relative frequencies of
distribution.
3) Find the mode and the median (using the discrete series).
4) Find empirical functions of distribution of continuous and discrete
variation series; and construct their graphs.

232
Variant 19

Variant 19
1. Two dice are tossed. Find the probabilities of the following events:
a) the sum of aces is equal to 7;
b) the product of aces is equal to 10;
c) the sum of aces is less than their product.
2. The first automatic device makes 30 %, the second – 25 %, the third
– 45 % of all nails. The spoilage makes 6, 5 and 3% respectively in
their production. A randomly chosen nail has been defective. Which
of automatic devices (1-st, 2-nd or 3-rd) has the nail been most likely
made (nail – гвоздь)?
3. The probability that a product of a given batch does not satisfy the
standard is equal to 0,3. Determine the probability that among randomly
selected 500 products:
a) exactly 140 products do not satisfy the standard;
b) from 145 up to 160 products are non-standard.
4. The table of distribution of a random variable X has the following
form:

X –1 0 2 4 5
Р 0,35 0,15 0,2 0,12 0,18
Find the mathematical expectation and the dispersion of the random
variable X.
5. The density of distribution of a random variable X is given by:

F (x) = )Cx if 0 # x # 1,
3

0 if x < 0 or x > 1
Find: a) the parameter C; b) the mathematical expectation and the
dispersion of the random variable.
6. A random variable X is distributed under a normal law. The

233
INDIVIDUAL HOMEWORKS

mathematical expectation and the mean square deviation of the random


variable are 40 and 12 respectively. Find: a) the probability that the
random variable X takes on a value belonging to the interval (20; 55);
b) the probability that the absolute value of the deviation X – M(X) is
less than 7.
7. By using the Chebyshev’s inequality estimate the probability that a
random variable X will differ from its mathematical expectation M(X)
no more than on 6 if the mean square deviation of the random variable
X is equal to 3.
8. There are the following data on annual volume of confectionery
products (in thousands of tons) of 60 commercial shops:
1) Compose the interval and the discrete variation series taking the
432 487 752 361 552 364 779 667 358 546
558 653 383 431 756 450 565 647 458 577
453 447 551 670 340 554 653 326 713 455
606 586 473 756 401 669 562 739 649 777
448 466 653 551 678 766 352 463 553 788
768 316 431 574 345 749 466 580 395 493

beginning of the first interval equal 300, and the width of each interval
equal 50.
2) Construct the histogram and the polygon of relative frequencies of
distribution.
3) Find the mode and the median (using the discrete series).
4) Find empirical functions of distribution of continuous and discrete
variation series; and construct their graphs.

234
Variant 20

Variant 20
1. The probability of hit in a target by the first shooter is 0,8, and by the
second shooter – 0,7. The shooters have shot simultaneously. What is
the probability that only one of them hit in the target?
2. 1100 of 2000 tubes belong to the first batch, 650 – to the second, and
250 – to the third. 7% of the first batch, 6% of the second and 8% of the
third are defective tubes. One tube is chosen at random. Determine the
probability that the chosen tube is defective.
3. It has been established by long observations that 14 days at April
are on the average rainy in Chicago. What is the probability that 5 of 7
randomly chosen days of the month will be rainy?
4. Let X be a discrete random variable distributed under the following
law:

X 0 1 2 3 4 5
р 0,14 0,18 0,06 0,3 0,04 0,28
Find: a) the mathematical expectation and the dispersion of the random
variable X; b) the probability of the following event: 1 < X ≤ 3.
5. A random variable X is given by the density of distribution
f(x) = A(3x – x2) in the interval (0; 2), and f (x) = 0 outside of the interval.
Find:
а) parameter A;
b) the mathematical expectation and the dispersion of the variable X.
6. An item made by an automatic device is recognized suitable if
the deviation of its controllable weight from the design one does not
exceed 15 g. Random deviations of the controllable weight from the
design one are subordinated to a normal law with dispersion 16 and
mathematical expectation 20 g. How many percent of suitable items
does the automatic device produce?
7. Estimate the probability that the absolute value of the deviation of the
average height of 500 persons from the mathematical expectation of a

235
INDIVIDUAL HOMEWORKS

random variable expressing the height of each person will not exceed
1 cm, assuming that the dispersion of each of these random variables
does not exceed 7.
8. The following data on strength of a metal cable of a given batch have
been received after testing 60 samples:
1) Compose the interval and the discrete variation series taking the
709 707 738 735 726 738 702 734 774 785
761 787 757 769 728 755 736 749 725 736
768 759 736 782 716 766 741 752 754 753
774 777 713 759 753 743 746 761 765 755
771 732 748 766 759 739 749 751 731 768
713 717 752 795 792 751 765 746 775 773

beginning of the first interval equal 700, and the width of each interval
equal 10.
2) Construct the histogram and the polygon of relative frequencies of
distribution.
3) Find the mode and the median (using the discrete series).
4) Find empirical functions of distribution of continuous and discrete
variation series; and construct their graphs.

236
Variant 21

Variant 21
1. Participants of a toss-up pull tickets with numbers from 1 up to 110
from a box. Find the probability that the number of the first randomly
taken ticket does not contain the digit 6 (toss-up – жеребьевка; ticket
– жетон).
2. Products are delivered in a shop from two factories: 60% of them
from the first factory and 40% – from the second. 15% of products
made by the first factory are defective, and 19% of products made by
the second factory are defective. Find the probability that a product
bought in the shop will not be defective.
3. Let 6% of made products do not satisfy the standard in a given
technological process. Determine the probability that among 120
randomly chosen products: a) exactly 113 products will satisfy the
standard; b) from 98 up to 116 products will satisfy the standard.
4. Two independent random variables X and Y are given by the following
tables of distribution:

X –1 3 Y –2 0 1
P 0,6 0,4 P 0,3 0,1 0,6
Compose the table of distribution of the random variable X – Y and
check the property: D(X – Y) = D(X) + D(Y).
5. A random variable X has the following density of distribution:

F (x) = *Cx
0 if x # 1,
4
if 1 < x # 2,
0 if x > 2.
Find: а) the parameter C and the distribution function F(x); b) the
probability of hit of the random variable X into the interval (1,5; 4).
6. Results of measuring the distance between two cities are
subordinated to a normal law with mathematical expectation 200 km
and dispersion 16. Find the probability that the distance between these

237
INDIVIDUAL HOMEWORKS

points is: a) no less than 190 km; b) no more than 205 km; c) from 195
up to 203 km.
7. The dispersion of each of pairwise independent random variables
does not exceed 15. It is required to determine how many such random
variables should take in order to assert with probability no less than 0,95
that the absolute value of the deviation of the arithmetic mean of these
variables from the arithmetic mean of their mathematical expectations
will not exceed 0,4.
8. There are the following data on size of annual charges of 50 workers

115 131 132 174 192 149 135 155 147 174
143 147 163 149 117 173 132 112 146 146
123 124 162 152 102 125 122 172 108 136
144 151 166 152 136 104 124 144 182 134
150 148 142 152 142 148 198 168 188 168

of an enterprise on communication services:


1) Compose the interval and the discrete variation series taking the
beginning of the first interval equal 100, and the width of each interval
equal 10.
2) Construct the histogram and the polygon of relative frequencies of
distribution.
3) Find the mode and the median (using the discrete series).
4) Find empirical functions of distribution of continuous and discrete
variation series; and construct their graphs.

238
Variant 22

Variant 22
1. A student knows 25 of 30 questions on the first part and 40 of 50 on
the second part of a course. He or she is offered at random one question
from each part of the course. Determine the probability that the student
will answer correctly: а) only one question; b) at least one.
2. There are 6 green and 9 yellow balls in the first urn, and 4 green and
8 yellow balls in the second urn. A randomly chosen ball from the first
urn placed in the second urn, and after that one ball has taken out at
random from the second urn. Determine the probability that the ball is
green.
3. Products are delivered on a warehouse, and 70 % from which have
the first grade. Find the probability that no more than 110 products of
randomly taken 150 products will have the first grade.
4. Discrete independent random variables X and Y are given by the laws
of distribution:

X 2 3 Y 3 4
P 0,6 0,4 P 0,8 0,2

Find M(X · Y) by two ways: 1) composing the law of distribution


of X · Y; 2) using the property of mathematical expectation
M(X · Y) = M(X) · M(Y).
5. Compose the law of distribution of the number of hits in a target X at
six shots if the probability of hit at one shot is 0,8. Find the mathematical
expectation and the dispersion of X.
6. Let the weight of a peach (in grammes) is a random variable X of
which the density of distribution is described by the following function:
1 (x - 160)2
f (x) = e- 800 .
20 2r
Determine the probability that the weight X of a randomly taken peach
from the given batch will be in limits from 150 up to 180.

239
INDIVIDUAL HOMEWORKS

7. There are electric bulbs made by two factories on a warehouse. 70 %


of them have been made by the first factory, and 30 % – by the second
factory. Estimate the probability that the part of electric bulbs of the
first factory among randomly selected 200000 will differ from the
probability of making an electric bulb by the first factory no more than
on 0,005 by absolute value.
8. The following data on lifetime (in hours) of a separate radio lamp
have been obtained at inspecting 80 radio lamps of a given set:

730 780 705 402 859 1258 435 1085 656 1179
989 956 69 849 898 1001 1233 1110 681 770
625 600 191 1305 1195 703 667 379 731 474
899 685 1209 887 268 735 267 833 960 332
967 650 87 264 807 159 76 573 1075 875
631 995 135 1158 432 580 370 930 479 1067
558 1095 985 841 897 940 901 376 571 973
196 470 609 890 469 889 373 460 1196 612

1) Compose the interval and the discrete variation series taking


the beginning of the first interval equal 0, and the width of each interval
equal 100.
2) Construct the histogram and the polygon of relative frequencies of
distribution.
3) Find the mode and the median (using the discrete series).
4) Find empirical functions of distribution of continuous and discrete
variation series; and construct their graphs.

240
Variant 23

Variant 23
1. There are 11 green and 7 red balls in a box. Two balls are taken out at
random. What is the probability that these balls will be the same color?
2. A warehouse has received a big batch of goods from two factories,
and 70% of them have been delivered from the first factory. It is known
that 75% goods from the first factory and 65% goods from the second
have the higher grade. What is the probability that a randomly taken
unit of goods in the warehouse has the higher grade?
3. The probability of hit in a target at one shot by a shooter is equal
to 0,8. Determine the most probable number of hits in the target
at 25 shots and calculate the probability of this number.
4. Two independent random variables are given by the following tables
of distribution:

X –1 0 2 Y 1 2
P 0,4 0,3 0,3 P 0,4 0,6
Compose the law of distribution of the random variable X · Y and check
the property: M(X · Y) = M(X) · M(Y).
C
5. A random variable X has the density of distribution f (x) =
1 - x4
in the interval (0; 1), and f(x) = 0 outside of the interval. Find:
a) the parameter C and the distribution function F(x);
b) the probability that the following inequality holds: 1/2 < X < 3 /2 .
6. The height of an adult man is a random variable distributed under a
normal law with parameters: a = 175 cm, σ = 6 cm. Find the density
of probability and the probability that the height of a randomly chosen
man is in the interval from 170 up to 180 cm.
7. The dispersion of each of 4000 independent random variables does
not exceed 16. Estimate the probability that the absolute value of the
deviation of the arithmetic mean of these variables from the arithmetic

241
INDIVIDUAL HOMEWORKS

mean of their mathematical expectations will not exceed 0,8.


8. The following data on lifetime (in hours) of a separate vacuum cleaner
have been obtained at inspecting 100 vacuum cleaners of a given set:
1) Compose the interval and the discrete variation series taking the
548 956 521 1220 575 1121 1039 989 223 403
630 680 605 302 759 1158 335 985 556 1079
889 856 969 749 798 901 1133 1010 581 670
525 500 91 1205 1095 603 567 279 631 374
799 585 1209 887 268 735 267 833 960 332
967 650 86 264 807 159 78 573 1075 875
631 995 135 1158 432 580 370 930 479 1067
558 1095 985 841 897 940 901 376 570 970
196 470 609 890 469 889 373 459 1195 611
699 611 339 154 875 988 1169 531 957 551

beginning of the first interval equal 0, and the width of each interval
equal 200.
2) Construct the histogram and the polygon of relative frequencies of
distribution.
3) Find the mode and the median (using the discrete series).
4) Find empirical functions of distribution of continuous and discrete
variation series; and construct their graphs.

242
Variant 24

Variant 24
1. A batch of footwear in amount of 100 pairs packed into identical
boxes has been delivered in a shop. 70 pairs of them are black color,
and the others – red color. A shop assistant has been selected at random
two boxes from the batch. Find the probability that both selected pairs
of footwear will be: a) different colors; b) red color.
2. Two shooters shoot simultaneously in a target. The probability of
hit in the target by the first shooter is 0,6, and by the second – 0,7. The
target has been struck once. Find the probability that the target has been
struck by the second shooter.
3. The probability that a subscriber will call on the switchboard within
one hour is equal to 0,02. The telephone station serves 900 subscribers.
What is the probability that 4 subscribers will call within one hour?
4. Two independent random variables X and Y are given by the following
tables of distribution:
X 3 5 Y 0 3 6
P 0,6 0,4 P 0,7 0,2 0,1

Compose the table of distribution of the random variable X + Y and


check the property: M(X + Y) = M(X) + M(Y).
5. A random variable X is given by the distribution function:

F (x) = * x2 /9
0 if x # 0,
if 0 < x # 3,
1 if x > 3.
Find: a) the density of probability f(x); b) the mathematical expectation
and the dispersion of X; c) the probability of hit of the random variable
into the interval (1; 2,5).
6. The mathematical expectation and the dispersion of a normally
distributed random variable X are 25 and 9 respectively. Find the
probability that: a) the random variable X will take on a value from

243
INDIVIDUAL HOMEWORKS

the interval (20; 28); b) the difference X – M(X) will be less than 2 by
absolute value.
7. The width of produced items represents a random variable with
mathematical expectation equal 6 cm and mean square deviation equal
1 mm. Estimate the probability that the deviation of the width of a
produced item from its mathematical expectation will not exceed 3 mm
by absolute value.
8. There are the following data on size of annual charges of 100
consumers on transport services:
315 331 332 374 392 349 335 355 347 374
343 347 363 349 317 373 332 312 346 346
323 324 362 352 302 325 322 372 308 336
344 351 366 352 336 304 324 344 382 334
350 348 342 352 342 348 398 368 388 368
381 344 372 366 334 342 338 321 338 396
324 332 325 338 398 389 324 342 345 355
314 343 298 362 375 340 377 358 346 322
357 305 383 312 351 367 303 332 316 352
343 331 355 318 340 342 392 349 355 314
1) Compose the interval and the discrete variation series taking the
beginning of the first interval equal 290, and the width of each interval
equal 10.
2) Construct the histogram and the polygon of relative frequencies of
distribution.
3) Find the mode and the median (using the discrete series).
4) Find empirical functions of distribution of continuous and discrete
variation series; and construct their graphs.

244
Variant 25

Variant 25
1. There are 6 green and 9 yellow balls in the first urn, and 9 green and
6 yellow balls in the second urn. One ball is randomly taken out from
each urn. Find the probability that at least one of them is green.
2. A warehouse has received two equal batches of footwear in identical
packing boxes. It is known that 50 % of footwear in the first batch and
80 % of footwear in the second batch have a black color. What is the
probability that a randomly taken pair of footwear in the warehouse will
have a black color?
3. An automatic device is making plastic hangers. The probability that
no non-standard hanger will be made for one hour is equal to 0,8. Find
the probability that all hangers made for four hours will be standard
(hanger – вешалка).
4. Two independent random variables X and Y are given by the following
tables of distribution:
X 2 3 4 Y –3 –2 –1
P 0,7 0,2 0,1 P 0,3 0,5 0,2

Compose the law of distribution of the random variable Z = X + Y and


check the property: M(X + Y) = M(X) + M(Y).

5. A random variable X is given by the probability density of distribution


f (x) = cos x in the interval `- r ; r j , and f (x) = 0 outside of the
1
2 2 2
interval. Find the dispersion of the random variable X.
6. Let the lifetime of a radio tube of a given batch be a normally
distributed random variable X with mathematical expectation a = 800
hours and mean square deviation σ = 30 hours. Determine the probability
that the lifetime X of a randomly chosen tube from the batch: a) is in
limits from α = 830 hours up to β = 950 hours; b) deviates from the
mathematical expectation no more than on 20 hours by absolute value.
7. The discrete random variable X is given by the law of distribution:

245
INDIVIDUAL HOMEWORKS

X 2 3 4
P 0,3 0,4 0,3
By using the Chebyshev’s inequality estimate the probability that the
random variable will deviate from the mathematical expectation no
more than on 2,5 by absolute value.
8. The following data on strength of a steel wire of a given batch have
been received after testing 100 samples:
453 407 438 435 426 438 402 434 474 485
468 487 457 469 428 455 436 449 425 436
468 459 436 482 416 466 441 452 454 453
474 477 413 459 453 443 446 461 465 455
471 432 448 466 459 439 449 451 431 468
413 417 452 495 492 451 465 446 475 473
442 452 449 479 448 498 449 432 463 455
462 455 462 443 465 445 478 442 422 443
469 445 446 455 445 463 451 468 479 463
452 469 457 454 468 452 477 469 453 457
1) Compose the interval and the discrete variation series taking the
beginning of the first interval equal 400, and the width of each interval
equal 10.
2) Construct the histogram and the polygon of relative frequencies of
distribution.
3) Find the mode and the median (using the discrete series).
4) Find empirical functions of distribution of continuous and discrete
variation series; and construct their graphs.

246
TESTS FOR SELF-CHECKING
1. Let A and B be events connected with the same trial. Show the event
that means occurrence of at least one of the events A and B.
A) AB̅ ; B) А̄В; C) A + B; D) AB̅ + A̅ B + AB;
E) There is no right answer.
2. Let А1, А2, А3 be events connected with the same trial. Let A be the
event that means occurrence of exactly two of the events А1, А2 and А3.
Express the event A by the events А1, А2 and А3.
A) A = А1 А2 А3; B) A = А1А2 + А2А3 + А1 А3;
C) A = А2А3; D) A = А1А3;
E) A = А̅ 1А2A3 + A1А̅ 2А3 + А1 A2 А̅ 3.
3. A coin is tossed twice. Find the probability that the coin lands on
«tails» in both times.
A) 1/2; B) 3/4; C) 1; D) 1/4; E) There is no right answer.
4. Two dice are tossed. Find the probability that the product of aces does
not exceed 5.
A) 0,5; B) 5/6; C) 1/6; D) 5/36; E) 5/18.
5. There are 10 white, 15 black, 20 blue and 25 red balls in an urn. One
ball is randomly extracted. Find the probability that the extracted ball
is blue or red.
A) 5/14; B) 1/70; C) 1/7; D) 9/14; E) 3/98.
6. There are 10 white, 15 black, 20 blue and 25 red balls in an urn. One
ball is randomly extracted. Find the probability that the extracted ball is
either white, either black or blue.
A) 5/14; B) 1/70; C) 1/7; D) 9/14; E) 3/98.

247
TESTS FOR SELF-CHECKING

7. One letter is randomly chosen from the word “ECONOMIST”. What


is the probability that this letter is “A”?
A) 0,6; B) 1/6; C) 1/2; D) 0; E) There is no right answer.
8. There are 5 white and 5 black balls in an urn. What is the probability
of extracting a red ball from the urn?
A) 1; B) 0; C) 1/2; D) 1/5; E) 1/3.
9. There are 10 white, 4 black, 12 yellow and 15 red balls in a box. Find
the probability that a randomly taken ball will be black.
A) 1/13; B) 4/41; C) 5/14; D) 3/41; E) 1/14.
10. There are 10 white, 15 black, 10 yellow and 25 red balls in a box.
Find the probability that a randomly taken ball will be white.
A) 2/3; B) 1/7; C) 3/14; D) 1/6; E) 1/14.
11. Three coins are tossed. What is the probability that at least one of
the coins lands on «heads»?
A) 1/8; B) 7/8; C) 1/4; D) 3/4; E) 1/3.
12. Three shooters shoot in a target. The probability of hit in the target at
one shot for the 1st shooter is 0,75; for the 2nd – 0,8 and for the 3rd – 0,2.
Find the probability that at least one of the shooters will hit in the target.
A) 0,995; B) 0,96; C) 0,95; D) 0,65; E) 0,504.
13. Two dice are tossed. Find the probability that the sum of aces is
divided on 5.
A) 1/7; B) 1/4; C) 7/36; D) 1/6; E) There is no right answer.
14. The probability of delay for the train № 1 is equal to 0,2, and for the
train № 2 – 0,64. Find the probability that only one train will be late.
A) 0,615; B) 0,584; C) 0,715; D) 0,9; E) There is no right answer.
15. The events A and B are independent, Р(А) = 0,3; Р(В) = 0,1. Find .
A) P(A̅ · B̅ ) = 0,63; B) P(A̅ · B̅ ) = 0,27; C) P(A̅ · B̅ ) = 0,07;

248
D) P(A̅ · B̅ ) = 0,04; E) There is no right answer.
16. The probability of distorting a sign at transmitting a message is
equal to 0,01. Find the probability that a transmitted message consisting
of 10 signs contains exactly 3 distortions (distortion – искажение).
A) C 10
3
(0,01)3; B) C 710 (0,01)7 (0,99)3;
C) 0,013; D) C 10
3
(0,01)3 (0,99)7;
E) There is no right answer.
17. Two shots are made in a target by two guns. The probability of hit
from the first gun is 0,8, from the second gun – 0,9. Find the probability
of only one hit in the target.
A) 0,2; B) 0,17; C) 0,26; D) 0,72; E) 0,5.
18. The law of distribution of a discrete random variable X is given by
the following table:

X –1 Y
P 0.4 0.6

It is known that M (X) = 2,6. Find Y.


A) 1; B) 2; C) 3; D) 4; E) 5.
19. A discrete random variable X is given by the following law of
distribution:

X -5 0 5
P 0,1 0,6 0,2

Find the mathematical expectation M(X).


A) 0,5; B) 2,5; C) 1; D) 10; E) 3.
20. A discrete random variable X is given by the following law of
distribution:
249
TESTS FOR SELF-CHECKING

X x1 x2 x3 x4
P 0,2 0,1 p3 0.3
Find p3.
A) 0,5; B) 0,2; C) 0; D) 1; E) 0,4.
21. The dispersion of the number of occurrences of an event A for n
independent trials in each of which the probability of occurrence of the
event A is p, is equal to:
A) D(X) = np; B) D(X) = nq; C) D (X) = npq ; D) D(X) = npq;
E) There is no right answer.
22. The random variables X and Y are independent. Find the dispersion
of the random variable Z = 3 · X + 4 · Y if it is known that D(X) = 2,
D(Y) = 6.
A) 30; B) 12; C) 78; D) 114; E) 110.
23. If F(x) is an integral function of distribution of a random variable,
X ∈ (a, b) then for x > b F(x) is equal to:
A) 1; B) 0; C) 0,5; D) – 1; E) 2.
24. If all possible values of a random variable X belong to an interval
(a, b) then the integral function of distribution F(x) for x ≤ a is equal to:
A) 1; B) 0; C) 0,5; D) – 1; E) 2.
25. The graph of the density of distribution f(x) of a continuous random
variable X cannot be:
A) below the axis Ox; B) upper the axis Ox;
C) to the left of the axis Оу; D) to the right of the axis Оу;
E) There is no right answer.
26. If a continuous random variable X is uniformly distributed over

250
(a, b), then the density has the following form:

A) f (x) = )0, x g (a, b)


B) f (x) = )0, - mx x < a , x > b
1/ (b - a), x ! (a, b) me , a < x < b

C) f (x) = )0, x ! (a, b)


D) f (x) = )0, x g (a, b)
1/ (b - a), x g (a, b) b - a, x ! (a, b)

Е) f (x) = )1/ (b - a), x < a , x > b


0, a<x<b

27. A random variable X has the following density of distribution:

e- 18 .
1 ( x + 3) 2

f (x) =
3 2r

Find the mathematical expectation M(X) and the dispersion D(X).


A) M(X) = – 3, D(X) = 3; B) M(X) = 3, D(X) = 3;
C) M(X) = – 3, D(X) = 9; D) M(X) = 0, D(X) = 1;
E) There is no right answer.
28. Write the density of probability of a normally distributed random
variable X if M(X) = 5, D(X) = 16.

A) f (x) = 1 ( x + 3) 2
1 2
(x - 5)
e- 18 ; B) f (x) = e- 32 ;
3 2r 4 2r

C) f (x) = 1 (x + 5) 2
1 2
(x + 5)
e- 8 ; D) f (x) = e- 16 ;
3 2r 3 2r

E) f (x) = 1 2

e- 16 .
(x - 4)

5 2r

251
TESTS FOR SELF-CHECKING

29. An urn contains 5 red, 3 white, and 4 blue balls. What is the
probability of extracting a black ball from the urn?
A) 1/3; B) 0; C) 0,25; D) 0,5; E) 5/12.
30. An urn contains 3 white and 7 black balls. Two balls are randomly
extracted from the urn. What is the probability that both balls are white?
A) 6/15; B) 7/15; C) 7/10; D) 8/15; E) 1/15.
31. In a batch of 5 products 2 of them have the first grade, and 3 – the
second grade. Two products are randomly taken. Find the probability
that both products have the second grade.
A) 0,6; B) 0,3; C) 0,1; D) 0,5; E) 0,4.
32. In a batch of 5 products 2 of them have the first grade, and 3 – the
second grade. Two products are randomly taken. Find the probability
that they have different grades.
A) 0,6; B) 0,3; C) 0,1; D) 0,5; E) 0,4.
33. Two shooters simultaneously shoot in a target. The probability of hit
in the target by the first shooter is 0,7, and by the second – 0,8. Find the
probability that none of shooters will hit in the target.
A) 0,38; B) 0,94; C) 0,06; D) 0,56; E) 0,96.
34. A coin is tossed 6 times. Find the probability that the coin lands on
heads only once.
A) 7/64; B) 15/64; C) 57/64; D) 21/32; E) 3/32.
35. A coin is tossed 6 times. Find the probability that the coin lands on
heads 6 times.
A) 7/64; B) 15/64; C) 1/64; D) 21/32; E) 3/32.
36. A coming up seeds of wheat makes 80 %. Find the probability that
two of five sown seeds will come up.
A) 0,496; B) 0,0064; C) 0,0512; D) 0,00032; E) 0,2048.

252
37. A coming up seeds of wheat makes 80 %. Find the probability that
none of five sown seeds will come up.
A) 0,00032; B) 0,0064; C) 0,0512; D) 0,496; E) 0,2048.
38. During the year two firms have an opportunity independently from
each other to go bankrupt with probabilities 0,05 and 0,1. Find the
probability that at the end of the year both firms will function.
A) 0,64; B) 0,855; C) 0,45; D) 0,38; E) 0,256.
39. A shooter makes 3 shots in a target. The probability of hit in the
target at one shot is equal to 0,7. Find the probability that at least one
shot will hit in the target.
A) 0,973; B) 0,027; C) 0,316; D) 0,36; E) 0,3.
40. Find the mathematical expectation M(X) of a random variable X,
knowing its law of distribution:
xi 6 3 1
pi 0,2 0,3 0,5

A) 3,4; B) 2,8; С) 2,6; D) 2,4; E) 0,76.


41. Find the dispersion D(X) of a random variable X, knowing its law
of distribution:

xi 1 2 3
pi 0,3 0,2 0,5
А) 0,76; B) 2,6; С) 2,8; D) 2,4; E) 3,4.
42. An integral function of distribution F(х) is:
A) decreasing; B) periodic; C) even; D) non-decreasing; E) odd.
43. A random variable X is given by the integral function of distribution:

253
TESTS FOR SELF-CHECKING

F (x) = * x
0 if x # 0,
if 0 < x # 1,
1 if x > 1.

Find the mathematical expectation M(X).


A) 1/3; B) 1; C) 2/3; D) 0,5; E) 0.
44. A random variable X is given by the integral function of distribution:

F (x) = * x
0 if x # 0,
if 0 < x # 1,
1 if x > 1.
Find the dispersion D(X).
A) 1/3; B) 1; C) 2/3; D) 0,5; E) 1/12.
45. A random variable X is given by the integral function of distribution:
Z
]]0 if x # - 1,
1 1
F (x) = [ x + if - 1 < x # 3,
]] 4 4
\1 if x > 3.
Calculate the probability of hit of the random variable X in the interval
(0; 2).
A) 0,5; B) 0; C) 1/4; D) 1; E) 2/3.
46. A random variable X has the following law of distribution:

xi 0 1 2 3
pi 1/30 3/10 ½ 1/6

Find the mathematical expectation of X.


A) 1; B) 1,5; C) 2; D) 1,8; E) 2,3.

254
47. Let a random variable X be given by the following law of distribution:

xi –3 –2 0 2 3
pi 0,1 0,2 0,4 0,2 0,1

Find the mathematical expectation M(X).


A) 2; B) 0,1; C) 0,2; D) 0; E) 1
48. Let a random variable X be given by the following law of distribution:

xi -3 -2 0 2 3
pi 0,1 0,2 0,4 0,2 0,1

Find the dispersion D(X).


A) 0; B) 3,4; C) 1; D) 0,34; E) 0,1.
49. A random variable X is given by the integral function of distribution:
Z
]]0 if x # 2,
1
F (x) = [ x - 1 if 2 < x # 4,
]] 2
\1 if x > 4.

Find the probability of hit of the random variable X into the interval
(2; 3).
A) 0,25; B) 0,5; C) 1/3; D) 2/3; E) 1.
50. Show one of the properties of mathematical expectation (C is a
constant):
A) M(CX) = M(C) + M(X); B) M(C + X) = M(X);
C) M(CX) = CM(X); D) M(CX) = M(C) – M(X);
E) M(C + X) = M(C)

255
TESTS FOR SELF-CHECKING

51. The sample is given by the following table:

xi 2 5 7 10
ni 16 12 8 14
Find the sample mean x̅ .
A) 5; B) 6; C) 5,76; D) 4,65; E) There is no right answer.
52. The sample has been taken from a parent population:

xi 1 3 6 26
ni 8 40 10 2
Find the unbiased estimator of the parent mean (the sample mean x̅ ).
A) 18; B) 4; C) 3: D) 6; E) 30.
53. The sample is given by the following table:

xi –20 0 15 20
ni 20 50 18 12

Find the sample dispersion s2.


A)167,29; B) 152,25; C)145,5; D)182,35; E)175,43.
54. The sample is given by the following table:

xi 0,5 –2 5
ni 2 5 4
Find the unbiased estimator of the dispersion s2 (the corrected
dispersion).
A) 5; B) 6; C) 10,95; D) 4,65; E) 1.

256
54. The unbiased estimator of the dispersion found on a sample of
volume n = 41 is s|2 = 3. Find the displaced estimation of the dispersion
s 2.
A) 4,05; B) 7,8; C) 5,3; D) 2,93; E) 20.
55. Find the mode of the following variation series:
xi 1 3 7 9
ni 5 2 10 6

A) 10; B) 11,5; C) 7; D) 3; E) 1.
56. The distribution of an attribute X – the number of transactions at
stock exchange for a quarter received on n = 400 observations is given:

xi 0 1 2 3 4 5 6 7 8 9 10
ni 146 97 73 34 23 10 8 3 4 2 2

Find the arithmetic mean .


A) x̅ = 0,634; B) x̅ = 1,535; C) x̅ = 2,5; D) x̅ = 3,864; E) x̅ = 4,365.
57. Find the mode of the following sample:
{10, 10, 15, 16, 16, 16, 16, 16}.
A) 16; B) 10; C) 20; D) 5; E) 125.
58. Find the median of the following sample:
{10, 10, 15, 16, 16, 16, 16}.
A) 16; B) 10; C) 25; D) 15; E) 148.
59. Results of sampling inspections of one-day profits of city shops are
given:
{5, 5, 10, 14, 18, 20, 25}.

257
TESTS FOR SELF-CHECKING

Find the median.


A) 30; B) 10; C) 25; D) 14; E) 148.
60. How will the arithmetic mean change if all variants are increased
in 5 times?
A) will increase in 5 times; B) will decrease in 5 times;
C) will increase in 25 times; D) will decrease in 25 times;
E) will not change.
61. How will the sample dispersion change if all variants are increased
on 6?
A) will decrease on 36; B) will increase on 6;
C) will increase in 6 times; D) will decrease in 6 times;
E) will not change.
62. How will the length of confidence interval change if the volume of
a sample will increase in 9 times?
A) will decrease in 3 times; B) will increase in 3 times;
C) will decrease in 9 times; D) will increase in 9 times;
E) will not change.
63. A discrete variation series of a sample is given:
Find the volume of the variation series:

xi –2 1 4
ni 4 16 5

A) 3; B) 25; C) 16; D) 4; E) 1.
64. Find the confidence interval for an estimation with reliability
γ = 0,95 of the parent mean x̅ 0 of a normally distributed attribute X if

258
the parent mean square deviation σ = 9, the sample mean x̅ = 18,31,
the volume of a sample n = 49. It has been found on the table: t = 1,96.
A) (15,79; 20,83); B) (17,138; 19,882);
C) (16,23; 18,65); D) (14,83; 17,12);
E) (15,8; 21,02).
65. What is the mode of a normal distribution with mathematical
expectation 0 and dispersion 1?
A) е; B) 0; C) 1/2; D) –1/2; E) 1.
66. A discrete variation series is given:

xi –5 1 3
ni 2 5 3
Find the arithmetic mean.
A) 3; B) 4; C) 0,4; D) 3,5; E) 2.
67. The sample is given as the following distribution of frequencies:

xi 2 5 7
ni 1 3 6
Find the sample mean.
A) 5,9; B) 4; C) 3; D) 4,5; E) 3,5.
68. The variation series of a sample is given:

xi 2 4 5 6
ni 8 9 10 3

Find the sample dispersion s2.


A) 2; B) 1,8; C) 4; D) 5; E) 120.
259
TESTS FOR SELF-CHECKING

69. A random variable X is distributed under a Poisson law. Find by the


method of moments a pointwise estimator of unknown parameter λ of
the Poisson distribution on the following sample:

xi 0 1 2 3 4 5 6
ni 405 366 175 40 8 4 2

A) 2,5; B) 4; C) 3,5; D)0,59; E) 0,9.


70. A random variable X is distributed under a normal law. Find by
the method of maximal plausibility a pointwise estimator of unknown
parameter a of the normal distribution on the following sample:

xi 1 4 8
ni 5 3 2
A) 3,3; B) 4; C) 0,5; D) 0,59; E) 0,9.

260
Appendix 1

APPENDICES

Appendix 1
x
1 1
1 #e
2

Values the functions { (x) = $ e- 2 x and U (x) = - 1 x2


2 dt
2r 2r 0

x φ(x) Φ(x) x φ(x) Φ(x) x φ(x) Φ(x)


0,00 0,3989 0,0000 0,36 0,3739 0,1406 0,72 0,3079 0,2642
0,01 0,3989 0,0040 0,37 0,3726 0,1443 0,73 0,3056 0,2673
0,02 0,3989 0,0080 0,38 0,3712 0,1480 0,74 0,3034 0,2703
0,03 0,3988 0,0120 0,39 0,3697 0,1516 0,75 0,3011 0,2734
0,04 0,3986 0,0160 0,40 0,3683 0,1554 0,76 0,2989 0,2764
0,05 0,3984 0,0199 0,41 0,3668 0,1591 0,77 0,2966 0,2794
0,06 0,3982 0,0239 0,42 0,3652 0,1628 0,78 0,2943 0,2823
0,07 0,3980 0,0279 0,43 0,3637 0,1664 0,79 0,2920 0,2852
0,08 0,3977 0,0319 0,44 0,3621 0,1700 0,80 0,2897 0,2881
0,09 0,3973 0,0359 0,45 0,3605 0,1736 0,81 0,2874 0,2910
0,10 0,3970 0,0398 0,46 0,3589 0,1772 0,82 0,2850 0,2939
0,11 0,3965 0,0438 0,47 0,3572 0,1808 0,83 0,2827 0,2967
0,12 0,3961 0,0478 0,48 0,3555 0,1844 0,84 0,2803 0,2995
0,13 0,3956 0,0517 0,49 0,3538 0,1879 0,85 0,2780 0,3023
0,14 0,3951 0,0557 0,50 0,3521 0,1915 0,86 0,2756 0,3051
0,15 0,3945 0,0596 0,51 0,3503 0,1950 0,87 0,2732 0,3078
0,16 0,3939 0,0636 0,52 0,3485 0,1985 0,88 0,2709 0,3106
0,17 0,3932 0,0675 0,53 0,3467 0,2019 0,89 0,2685 0,3133
0,18 0,3925 0,0714 0,54 0,3448 0,2054 0,90 0,2661 0,3159
0,19 0,3918 0,0753 0,55 0,3429 0,2088 0,91 0,2637 0,3186
0,20 0,3910 0,0793 0,56 0,3410 0,2123 0,92 0,2613 0,3212
0,21 0,3902 0,0832 0,57 0,3391 0,2157 0,93 0,2589 0,3238
0,22 0,3894 0,0871 0,58 0,3372 0,2190 0,94 0,2565 0,3264
0,23 0,3885 0,0910 0,59 0,3352 0,2224 0,95 0,2541 0,3289
0,24 0,3876 0,0948 0,60 0,3332 0,2257 0,96 0,2516 0,3315
0,25 0,3867 0,0987 0,61 0,3312 0,2291 0,97 0,2492 0,3340

261
x φ(x) Φ(x) x φ(x) Φ(x) x φ(x) Φ(x)
0,26 0,3857 0,1026 0,62 0,3292 0,2324 0,98 0,2468 0,3365
0,27 0,3847 0,1064 0,63 0,3271 0,2357 0,99 0,2444 0,3389
0,28 0,3836 0,1103 0,64 0,3251 0,2389 1,00 0,2420 0,3413
0,29 0,3825 0,1141 0,65 0,3230 0,2422 1,01 0,2396 0,3438
0,30 0,3814 0,1179 0,66 0,3209 0,2454 1,02 0,2371 0,3461
0,31 0,3802 0,1217 0,67 0,3187 0,2486 1,03 0,2347 0,3485
0,32 0,3790 0,1255 0,68 0,3166 0,2517 1,04 0,2323 0,3508
0,33 0,3778 0,1293 0,69 0,3144 0,2549 1,05 0,2299 0,3531
0,34 0,3765 0,1331 0,70 0,3123 0,2580 1,06 0,2275 0,3554
0,35 0,3752 0,1368 0,71 0,3101 0,2611 1,07 0,2251 0,3577
1,08 0,2227 0,3599 1,49 0,1315 0,4319 1,90 0,0656 0,4713
1,09 0,2203 0,3621 1,50 0,1295 0,4332 1,91 0,0644 0,4719
1,10 0,2179 0,3643 1,51 0,1276 0,4345 1,92 0,0632 0,4726
1,11 0,2155 0,3665 1,52 0,1257 0,4357 1,93 0,0620 0,4732
1,12 0,2131 0,3686 1,53 0,1238 0,4370 1,94 0,0608 0,4738
1,13 0,2107 0,3708 1,54 0,1219 0,4382 1,95 0,0596 0,4744
1,14 0,2083 0,3729 1,55 0,1200 0,4394 1,96 0,0584 0,4750
1,15 0,2059 0,3749 1,56 0,1182 0,4406 1,97 0,0573 0,4756
1,16 0,2036 0,3770 1,57 0,1163 0,4418 1,98 0,0562 0,4761
1,17 ,2012 0,3790 1,58 0,1145 0,4429 1,99 0,0551 0,4767
1,18 0,1989 0,3810 1,59 0,1127 0,4441 2,00 0,0540 0,4772
1,19 0,1965 0,3830 1,60 0,1109 0,4452 2,02 0,0519 0,4783
1,20 0,1942 0,3849 1,61 0,1092 0,4463 2,04 0,0498 0,4793
1,21 0,1919 0,3869 1,62 0,1074 0,4474 2,06 0,0478 0,4803
1,22 0,1895 0,3883 1,63 0,1057 0,4484 2,08 0,0459 0,4812
1,23 0,1872 0,3907 1,64 0,1040 0,4495 2,10 0,0440 0,4821
1,24 0,1849 0,3925 1,65 0,1023 0,4505 2,12 0,0422 0,4830
1,25 0,1826 0,3944 1,66 0,1006 0,4515 2,14 0,0404 0,4838
1,26 0,1804 0,3962 1,67 0,0989 0,4525 2,16 0,0387 0,4846
1,27 0,1781 0,3980 1,68 0,0973 0,4535 2,18 0,0371 0,4854
1,28 0,1758 0,3997 1,69 0,0957 0,4545 2,20 0,0355 0,4861
1,29 0,1736 0,4015 1,70 0,0940 0,4554 2,22 0,0339 0,4868

262
Appendix 1

x φ(x) Φ(x) x φ(x) Φ(x) x φ(x) Φ(x)


1,30 0,1714 0,4032 1,71 0,0925 0,4564 2,24 0,0325 0,4875
1,31 0,1691 0,4049 1,72 0,0909 0,4573 2,26 0,0310 0,4881
1,32 0,1669 0,4066 1,73 0,0893 0,4582 2,28 0,0297 0,4887
1,33 0,1647 0,4082 1,74 0,0878 0,4591 2,30 0,0283 0,4893
1,34 0,1626 0,4099 1,75 0,0863 0,4599 2,32 0,0270 0,4898
1,35 0,1604 0,4115 1,76 0,0848 0,4608 2,34 0,0258 0,4904
1,36 0,1582 0,4131 1,77 0,0833 0,4616 2,36 0,0246 0,4909
1,37 0,1561 0,4147 1,78 0,0818 0,4625 2,38 0,0235 0,4913
1,38 0,1539 0,4162 1,79 0,0804 0,4633 2,40 0,0224 0,4918
1,39 0,1518 0,4177 1,80 0,0790 0,4641 2,42 0,0213 0,4922
1,40 0,1497 0,4192 1,81 0,0775 0,4649 2,44 0,0203 0,4927
1,41 0,1476 0,4207 1,82 0,0761 0,4656 2,46 0,0194 0,4931
1,42 0,1456 0,4222 1,83 0,0748 0,4664 2,48 0,0184 0,4934
1,43 0,1435 0,4236 1,84 0,0734 0,4671 2,50 0,0175 0,4938
1,44 0,1415 0,4251 1,85 0,0721 0,4678 2,52 0,0167 0,4941
1,45 0,1394 0,4265 1,86 0,0707 0,4686 2,54 0,0158 0,4945
1,46 0,1374 0,4279 1,87 0,0694 0,4693 2,56 0,0151 0,4948
1,47 0,1354 0,4292 1,88 0,0681 0,4699 2,58 0,0143 0,4951
1,48 0,1334 0,4306 1,89 0,0669 0,4706 2,60 0,0136 0,4953
2,62 0,0129 0,4956 2,88 0,0063 0,4980
2,70 0,0122 0,4959 2,90 0,0060 0,4981 3,50 0,00087 0,49977
2,66 0,0116 0,4961 2,92 0,0056 0,4982
2,68 0,0110 0,4963 2,94 0,0053 0,4984 3,60 0,00061 0,499841
2,70 0,0104 0,4965 2,96 0,0050 0,4985 3,70 0,00042 0,49989
2,72 0,0099 0,4967 2,98 0,0047 0,4986 3,80 0,00029 0,499928
2,74 0,0093 0,4969 3,00 0,00443 0,49865
2,76 0,0088 0,4971 3,90 0,00020 0,49995
2,78 0,0084 0,4973 3,10 0,00327 0,49903 4,00 0,0001338 0,499968
2,80 0,0079 0,4974 3,20 0,00238 0,49931
2,82 0,0075 0,4976 4,50 0,0000160 0,499997
2,84 0,0071 0,4977 3,30 0,00172 0,49952 5,00 0,0000015 0,49999997
2,86 0,0067 0,4979 3,40 0,00123 0,49966

263
Appendix 2
Table of values of tγ = t(γ, n)
γ γ
0,95 0,99 0,999 0,95 0,99 0,999
n n
5 2,78 4,60 8,61 20 2,093 2,861 3,883
6 2,57 4,03 6,86 25 2,064 2,797 3,745
7 2,45 3,71 5,96 30 2,045 2,756 3,659
8 2,37 3,50 5,41 35 2,032 2,720 3,600
9 2,31 3,36 5,04 40 2,023 2,708 3,558
10 2,26 3,25 4,78 45 2,016 2,692 3,527
11 2,23 3,17 4,59 50 2,009 2,679 3,502
12 2,20 3,11 4,44 60 2,001 2,662 3,464
13 2,18 3,06 4,32 70 1,996 2,649 3,439
14 2,16 3,01 4,22 80 1,991 2,640 3,418
15 2,15 2,98 4,14 90 1,987 2,633 3,403
16 2,13 2,95 4,07 100 1,984 2,627 3,392
17 2,12 2,92 4,02 120 1,980 2,617 3,374
18 2,11 2,90 3,97 ∞ 1,960 2,576 3,291
19 2,10 2,88 3,92
Appendix 3
Table of values of q = q(γ, n)
γ γ
0,95 0,99 0,999 0,95 0,99 0,999
n n
5 1,37 2,67 5,64 20 0,37 0,58 0,88
6 1,09 2,01 3,88 25 0,32 0,49 0,73
7 0,92 1,62 2,98 30 0,28 0,43 0,63
8 0,80 1,38 2,42 35 0,26 0,38 0,56
9 0,71 1,20 2,06 40 0,24 0,35 0,50
10 0,65 1,08 1,80 45 0,22 0,32 0,46
11 0,59 0,98 1,60 50 0,21 0,30 0,43
12 0,55 0,90 1,45 60 0,188 0,269 0,38
13 0,52 0,83 1,33 70 0,174 0,245 0,34

264
Appendix 4

γ γ
0,95 0,99 0,999 0,95 0,99 0,999
n n
14 0,48 0,78 1,23 80 0,161 0,226 0,31
15 0,46 0,73 1,15 90 0,151 0,211 0,29
16 0,44 0,70 1,07 100 0,143 0,198 0,27
17 0,42 0,66 1,01 150 0,115 0,160 0,211
18 0,40 0,63 0,96 200 0,099 0,136 0,185
19 0,39 0,60 0,92 250 0,089 0,120 0,162

Appendix 4
Critical points of the distribution χ 2

The The significance level


number of
freedom 0,01 0,025 0,05 0,95 0,975 0,99
degrees k
1 6,6 5,0 3,8 0,0039 0,00098 0,00016
2 9,2 7,4 6,0 0,103 0,051 0,020
3 11,3 9,4 7,8 0,352 0,216 0,115
4 13,3 11,1 9,5 0,711 0,484 0,297
5 15,1 12,8 11,1 1,15 0,831 0,554
6 16,8 14,4 12,6 1,64 1,24 0,872
7 18,5 16,0 14,1 2,17 1,69 1,24
8 20,1 17,5 15,5 2,73 2,18 1,65
9 21,7 19,0 16,9 3,33 2,70 2,09
10 23,2 20,5 18,3 3,94 3,25 2,56
11 24,7 21,9 19,7 4,57 3,82 3,05
12 26,2 23,3 21,0 5,23 4,40 3,57
13 27,7 24,7 22,4 5,89 5,01 4,11
14 29,1 26,1 23,7 6,57 5,63 4,66
15 30,6 27,5 25,0 7,26 6,26 5,23
16 32,0 28,8 26,3 7,96 6,91 5,81
17 33,4 30,2 27,6 8,67 7,56 6,41
18 34,8 31,5 28,9 9,39 8,23 7,01

265
The The significance level
number of
freedom 0,01 0,025 0,05 0,95 0,975 0,99
degrees k
19 36,2 32,9 30,1 10,1 8,91 7,63
20 37,6 34,2 31,4 10,9 9,59 8,26
21 38,9 35,5 32,7 11,6 10,3 8,90
22 40,3 36,8 33,9 12,3 11,0 9,54
23 41,6 38,1 35,2 13,1 11,7 10,2
24 43,0 39,4 36,4 13,8 12,4 10,9
25 44,3 40,6 37,7 14,6 13,1 11,5
26 45,6 41,9 38,9 15,4 13,8 12,2
27 47,0 43,2 40,1 16,2 14,6 12,9
28 48,3 44,5 41,3 16,9 15,3 13,6
29 49,6 45,7 42,6 17,7 16,0 14,3
30 50,9 47,0 43,8 18,5 16,8 15,0

266
Appendix 5 (Part 1)
Critical points of the Fisher-Snedecor distribution F
(k1 – the number of freedom degrees of greater dispersion,
k2 – the number of freedom degrees of smaller dispersion)
The significance level α = 0,01
k1
1 2 3 4 5 6 7 8 9 10 11 12
k2
1 4052 4999 5403 5625 5764 5889 5928 5981 6022 6056 6082 6106
2 98,49 99,01 99,17 99,25 99,30 99,33 99,34 99,36 99,38 99,40 99,41 99,42
3 34,12 30,81 29,46 28,71 28,24 27,91 27,67 27,49 27,34 27,23 27,13 27,05
4 21,20 18,00 16,69 15,98 15,52 15,21 14,98 14,80 14,66 14,54 14,45 14,37
5 16,26 13,27 12,06 11,39 10,97 10,67 10,45 10,27 10,15 10,05 9,96 9,89
6 13,74 10,92 9,78 9,15 8,75 8,47 8,26 8,10 7,98 7,87 7,79 7,72
7 12,25 9,55 8,45 7,85 7,46 7,19 7,00 6,84 6,71 6,62 6,54 6,47
8 11,26 8,65 7,59 7,01 6,63 6,37 6,19 6,03 5,91 5,82 5,74 5,67
9 10,56 8,02 6,99 6,42 6,06 5,80 5,62 5,47 5,35 5,26 5,18 5,11
10 10,04 7,56 6,55 5,99 5,64 5,39 5,21 5,06 4,95 4,85 4,78 4,71
11 9,86 7,20 6,22 5,67 5,32 5,07 4,88 4,74 4,63 4,54 4,46 4,40
12 9,33 6,93 5,95 5,41 5,06 4,82 4,65 4,50 4,39 4,30 4,22 4,16
13 9,07 6,70 5,74 5,20 4,86 4,62 4,44 4,30 4,19 4,10 4,02 3,96
14 8,86 6,51 5,56 5,03 4,69 4,46 4,28 4,14 4,03 3,94 3,86 3,80
15 8,68 6,36 5,42 4,89 4,56 4,32 4,14 4,00 3,89 3,80 3,73 3,67
16 8,53 6,23 5,29 4,77 4,44 4,20 4,03 3,89 3,78 3,69 3,61 3,55
17 8,40 6,11 5,18 4,67 4,34 4,10 3,93 3,79 3,68 3,59 3,52 3,45

267
Appendix 5 (Part 1)
Appendix 5 (Part 2)

268
Critical points of the Fisher-Snedecor distribution F
(k1 – the number of freedom degrees of greater dispersion,
k2 – the number of freedom degrees of smaller dispersion)
The significance level

1 2 3 4 5 6 7 8 9 10 11 12
1 161 200 216 225 230 234 237 239 241 242 243 244
2 18,51 19,00 19,16 19,25 19,30 19,33 19,36 19,37 19,38 19,39 19,40 19,41
3 10,13 9,55 9,28 9,12 9,01 8,94 8,88 8,84 8,81 8,78 8,76 8,74
4 7,71 6,94 6,59 6,39 6,26 6,16 6,09 6,04 6,00 5,96 5,93 5,91
5 6,61 5,79 5,41 5,19 5,05 4,95 4,88 4,82 4,78 4,74 4,70 4,68
6 5,99 5,14 4,76 4,53 4,39 4,28 4,21 4,15 4,10 4,06 4,03 4,00
7 5,59 4,74 4,35 4,12 3,97 3,87 3,79 3,73 3,68 3,63 3,60 3,57
8 5,32 4,46 4,07 3,84 3,69 3,58 3,50 3,44 3,39 3,34 3,31 3,28
9 5,12 4,26 3,86 3,63 3,48 3,37 3,29 3,23 3,18 3,13 3,10 3,07
10 4,96 4,10 3,71 3,48 3,33 3,22 3,14 3,07 3,02 2,97 2,94 2,91
11 4,84 3,98 3,59 3,36 3,20 3,09 3,01 2,95 2,90 2,86 2,82 2,79
12 4,75 3,88 3,49 3,26 3,11 3,00 2,92 2,85 2,80 2,76 2,72 2,69
13 4,67 3,80 3,41 3,18 3,02 2,92 2,84 2,77 2,72 2,67 2,63 2,60
14 4,60 3,74 3,34 3,11 2,96 2,85 2,77 2,70 2,65 2,60 2,56 2,53
15 4,54 3,68 3,29 3,06 2,90 2,79 2,70 2,64 2,59 2,55 2,51 2,48
16 4,49 3,63 3,24 3,01 2,85 2,74 2,66 2,59 2,54 2,49 2,45 2,42
17 4,45 3,59 3,20 2,96 2,81 2,70 2,62 2,55 2,50 2,45 2,41 2,38
LIST OF THE USED BOOKS
LIST OF THE USED BOOKS

1. Гмурман В.Е., Теория вероятностей и математическая


статистика. Издание седьмое, стереотипное, Москва: «Высшая
школа», 2000.
2. Сборник задач. Математика для экономистов. Под редакцией
профессора Кельтеновой Р.Т., Алматы: «Экономика», 2000.
3. Горелова Г.В., Кацко И.А., Теория вероятностей и
математическая статистика в примерах и задачах с применением
Excel. – Издание четвертое, Ростов на Дону: «Феникс», 2006.
4. Коршунов Д.А., Фосс С.Г., Сборник задач и упражнений по
теории вероятностей, Новосибирск, 1997.
5. Коршунов Д.А., Чернова Н.И., Сборник задач и упражнений по
математической статистике, Новосибирск. 2001.
6. Кремер Н.Ш., Теория вероятностей и математическая
статистика: Учебник для ВУЗов. — 2-е изд., перераб. и доп. М:
ЮНИТИ-ДАНА, 2004. — 573 с.
7. Данко П.Е., Попов А.Г., Кожевникова Т.Я., Высшая математика
в упражнениях и задачах. Издание пятое, исправленное. –
Москва: «Высшая школа», 1999, часть 2.
8. Манита А.Д., Теория вероятностей и математическая
статистика: Учебное пособие. – М.: Издательский отдел
УНЦ ДО, 2001, 120 с.
9. Гмурман В.Е., Руководство к решению задач по теории
вероятностей и математической статистике: Учебное
пособие — 11-е изд., перераб. — М.: Высшее образование,
2006, 404 с.
10. Olav Kallenberg, Foundations of Modern Probability, second
edition, Springer Series in Statistics, 2002, 650 pages.
11. Olav Kallenberg, Probabilistic Symmetries and Invariance
Principles, Springer-Verlag, New York, 2005, 510 pages.

269
12. Hopkins W.G., A New View of Statistics, 2000, Internet Society
for Sport Science: http://www.sportsci.org/resource/stats/.
13. Sheldon Ross, A First Course in Probability, seventh edition,
University of Southern California, Pearson Education International,
2006, 561 pages.

270
CONTENTS
CONTENTS
INTRODUCTION. . . . . . . . . . . . . . . . . . . . . . . . . . . . . . . . . . . . . . . . . . . . 3
LECTURE 1. Basic Concepts of Probability Theory. . . . . . . . . . . . . . . . . 5
Classical Definition of Probability . . . . . . . . . . . . . . . . . . . . . . . . . . . . . . . . 6
Relative Frequency . . . . . . . . . . . . . . . . . . . . . . . . . . . . . . . . . . . . . . . . . . . 8
Geometric Probabilities . . . . . . . . . . . . . . . . . . . . . . . . . . . . . . . . . . . . . . . . 9
Exercises . . . . . . . . . . . . . . . . . . . . . . . . . . . . . . . . . . . . . . . . . . . . . . . . . . 11

LECTURE 2. Basic Formulas of Combinatorial Analysis . . . . . . . . . . . 15


Operations over Events . . . . . . . . . . . . . . . . . . . . . . . . . . . . . . . . . . . . . . . 19
Exercises . . . . . . . . . . . . . . . . . . . . . . . . . . . . . . . . . . . . . . . . . . . . . . . . . . 20

LECTURE 3. Theorem of Addition of Probabilities


of Incompatible Events . . . . . . . . . . . . . . . . . . . . . . . . . . . . . . . . . . . . . . . 25
Complete Group of Events . . . . . . . . . . . . . . . . . . . . . . . . . . . . . . . . . . . . . 26
Opposite Events . . . . . . . . . . . . . . . . . . . . . . . . . . . . . . . . . . . . . . . . . . . . . 26
Conditional Probability. . . . . . . . . . . . . . . . . . . . . . . . . . . . . . . . . . . . . . . . 27
Theorem of Multiplication of Probabilities . . . . . . . . . . . . . . . . . . . . . . . . 28
Exercises . . . . . . . . . . . . . . . . . . . . . . . . . . . . . . . . . . . . . . . . . . . . . . . . . . 30

LECTURE 4. Independent Events . . . . . . . . . . . . . . . . . . . . . . . . . . . . . . 34


Probability of Appearance of At Least One Event . . . . . . . . . . . . . . . . . . . 37
Exercises . . . . . . . . . . . . . . . . . . . . . . . . . . . . . . . . . . . . . . . . . . . . . . . . . . 40

LECTURE 5. Theorem of Addition of Probabilities


of Compatible Events . . . . . . . . . . . . . . . . . . . . . . . . . . . . . . . . . . . . . . . . . 43
Formula of Total Probability. . . . . . . . . . . . . . . . . . . . . . . . . . . . . . . . . . . . 44
Probability of hypotheses. Bayes’s formulas . . . . . . . . . . . . . . . . . . . . . . . 46
Exercises . . . . . . . . . . . . . . . . . . . . . . . . . . . . . . . . . . . . . . . . . . . . . . . . . . 48

LECTURE 6. Repetition (Recurrence) of Trials.


The Bernoulli Formul. . . . . . . . . . . . . . . . . . . . . . . . . . . . . . . . . . . . . . . . . 52
Local Theorem of Laplace . . . . . . . . . . . . . . . . . . . . . . . . . . . . . . . . . . . . . 54
Integral Theorem of Laplace. . . . . . . . . . . . . . . . . . . . . . . . . . . . . . . . . . . . 56
Exercises . . . . . . . . . . . . . . . . . . . . . . . . . . . . . . . . . . . . . . . . . . . . . . . . . . 58

271
LECTURE 7. Random Variables. The Law of Distribution of a Discrete
Random Variable. . . . . . . . . . . . . . . . . . . . . . . . . . . . . . . . . . . . . . . . . . . . . 62
Mathematical Operations over Random Variables. . . . . . . . . . . . . . . . . . . 65
(Mathematical) Expectation of a Discrete Random Variable. . . . . . . . . . . 67
Dispersion of a Discrete Random Variable. . . . . . . . . . . . . . . . . . . . . . . . . 69
Exercises . . . . . . . . . . . . . . . . . . . . . . . . . . . . . . . . . . . . . . . . . . . . . . . . . . 73

LECTURE 8. Distribution Function of a Random Variable. . . . . . . . . . . 77


Properties of a Distribution Function. . . . . . . . . . . . . . . . . . . . . . . . . . . . . 78
Continuous Random Variables. Probability Density.. . . . . . . . . . . . . . . . . 79
Properties of Probability Density. . . . . . . . . . . . . . . . . . . . . . . . . . . . . . . . 80
Exercises . . . . . . . . . . . . . . . . . . . . . . . . . . . . . . . . . . . . . . . . . . . . . . . . . . 83

LECTURE 9. Basic Laws of Distribution of Discrete Random Variables.90


Binomial Law of Distribution. . . . . . . . . . . . . . . . . . . . . . . . . . . . . . . . . . . 90
The Law of Distribution of Poisson.. . . . . . . . . . . . . . . . . . . . . . . . . . . . . . 91
Geometric Distribution . . . . . . . . . . . . . . . . . . . . . . . . . . . . . . . . . . . . . . . 94
Hypergeometric Distribution . . . . . . . . . . . . . . . . . . . . . . . . . . . . . . . . . . . 95
Exercises . . . . . . . . . . . . . . . . . . . . . . . . . . . . . . . . . . . . . . . . . . . . . . . . . . 97

LECTURE 10. Basic Laws of Distribution of Continuous


Random Variables. . . . . . . . . . . . . . . . . . . . . . . . . . . . . . . . . . . . . . . . . . . 101
Uniform Law of Distribution . . . . . . . . . . . . . . . . . . . . . . . . . . . . . . . . . . 101
Exponential Law of Distribution.. . . . . . . . . . . . . . . . . . . . . . . . . . . . . . . 102
Normal Law of Distribution. . . . . . . . . . . . . . . . . . . . . . . . . . . . . . . . . . . 104
Exercises . . . . . . . . . . . . . . . . . . . . . . . . . . . . . . . . . . . . . . . . . . . . . . . 108

LECTURE 11. The law of Large Numbers and Limit Theorems. . . . . . 113
The Central Limit Theorem . . . . . . . . . . . . . . . . . . . . . . . . . . . . . . . . . . . 118
Exercises . . . . . . . . . . . . . . . . . . . . . . . . . . . . . . . . . . . . . . . . . . . . . . . . . 120

LECTURE 12. Mathematical Statistics. Variation Series and Their


Characteristics.. . . . . . . . . . . . . . . . . . . . . . . . . . . . . . . . . . . . . . . . . . . . . 126
Numerical Characteristics of Variation Series.. . . . . . . . . . . . . . . . . . . . . 133
Exercises . . . . . . . . . . . . . . . . . . . . . . . . . . . . . . . . . . . . . . . . . . . . . . . . . 144

272
CONTENTS
LECTURE 13. Statistical Estimations of Parameters
of Distribution . . . . . . . . . . . . . . . . . . . . . . . . . . . . . . . . . . . . . . . . . . . . . 152
Pointwise Estimators of Mathematical Expectation and Dispersion . . . . 155
Exercises . . . . . . . . . . . . . . . . . . . . . . . . . . . . . . . . . . . . . . . . . . . . . . . 160

LECTURE 14. Methods of Finding of Estimations.. . . . . . . . . . . . . . . . 163


Method of Moments .. . . . . . . . . . . . . . . . . . . . . . . . . . . . . . . . . . . . . . . . 163
Method of Maximal (the Greatest) Plausibility.. . . . . . . . . . . . . . . . . . . . 165
Exercises . . . . . . . . . . . . . . . . . . . . . . . . . . . . . . . . . . . . . . . . . . . . . . 170

LECTURE 15. Interval Estimators of Parameters. Testing of Statistical


Hypotheses . . . . . . . . . . . . . . . . . . . . . . . . . . . . . . . . . . . . . . . . . . . . . . . . 177
Interval Estimators of Parameters . . . . . . . . . . . . . . . . . . . . . . . . . . . . . . 177
Testing of Statistical Hypotheses . . . . . . . . . . . . . . . . . . . . . . . . . . . . . . . 182
Exercises . . . . . . . . . . . . . . . . . . . . . . . . . . . . . . . . . . . . . . . . . . . . . . . . . 190

INDIVIDUAL HOME WORKS . . . . . . . . . . . . . . . . . . . . . . . . . . . . . . 197

TESTS FOR SELF-CHECKING . . . . . . . . . . . . . . . . . . . . . . . . . . . . . 247

APPENDICES . . . . . . . . . . . . . . . . . . . . . . . . . . . . . . . . . . . . . . . . . . . . 261

LIST OF THE USED BOOKS . . . . . . . . . . . . . . . . . . . . . . . . . . . . . . . 269

273
B.Sh. Kulpeshov

Short course of probability theory


and mathematical statistics

Кулпешов Бейбут Шайыкович

Краткий курс теории вероятностей


и математической статистики

Утверждено к печати Ученым Советом Международного


Университета Информационных Технологий Министерства
образования и науки Республики Казахстан.

Технический редактор: Н.Ж. Абдрахманова


Дизайн и верстка: В.В. Афонина

Подписано в печать 29.03.2017 г.


Тираж 500 экз. Формат 60x88/16. Заказ № .

Книга отпечатана в типографии «Ак-Шагыл»


г.Алматы, ул. Муратбаева 138

You might also like